Sunteți pe pagina 1din 138

CLASA a-X-a

1. Metode trigonometrice în rezolvarea problemelor de algebră


(E.Jecan)

2. Şiruri şi progresii (E.Jecan, I.Magdaş)


2.1. Progresii aritmetice
2.2. Progresii geometrice
2.3. Şiruri recurente. Probleme rezolvate şi probleme propuse

3. Numere complexe în algebră (N.Muşuroia)

4. Aplicaţii ale numerelor complexe în geometrie (N.Muşuroia)

5. Ecuaţii în mulţimea numerelor complexe (N.Muşuroia)

6. Metoda vectorială în rezolvarea problemelor de algebră (E.Jecan,


V.Lupşor)
6.1. Probleme cu vectori
6.2. Aplicaţii ale produsului scalar
6.3. Probleme rezolvate
6.4. Inegalităţi deduse din produs scalar
6.5. Sisteme de ecuaţii

7. Ecuaţii exponenţiale şi logaritmice nonstandard (N.Muşuroia)


7.1. Utilizarea monotoniei
7.2. Probleme rezolvate
7.3. Utilizarea inegalităţilor clasice şi a convexităţii
7.4. Probleme rezolvate

8. Probleme de numărare (Gh.Boroica, V.Pop)

9. Sume combinatorice (Gh.Lobonţ)


9.1. Noţiuni teoretice
9.2. Metode de calcul al sumelor cu combinări

10. Probleme de geometrie în spaţiu (Gh.Boroica, N.Muşuroia)

11. Criterii de ireductibilitata pentru polinoame (Gh.Lobonţ)


11.1. Criteriul lui Eisenstein şi criteriul lui Schöneman
11.2. Aplicaţii ale criteriilor de ireductibilitate ale lui Eisenstein şi
Schöneman

1
Coordonator Vasile Pop
Viorel Lupşor

2
MATEMATICĂ
PROGRAMA ŞCOLARA PENTRU CLASELE DE EXCELENŢA
X

ARGUMENT

Studiul matematicii prin clasele de excelenţă, urmăreşte în principal


crearea unui cadru organizat, în care elevii talentaţi la matematică, proveniţi din
diferite medii şcolare, să poată intra în contact, şi în timp relativ scurt, să
formeze un grup performant. Aceşti elevi, beneficiind de o pregătire pe măsura
potenţialului lor intelectual, vor contribui ulterior la formarea unei elite
româneşti în domeniul matematicii.
Realizarea unei programe pentru clasele de excelenţă, precum şi modul
în care se va lucra pe această programă, constituie o noutate pentru
învăţământul românesc. Din acest motiv elaborarea prezentei programe trebuie
înţeleasă ca o etapă necesară unui început de drum.
Un colectiv de cadre didactice din învăţământul preuniversitar şi
universitar din CRTCP Cluj, cu experienţă în domeniul pregătirii elevilor
capabili de performanţe superioare, au format o echipă care a realizat programa
şi manualul care conţine exerciţii şi probleme extrem de utile pentru
desăvârşirea pregătirii acestor elevi.
În selectarea conţinuturilor programei s-a ţinut cont de tendinţele actuale
în formularea subiectelor la concursurile şi olimpiadele şcolare, dar şi de
tradiţiile şcolii româneşti de matematică. Numeroasele cărţi şi reviste adresate
„vârfurilor” au constituit o importantă sursă bibliografică în tratarea temelor.
Temele propuse constituie o extindere firească a programei analitice obligatorii
de matematică şi parcurgerea lor este necesară pentru abordarea unor probleme
mai dificile. Anumite teme vor fi tratate pe parcursul mai multor ani de studiu
(evident cu o problematică corespunzătoare) asigurându-se astfel continuitatea
şi coerenţa procesului de învăţare. Mai trebuie precizat că la elaborarea
programei echipa a avut în vedere faptul că matematica nu este un produs finit,
ci un proces intelectual în care, pe suportul unor cunoştinţe solide, primează
iniţiativa personală. Astfel, această programă oferă posibilităţi autentice de
opţiune pentru profesori şi elevi.
Programa se adresează elevilor claselor X-XII şi a fost concepută pentru
un număr de 2 ore/săptămână (în cele 30 de săptămâni ale anului şcolar în care
se lucrează cu clasele sau grupele de excelenţă). Ca o completare la programa
obligatorie de matematică, competenţelor generale le-au mai fost adăugate încă
două care au rolul de a orienta demersul didactic către formarea unor
ansambluri structurate de cunoştinţe generate de specificul activităţii intelectua-

5
le matematice la nivel de performanţe superioare. Programa are următoarele
componente:
- competenţe generale
- competenţe specifice şi conţinuturile corelate cu acestea
- valori şi atitudini
- sugestii metodologice.

Competenţe generale

1. Folosirea corectă a terminologiei specifice matematicii în


contexte variate
2. Prelucrarea datelor de tip cantitativ, calitativ, structural,
contextual cuprinse în enunţuri matematice
3. Utilizarea corectă a algoritmilor matematici în rezolvarea de
probleme cu grade diferite de dificultate
4. Exprimarea şi redactarea corectă şi coerentă în limbaj formal
sau în limbaj cotidian, a rezolvării sau a strategiilor de rezolvare
a unei probleme
5. Analiza unei situaţii problematice şi determinarea ipotezelor
necesare pentru obţinerea concluziei
6. Generalizarea unor proprietăţi prin modificarea contextului
iniţial de definire a problemei sau prin îmbunătăţirea sau
generalizarea algoritmilor
7. Emiterea unor judecăţi de valoare pentru rezolvarea probleme-
lor inventiv şi euristic-creative
8. Dobândirea unei imagini de ansamblu a matematicii elementare
ca parte a unui sistem aflat în permanentă evoluţie şi
interacţiune cu lumea înconjurătoare

6
Competenţe specifice Conţinuturi
1.Observarea şi diferenţierea diferite- Metode de rezolvare a problemelor
lor tipuri de ecuaţii, inecuaţii, inega- de algebră
lităţi, identităţi şi sisteme de ecuaţii • Metoda trigonometrică
2. Identificarea unor identităţi şi inega- • Metoda vectorială
lităţi clasice şi rezolvarea de probleme • Metode de rezolvare a ecua-
pe baza acestora ţiilor, inecuaţiilor şi sistemelor
3. Alegerea modelului matematic al exponenţiale şi logaritmice
unei probleme algebrice utilizând tri- nestandard
gonometria, vectorii sau numerele
complexe în scopul optimizării efectu- Şiruri şi progresii
ării unor calcule
4.1. Transpunerea în limbajul nume- Numere complexe
relor complexe a unor proprietăţi • Numere complexe în algebră
algebrice şi geometrice • Numere complexe în geome-
4.2. Stabilirea de condiţii necesare şi trie
suficiente ca un şir să fie progresie • Ecuaţii în C
aritmetică sau geometrică
5.1. Utilizarea proprietăţilor unor Elemente de combinatorică
funcţii (monotonie, convexitate) în
• Probleme de numărare
scopul rezolvării de ecuaţii, inecuaţii,
(de ex. : determinarea numărului
sisteme exponenţiale şi logaritmice
de funcţii, de submulţimi, de
nestandard
puncte din plan sau spaţiu cu
5.2.Determinarea unor polinoame sau
anumite proprietăţi)
a numărului de elemente ale unei mul-
• Sume combinatorice
ţimi care satisfac anumite condiţii date
(de ex. : formula lui Li- Jen- Shu,
6. Utilizarea elementelor de combina-
formula lui Dixon, identităţile lui
torică şi progresiile la calculul unor
Abel)
sume
7. Realizarea unor implicaţii între
Probleme de geometrie în spaţiu
problemele tipice cu progresii, funcţii
exponenţiale şi logaritmice, polinoa-
Polinoame
me, combinatorică, probleme de
• Criterii de ireductibilitate
geometrie în spaţiu şi cele propuse la
pentru polinoame
concursurile şi olimpiadele şcolare
(Criteriul lui Eisenstein şi al lui
8.1. Conştientizarea şi abilitatea
Schőnemann)
utilizării unei varietăţi de metode care
stau la baza rezolvării ecuaţiilor,
inecuaţiilor, inegalităţilor şi sistemelor
de ecuaţii

7
8.2. Realizarea de conexiuni între
algebră, vectori, geometrie şi trigono-
metrie prin rezolvarea problemelor
utilizând diferite tehnici

VALORI ŞI ATITUDINI

Noul curriculum şcolar pentru clasele de excelenţă propus la matematică


are în vedere formarea la elevi a următoarelor valori şi atitudini în plus faţă de
cele specificate prin curriculumul şcolar obligatoriu :
• Manifestarea unor opinii competente cu privire la abordarea
problemelor intuitiv şi euristic-creative bazate pe explorare, inspiraţie şi
invenţie
• Dezvoltarea unei gândiri reflexive, independente, flexibilă şi abstractă
specifică matematicii
• Interesul pentru modul de dezvoltare a ideilor şi rezultatelor matematice
• Curiozitatea faţă de noile deschideri din domeniul matematicii

SUGESTII METODOLOGICE

Prin prezentul curriculum pentru clasele de excelenţă se intenţionează


ca, pe parcursul liceului, elevii să dobândească competenţe şi să-şi structureze
un set de valori şi atitudini specifice pregătirii de înaltă performanţă. Acestea se
regăsesc în următoarele aspecte ale învăţării, vizate de practica pedagogică :
• Analizarea şi elaborarea unui plan de rezolvare pentru problemele
atipice şi/sau dificile din domeniile studiate
• Formarea obişnuinţei de a formula probleme şi situaţii problemă
• Analiza unei probleme din punct de vedere al ideii centrale
• Reparcurgerea căii de rezolvare a problemei pentru a obţine un rezultat
mai bun, ameliorat sau optimizat printr-o reproiectare creativă
• Identificarea unor metode de lucru valabile pentru clase de probleme
• Iniţeirea şi realizarea creativă a unei investigaţii pornind de la tematica
propusă
• Formarea deprinderii de a anticipa rezultate matematice pornind de la
datele existente
• Formarea obişnuinţei de a face conexiuni intra şi interdisciplinare

8
Acest curriculum are drept obiectiv ca fiecare elev capabil de performanţe
superioare să-şi poată dezvolta competenţele într-un ritm individual, de a-şi
transfera cunoştinţele acumulate dintr-o zonă de studiu în alta. Pentru aceasta se
recomandă următoarele activităţi :
• Alternarea prezentării conţinuturilor, cu moduri variate de antrenare a
gândirii
• Solicitarea de frecvente corelaţii intra şi interdisciplinare
• Punerea elevului în situaţia ca el însuşi să formuleze sarcini de lucru
adecvate
• Obţinerea de soluţii sau interpretări variate pentru aceeaşi unitate
informaţională
• Prevederea de sarcini rezolvabile prin activitatea în grup
• Utilizarea unor softuri educaţionale

Având în vedere specificul claselor de excelenţă, metodele folosite in practice


instructiv-educativă vizează următoarele aspecte:
• Utilizarea strategiilor euristice, care lasăelevul să-şi asume riscul
incertitudinii, al încercării şi erorii, specifice investigaţiei ştiinţifice
• Utilizarea strategiilor creative, care lasă elevul să se afirme în planul
originalităţii, spontaneităţii, diversităţii şi care pun accentul pe
capacitatea de reflecţie, sinteză, evaluare critică şi creaţie
• O îmbinare şi o alternanţă sistematică a activităţii bazate pe efort
individual cu cele care solicită efort colectiv
• Însuşirea unor metode de informare şi de documentare independentă,
care oferă deschiderea spre autoinstruire şi spre învăţarea continuă

9
1. Metoda trigonometrică in rezolvarea problemelor de algebră

1.1. Introducere

O cale ingenioasă de rezolvare a unor probleme de algebră este


transformarea lor în probleme de trigonometrie, folosind substituţii potrivite.
Astfel, unele ecuaţii, sisteme, identităţi sau inegalităţi devin mai uşor
abordabile folosind cunoştinţe de trigonometrie. Pentru înţelegerea şi însuşirea
metodei trigonometrice este necesară cunoaşterea tuturor identităţilor studiate
precum şi a proprietăţilor funcţiilor trigonometrice directe şi a celor inverse.
În aplicarea metodei este util să se ţină seama de mulţimea valorilor
variabilelor care urmează a fi substituite, pentru ca substituţiile folosite să fie
într-adevăr operante.
În cele ce urmează vom exemplifica metoda trigonometrică la rezolvarea
unor identităţi, inegalităţi, ecuaţii şi sisteme de ecuaţii.

Bibliografie

• V.Tudor, Probleme de algebră cu rezolvări ingenioase, Ed. Carminis,


Piteşti, 1999
• M.Cocuz, Culegere de probleme de matematică, Ed.Academiei, 1984.
• M. Bălună, Zece lecţii alese de matematică, Ed S.S.M.,1998

13
Probleme rezolvate (1)

A. IDENTITĂŢI

R1.2.1. Dacă x, y, z ∈ R , cu proprietatea x + y + z = xyz , şi dacă nici unul


1
dintre ele nu este egal cu , să se demonstreze că:
3
3x − x 3 3 y − y 3 3z − z 3
+ + =
( )(
3x − x 3 3 y − y 3 3z − z 3)(
.
)
1 − 3x 2 1 − 3 y 2 1 − 3z 2 ( )( )(
1 − 3x 2 1 − 3 y 2 1 − 3z 2 )
Soluţie:
Forma expresiei de demonstrat ne sugerează folosirea identităţii
tgA + tgB + tgC = = tgA ⋅ tgB ⋅ tgC valabilă în orice triunghi nedreptunghic
 π π
ABC . Notăm x = tga , y = tgb , z = tgc , unde a, b, c ∈  − ,  . Avem
 2 2
xy ≠ 1 , deoarece, dacă xy = 1 , din x + y + z = xyz ar rezulta x + y = 0 şi deci
x 2 = −1 imposibil. Cum tga ⋅ tgb ≠ 1 , are sens relaţia
tga + tgb x+ y
tg (a + b ) = ⇔ tg (a + b ) = = − z = tg (− c ) , deci a + b + c = kπ ,
1 − tga ⋅ tgb 1 − xy
 π π
k ∈ Z şi cum a, b, c ∈  − ,  , k ∈ {− 1,0,1} . Reciproc, dacă avem trei
 2 2
unghiuri a, b, c diferite de
(2k + 1)π , k ∈ Z şi a + b = kπ − c ,
2
tga + tgb
= tg (a + b ) = −tgc , de unde tga + tgb + tgc = tga ⋅ tgb ⋅ tgc . Dacă
1 − tga ⋅ tgb

tgu ≠ ±
1
şi u≠
(2k + 1)π , ∀k ∈ Z , are sens tg 3u =
3tgu − tg 3 u
şi cum
3 2 1 − 3tg 2 u
3a + 3b + 3c = 3kπ , obţinem tg (3a + 3b + 3c ) = 0 , de unde tg 3a + tg 3b + tg 3c =
= tg 3a ⋅ tg 3b ⋅ tg 3c care este echivalentă cu egalitatea propusă.

Observaţie
1.Pornind de la identitatea

tg (a + b + c + d ) =
∑ tga − ∑ tga ⋅ tgb ⋅ tgc , obţinem
1 + tga ⋅ tgb ⋅ tgc ⋅ tgd − ∑ tga ⋅ tgb

tg 4 x =
( )
4tg 1 − tg 2 x
. Cu acestea putem obţine şi alte identităţi condiţionate
1 − 6tg 2 x + tg 4 x

14
cum ar fi: Dacă x + y + z + t = xyz + yzt + ztx + txy , atunci
X + Y + Z + T = XYZ + ZYT + ZTX + TXY (sau ∑ X = ∑ XYZ ), unde

X =
(
4x 1 − x 2 ) şi analoagele pentru Y , Z , T . Observăm că dacă
1 − 6x 2 + x 4
+ b + c + d = π , atunci ∑ tga = ∑ tga ⋅ tgb ⋅ tgc ;
∑ X − ∑ XYZ =
tg (4a + 4b + 4c + 4d ) =
1 + XYZT − ∑ XY
= tg 4π = 0 ⇒ ∑ X = ∑ XYZ .

2. Pornind de la aceeaşi formulă putem rezolva următoarea problemă:


Pentru toate valorile admisibile ale variabilelor reale a, b, c să se arate
a−b b−c c−a a−b b−c c−a
că + + = ⋅ ⋅ . Notând a = tgα , b = tgβ ,
1 + ab 1 + bc 1 + ac 1 + ab 1 + bc 1 + ca
 π π tgu − tgv
c = tgχ , α , β , χ ∈  − ,  , cu ajutorul formulei tg (u − v ) = ,
 2 2 1 + tgu ⋅ tgv
deducem relaţia
tg (α − β ) + tg (β − χ ) + tg (χ − α ) = tg (α − β ) ⋅ tg (β − χ ) ⋅ tg (χ − α ) care este
echivalentă cu egalitatea din enunţ.

R1.2.2. Dacă a 2 + b 2 ≤ 4 , a ≤ b , a, b ∈ [0,2] , să se demonstreze egalitatea:


2 2
 a b 4 − a2 − b2   2 
 + 1−  a − b 4 − a − b  = b
2
1−  +
2 2 a 2 + b 2  2 2 a 2 + b 2 
 
Soluţie:
Condiţiile problemei ne sugerează substituţiile a = sin α + sin β ,
b = cos α + cos β , care verifică a + b ≤ 4 , deoarece cos(α − β ) ≤ 1 . Avem
2 2

succesiv:
4 − a2 − b2 1 − cos(α − β ) α −β b 4 − a 2 − b 2 sin α − sin β
= = tg ; = .
a +b
2 2
1 + cos(α − β ) 2 2 a2 + b2 2
a b 4 − a2 − b2 a b 4 − a2 − b2
+ = sin α ; − = sin β .
2 2 a2 + b2 2 2 a2 + b2
Egalitatea devine 1 − sin 2 α + 1 − sin 2 β = cos α + cos β = b .

15
R1.2.3. Să se arate că:
1+ x 1+ y 1+ z
+ + = 2 3, dacă şi numai dacă
1+ x + x 2
1+ y + y 2
1+ z + z2
x = y = z = 1,
( x, y , z ≥ 0 ) .
Soluţie:
Observând că x 2 + x + 1 este latura opusă unghiului de 120° în triunghiul cu
laturile 1 şi x , considerăm ∆ABC cu AC = 1 , AB = x , BC = x 2 + x + 1 ,
m( A) = 120°. Din teorema sinusurilor în ∆ABC avem:
AC = 2 R sin α , α = m(∠ABC ) ;
AB = 2 R sin (60° − α ) ;
BC = 2 R sin 120°;
Cum AB + AC > BC , avem:
x +1 2 R sin α + 2 R sin (60° − α ) 2[sin α + sin (60° − α )]
1< = = =
x2 + x +1 2 R sin 120° 3
4 sin 30° cos(30° − α ) 2 x +1 2
= ≤ . Deci ≤ ,cu egalitate dacă şi numai
3 3 x + x +1
2
3
1+ y 2 1+ z 2
dacă x = 1 . Analog ≤ , ≤ şi prin urmare
y + y +1
2
3 z + z +1
2
3
egalitatea are loc dacă şi numai dacă x = y = z = 1 .

Observaţie:
1+ x
Pentru demonstrarea inegalităţii ∑ ≤ 2 3 , am arătat mai
x2 + x +1
întâi trei inegalităţi mai simple, pe care apoi le-am adunat. Acest procedeu se
numeşte „spargerea” inegalităţii.

16
B. INEGALITĂŢI

R1.2.4. Să se arate că dacă a, b, c sunt numere reale pozitive oarecare, iar


x, y, z numere reale, avem:
( )( )(
a(1 + yz ) 1 + x 2 + b(1 + zx ) 1 + y 2 + c(1 + xy ) 1 + z 2 ≤ (a + b + c ) 1 + x 2 1 + y 2 1 + z 2 )
Soluţie:
 π π
Deoarece x, y, z ∈ R , (∃)α , β , χ ∈  − ,  astfel încât x = tgα , y = tgβ ,
 2 2
z = tgχ . Cu aceste notaţii, primul membru al inegalităţii se scrie succesiv:
( )
a cos(β − χ )
∑ a(1 + yz ) 1 + x 2 = ∑ a (1 + tgβ ⋅ tgχ ) 1 + tg 2α = ∑ cos α cos β cos χ
a+b+c
Membrul doi se scrie: (a + b + c ) (1 + x 2 )(1 + y 2 )(1 + z 2 ) = ; Cu
cos α cos β cos χ
acestea, inegalitatea de demonstrat este echivalentă cu:
a cos(β − χ ) + b cos(χ − α ) + c cos(α − β ) ≤ a + b + c ,
care este evidentă. Egalitate avem dacă şi numai dacă cos(β − χ ) = 1 ,
cos(χ − α ) = 1 , cos(α − β ) = 1 , deci α = β = χ , adică x = y = z .

Observaţie:
Inegalitatea se poate demonstra şi prin „spargerea” ei în trei inegalităţi
care se pot deduce folosind aceeaşi metodă trigonometrică. Acestea sunt:
( )( )( )
a(1 + yz ) 1 + x 2 ≤ a 1 + x 2 1 + y 2 1 + z 2 şi analoagele.

R1.2.5. Se consideră numerele reale x1 , x 2 ,K, x n ∈ [− 1,1] . Ştiind că suma


n
cuburilor acestor numere este 0, să se arate că x1 + x 2 + L + x n ≤ .
3
Soluţie:
Pornim de la formula cos 3α = 4 cos 3 α − 3 cos α . Deoarece xi ∈ [− 1,1],
(∃) α i ∈ [0, π ] , i = 1, n astfel încât xi = cos α i . Deducem succesiv:
x1 + x 2 + L + x n =
4 cos 3 α 1 − cos 3α 1 4 cos 3 α 2 − cos 3α 2
= cos α 1 + cos α 2 + L + cos α n = + +L+
3 3

17
4 cos 3 α n − cos 3α n 1 n
+ = − (cos 3α 1 + cos 3α 2 + L + cos 3α n ) ≤ .
3 3 3

R1.2.6. Dacă x ∈ R, x ≤ 1 , n ∈ N , atunci (1 − x ) + (1 + x ) ≤ 2 n


n n

Soluţie:
 π
Deoarece x ≤ 1 , putem nota x = cos 2t , t ∈ 0,  . Inegalitatea din enunţ
 2
devine
(1 − cos 2t )n + (1 + cos 2t )n ≤ 2 n ⇔ 2 n sin 2n t + 2 n cos 2 n t ≤ 2 n ⇔ sin 2 n t + cos 2 n t ≤ 1
inegalitate care se obţine prin adunarea inegalităţilor sin 2 n t ≤ sin 2 t şi
cos 2 n t ≤ cos 2 t . Egalitate avem în cazul n = 1 .

C. PROBLEME DE EXTREM

x2 + 2
R1.2.7. Să se afle minimul funcţiei f : R → R , f ( x ) =
x2 +1
Soluţie:
x∈R, deci (∃)α ∈  − π , π  , astfel încât x = tgα . Atunci
 2 2
x2 + 2 tg 2α + 2
= =
x2 +1 tg 2α + 1
sin 2 α + 2 cos 2 α 1 + cos 2 α 2 cos α
= ⋅ cos α = ⋅ cos α ≥ ⋅ cos α = 2 . Minimul
cos α
2
cos α
2
cos 2 α
este 2 şi se atinge pentru x = 0 .

R1.2.8. Să se determine minimul şi maximul expresiei


E ( x , y ) = x 1 − y 2 + y 1 − x 2 , x, y ∈ R .
Soluţie:
 π π
x, y ∈ [− 1,1] , notăm x = sin α , α ∈ − ,  , y = cos β , β ∈ [0, π ] .
 2 2
Atunci E ( x, y ) = sin α ⋅ sin β + cos β ⋅ cos α = ± cos(α ± β ) ≤ 1 , deci

x 1 − y 2 + y 1 − x 2 ≤ 1 ⇔ −1 ≤ E ( x, y ) ≤ 1 .

18
  2 2 
Minimul se atinge pentru (x, y ) ∈ (0,−1), (− 1,0 ),  − ,−  iar maximul

  2 2 
  2 2 
pentru (x, y ) ∈ (0,1), (1,0 ),  ,  .

  2 2 

R1.2.9. Să se determine mulţimea valorilor funcţiei


3 + 2 1− x2
f : [− 1,1] → R , f ( x ) = .
1+ x + 1− x

Soluţie: Din x ≤ 1 , y = f ( x ) > 0 , este suficient să studiem mulţimea


valorilor lui y 2 . Prin schimbarea de variabilă x = cos α , α ∈ [0, π ] ,
3 + 2 sin α 4t 2 + 12 t + 9
y= , de unde y 2 = , cu t = sin α . Din
α α 2( t + 1)
2 cos + 2 sin
2 2
1
α ∈ [0, π ] , t ∈ [0,1] ,rezultă y 2 = 2(t + 1) + + 2 . Considerăm funcţia
2(t + 1)
1
g : [1, ∞ ) → R , g ( x ) = x + . Arătăm că g este strict crescătoare pentru
x
 1 
x ∈ [1, ∞ ) . Într-adevăr, 0 ≤ x1 ≤ x2 ⇒ g ( x1 ) − g ( x2 ) = ( x1 − x 2 )1 −  < 0 .
 x1 x 2 
Pentru x = 2(t + 1) , cum t ∈ [0,1] ⇒ x ∈ [2,4] .
 5 17   9 25  3 2 5 
g ([2,4]) =  ,  . Deducem y 2 ∈  ,  , deci y ∈  ,  = Im f
2 4  2 4   2 2

D. ECUAŢII

R1.2.10. Să se rezolve ecuaţia 1 − x 2 = 4 x 3 − 3 x , x ∈ R .


Soluţie:
Condiţia de existenţă a ecuaţiei este x ≤ 1 . Se impune condiţia
4 x 3 − 3 x ≥ 0 . Deoarece x ≤ 1 , putem face schimbarea de variabilă x = cos α ,
α ∈ [0, π ] şi ecuaţia se scrie succesiv

19
1 − cos 2 α = 4 cos 3 α − 3 cos α ⇔ sin α = cos 3α . Pentru α ∈ [0, π ] , sin α ≥ 0
π 
şi sin α = cos 3α ⇔ cos 3α = cos − α 
deci cu soluţiile
2 
 3π  π  π
α ∈  + kπ  ∪  + kπ  , k ∈ Z . Cum α ∈ [0, π ] obţinem α1 = ,
4  8  8
5π 3π π 2+ 2 2− 2 2
α2 = , α3 = , deci x1 = cos = , x2 = − , x3 = − .
8 4 8 2 2 2

(
R1.2.11. Să se rezolve ecuaţia: x + 1 − x 2 = 2 2 x 2 − 1 )
Soluţie:
Condiţia de existenţă este 1 − x 2 ≥ 0 ⇔ x ∈ [− 1,1] . Facem schimbarea de
variabilă x = cos α , α ∈ [0, π ] care ne conduce la ecuaţia
cos α + sin α = 2 cos 2α . Pentru cos 2α ≥ 0 ridicăm la pătrat şi obţinem
1
succesiv 1 + sin 2α = 2 cos 2 2α ⇔ ⇔ 2 sin 2 2α + sin 2α − 1 = 0 ⇔ sin 2α =
2
1  k π kπ 
sau sin 2α = −1 . Din sin 2α = rezultă α ∈ (− 1) + / k ∈ Z  . Convine
2  12 2 
π π π π  2+ 6
α= deci x = cos = cos −  = = . Din sin 2α = −1 rezultă
12 12 3 4 4
 π kπ  3π 3π 2
α ∈ (− 1)k +1 + / k ∈ Z  . Convine α = de unde x = cos =− .
 4 2  4 4 2

E. SISTEME DE ECUAŢII

R1.2.12. Să se rezolve în mulţimea numerelor reale pozitive sistemul:


 x + y + z = xyz

 x + y
+
z
=
3 3
 2
 1+ x 1+ y2 1+ z2
2

(Mihail Bencze)

20
Soluţie:
 π
Din x, y, z > 0 , rezultă că (∃) α , β , χ ∈  0,  astfel încât x = tgα , y = tgβ ,
 2
tgα + tgβ + tgχ − tgα ⋅ tgβ ⋅ tgδ
z = tgχ . Deoarece tg (α + β + χ ) = =0
1 − tgα ⋅ tgβ − tgβ ⋅ tgχ − tgχ ⋅ tgα
 π
rezultă α + β + χ = kπ , k ∈ Z , dar α , β , χ ∈  0,  , deci α + β + χ = π .
 2
3 3
A doua ecuaţie se scrie: sin α + sin β + sin χ = (1)
2
α β −χ
Avem sin α + sin β + sin χ = sin α + 2 cos cos . Presupunând α
2 2
α
constant, cum cos ≥ 0 , vom avea maximul expresiei sin α + sin β + sin χ
2
pentru β = χ . Analog α = β , deci maximul se obţine pentru triunghiul
3 3
echilateral, prin urmare sin α + sin β + sin χ ≤ .
2
π π
Din (1) rezultă α = β = χ = , deci x = y = z = tg = 3.
3 3

R1.2.13. Fie a, b, c > 0 . Să se arate că sistemul


 x y z
= =

a 1+ x( 2
)
b 1+ y 2
( c 1+ z2 ) ( )
 xy + yz + zx = 1

admite soluţii în R 3 , dacă şi numai dacă a, b, c sunt lungimile laturilor unui
triunghi.
(V. Paterău)
Soluţie:
Ecuaţia a treia ne sugerează folosirea identităţii
α β β χ χ α
tg ⋅ tg + tg tg =1+ tg tg
2 2 2 2 2 2
valabilă într-un triunghi cu unghiurile α , β , χ deci α + β + χ = π . Observăm
că dacă (x 0 , y 0 , z 0 ) este soluţie, atunci şi (− x 0 ,− y 0 ,− z 0 ) este soluţie a
α β χ
sistemului. Considerăm x, y, z > 0 şi notăm x = tg , y = tg , z = tg .
2 2 2

21
x sin α y sin β z sin χ
Deducem succesiv: = ; = ; = şi
( a 1+ x) 2
2a ( )
b 1+ y 2
2b ( )
c 1+ z 2
2c
sin α sin β sin χ
din primele două egalităţi avem = = care exprimă teorema
a b c
sinusurilor într-un triunghi ABC , de laturi a, b, c şi unghiuri α , β , χ .
Soluţiile sistemului sunt deci
(x, y, z ) ∈  tg α , tg β , tg χ  − tg α ,−tg β ,−tg χ  .
 2 2 2  2 2 2 

22
2. Şiruri şi progresii

2.1. Progresii aritmetice

2.1.1. Definiţii echivalente


i. Un şir ( a n )n≥1 este o progresie aritmetică dacă orice termen, începând cu
al doilea, este egal cu precedentul la care se adaugă o constantă reală r, numită
raţia progresiei.
ii. Un şir ( a n )n≥1 este o progresie aritmetică dacă există un număr real r,
numit raţia progresiei astfel încât ak +1 = ak + r ( ∀ ) k ≥ 1 .
iii. Un şir ( a n )n≥1 este o progresie aritmetică dacă diferenţa oricăror doi
termeni consecutivi este constantă, adică a − ak = r ( ∀ ) k > 1 , constanta este
k +1
numită raţia progresiei.
Observaţii:
1. Dacă r = 0 , progresia este un şir constant an = a1 , ( ∀ ) n ≥ 1
2. Vom considera fie progresii aritmetice cu un număr infinit de
termeni, fie progresii aritmetice cu un număr finit de termeni
÷ a 1, a 2 , ..., a n .
2.1.2. Propoziţie Un şir ( a n )n≥1 este o progresie aritmetică dacă şi numai
dacă termenul general an este dat de relaţia an = a1 + (n − 1)r , ( ∀ ) n ≥ 1 , unde r
este raţia progresiei.
Demonstraţia acestei propoziţii este imediată.
2.1.3. Propoziţie Un şir ( a n )n ≥1 este o progresie aritmetică dacă şi
numai dacă termenul general an este dat de relaţia
an = ak + (n − k )r , ( ∀ ) n ∈ * .
Demonstraţie. Dacă are loc relaţia pentru (∀) n ∈ *
, atunci
an +1 = ak + (n + 1 − k )r , deci an +1 − an = r , ( ∀ ) n ∈ *
. Reciproc, din
ak +1 − ak = r , ak + 2 − ak +1 = r ,..., an − an −1 = r , rezultă an − ak = (n − k )r , deci
an = ak + (n − k )r.
Observaţii:
1. Pentru k=1 se obţine formula termenului general din 2.1.2.
a −a
2. ak poate fi orice termen al progresiei , deci r = n k
n−k

23
2.1.4. Propoziţie. Un şir ( a n )n≥1 este o progresie aritmetică dacă şi
a −a
numai dacă are loc relaţia n m = constant= r ( ∀ ) n, m ∈ * , n ≠ m.
n−m
Demonstraţie: Dacă şirul este progresie aritmetică, atunci din propoziţia
a −a
2.1.3 , avem an = am + (n − m)r , sau n m = r . Reciproc, dacă are loc relaţia
n−m
pentru orice n ≠ m , atunci prin substituţia n → k + 1 şi m → k , rezultă
ak +1 − ak = r , (∀)k ∈ * şi deci şirul este o progresie aritmetică.
2.1.5 Un şir ( a n )n≥1 este o progresie aritmetică dacă şi numai dacă
oricare ar fi trei termeni consecutivi, cel din mijloc este media aritmetică a
a +a
celorlalţi doi, adică ak = k −1 k +1 , (∀)k ∈ * , k ≥ 2 .Demonstraţia este
2
imediată.
2.1.6 Propoziţie. Un şir ( a n )n≥1 este o progresie aritmetică dacă şi
numai dacă are loc relaţia (n − 1)an +1 = nan − a1 , (∀)n ∈ *
.
Demonstraţie : Dacă ( a n )n≥1 este progresie aritmetică , atunci din
ak +1 = ak + r , avem că relaţia din enunţ este adevărată. Reciproc, dacă este
semnificată relaţia din enunţ (∀)n ≥ 1 , atunci scriind-o pentru n − 1 , avem
(n − 2)an = (n − 1)an −1 − a1 şi obţinem an −1 + an +1 = 2an , (∀)n ≥ 2 şi din propoziţia
precedentă rezultă că şirul este progresie aritmetică.
2.1.7 Propoziţie. Într-o progresie aritmetică finită, suma termenilor
egal depărtaţi de extreme este egală cu suma termenilor extremi, adică
ak + an +1− k = a1 + an , (∀)k ≤ n . Propoziţia rezultă imediat din precedentele.
2.1.8 Propoziţie. Un şir ( a n )n≥1 este o progresie aritmetică dacă şi
numai dacă suma S n = a1 + a2 + ... + an , a primilor n termeni este dată de relaţia
(a + a )n
S n = 1 n , (∀)n ∈ * .
2
Demonstraţie:
Dacă ( a n )n≥1 este progresie aritmetică, atunci scriind
S n = a1 + a2 + ... + an 
 ⇒ 2 S n = n(ak + an − k +1 ) = n(a1 + an ) , rezultă
S n = an + an −1 + ... + a1 
(a1 + an )n
Sn = .
2

24
Reciproc, dacă suma primilor n termeni este dată de relaţia
(a + a )n n +1 n
S n = 1 n , atunci an −1 = Sn −1 − Sn = (a1 + an +1 ) − (a1 + an ) , deci
2 2 2
(n − 1)an +1 = nan − a1 şi conform cu 2.1.6, rezultă că şirul este o progresie
aritmetică.
Observaţie:
(a + an − k +1 )n
1. Să se mai scrie S n = k
2
2. S n =
[ 2a1 + (n − 1)r ] n = n r + (2a − r ) n = α n2 + β n ,deci S este
2

1 n
2 2 2
funcţie de gradul al doilea în n.
2.1.9 Propoziţie Un şir ( a n )n≥1 este o progresie aritmetică dacă şi
numai dacă suma Sn , a primilor n termeni este dată de relaţia
S n = α n + β n, (∀)n ∈
2 *
,α , β ∈ *.
Demonstraţie : Dacă şirul este o progresie aritmetică, atunci din
(a + a )n r 2a − r r
propoziţia precedentă S n = 1 n = n 2 + 1 n = α n 2 + β n unde α =
2 2 2 2
r
şi β = a1 − . Reciproc, dacă Sn = α n2 + β n , atunci
2
an +1 = Sn +1 − Sn = α (n + 1) 2 + β (n + 1) − α n 2 − β n = 2α n + α + β şi deci
an +1 − an = (2α n + α + β ) − [2α (n − 1) + α + β ] = 2α = constant, de unde rezultă
că şirul este o proiecţie aritmetică cu raţia r = 2α , şi a1 = α + β .
2.1.10 Propoziţie Numerele reale a, b, c distincte, sunt termini (nu
neapărat consecutivi) ai unei progresii aritmetice dacă şi numai dacă
b−a
∈ *.
c −b
Demonstraţie: Numerele a, b, c fiind termini ai unei progresii, putem
b − a am − ak (m − k )r m − k
lua a = ak , b = am , c = an , atunci = = = ∈ *.
c − b an − am (n − m)r n − m
Reciproc putem presupune fără a restrânge generalitatea că a < b < c şi fie
b−a k c−a
= ∈ * , sau (m + k )b = ma + kc . Definim progresia a1 = a, r = şi
c −b m k+m
c − a ka + ma + kc − ka kc + ma (m + k )b
atunci ak +1 = a1 + k = = = =b şi
k +m k +m k+m m+k

25
c−a
ak + m +1 = a1 + (k + m)r = a + (k + m) = c . Deci a, b, c sunt termini ai unei
k+m
b−a k
progresii aritmetice, a1 , ak +1 şi ak + m +1 , unde = .
c −b m
b−a
Observaţie: Din propoziţia precedentă rezultă că dacă ∉ , atunci
c −b
numerele a, b, c nu pot fi neaparat termeni ai unei progresii aritmetice.

2.2. Progresii geometrice

2.2.1. Definiţii echivalente

i. Un şir (bn ) n≥1 este o progresie geometrică dacă orice termen începând cu
al doilea, se obţine din precedentul înmulţit cu o constantă reală nenulă numită
raţia progresiei.
ii. Un şir (bn ) n≥1 este o progresie geometrică dacă există q ∈ * , astfel
încât bk +1 = bk ⋅ q , (∀)k ∈ *

iii. Un şir (bn ) n≥1 este o progresie geometrică dacă câtul oricăror doi
b +1
termeni consecutivi este constant, adică k = q = constant.
bk
Relaţia de recurenţă între termenii consecutivi ai unei progresii
geometrice este o relaţie de recurenţă liniară de forma bn +1 = α bn + β cu β = 0
şi α = q ≠ 0 .
Observaţii :
1. Dacă q = 0 , progresia devine b1 , 0, 0,...
2. Dacă q = 1 , progresia este şirul constant bn = b1.
b
3. Dacă b1 > 0 , pentru k +1 = q > 1 se obţine o progresie strict
bk
crescătoare, iar dacă 0 < q < 1 se obţine o progresie strict descrescătoare,
b
deoarece k +1 = q < 1 .
bk
4. Vom considera fie progresii geometrice cu un număr finit de termeni
−& b1 , b2 ,..., bn , sau cu o infinitate de termeni (bn ) n≥1 .
&&&

26
2.2.2. Propoziţie Un şir (bn ) n≥1 este o progresie geometrică dacă şi
numai dacă termenul general bn este dat de relaţia bn = b1q n −1 , (∀)n ≥ 1 , unde
q∈ *
este raţia progresiei.
Demonstraţie: Dacă şirul este o progresie geometrică, atunci dând lui k
valori de la 1 la n în relaţia bk +1 = bk ⋅ q şi înmulţind termen cu termen relaţiile
obţinute, avem: b2b3 ...bn = b1b2 ...bn −1q n −1 , rezultă bn = b1q n −1 .Reciproc, dacă
b
termenul general al şirului este dat de relaţia bn = b1q n −1 , atunci k +1 = q , deci
bk
şirul este o progresie de raţie q .
2.2.3. Propoziţie Un şir (bn ) n≥1 este o progresie geometrică dacă şi
numai dacă termenul general este dat de relaţia bn = bk q n − k , (∀)n ∈ * .
Demonstraţia este imediată.
Observaţie: bk poate fi orice termen al şirului, deci
1
 bn  n − k
  = constant=q, (∀)n ∈
*

b
 k
2.2.4. Propoziţie Un şir (bn ) n≥1 este o progresie geometrică dacă şi
1
 b  n−m
numai dacă are loc relaţia  n  = constant=q, (∀)n ≠ m.
 bm 
Demonstraţie. Dacă (bn ) n≥1 este progresie geometrică, atunci din
1
 b  n−m
bn = bm ⋅ q n − m rezultă  n  = constant=q.
b
 m
1
 bn  n − m
Reciproc, dacă   = constant=q, (∀)n ≠ m , atunci
 bm 
bn +1
= q ⇒ (bn ) n ≥1 este o progresie geometrică.
bn
2.2.5. Propoziţie Un şir (bn ) n≥1 , b1 ≠ 0 este o progresie geometrică dacă
şi numai dacă oricare ar fi trei termeni consecutivi, cel din mijloc este media
geometrică a celorlalţi doi, adică bk2 = bk −1 ⋅ bk +1 , (∀)k ≥ 2 . Demonstraţia rezultă
imediat, la fel ca şi următoarea:

27
2.2.6. Propoziţie Într-o progresie geometrică cu un numar finit de
termeni, produsul terminilor egal despărtaţi de extreme este egal cu produsul
terminilor extremi, adică bk bn − k +1 = b1bn .
2.2.7. Propoziţie Un şir (bn ) n≥1 , neconstant este o progresie
geometrică dacă şi numai dacă suma S n = b1 + b2 + ...bn ,a primilor n termeni este
1 − qn
dată de S n = b1 , q ≠ 1, q ≠ 0 .
1− q
n n n
1 − qn
Demonstraţie. S n = ∑ bk = ∑ b1q k −1 = b1 ∑ q k −1 = b1 . Reciproc,
k =1 k =1 k =1 1− q
1− q n
dacă S n = b1 ,rezultă că şirul este o progresie geometrică cu raţia q .
1− q
2.2.8. Propoziţie Un şir (bn ) n≥1 neconstant, b1 ≠ 0 este o progresie
geometrică dacă şi numai dacă suma Sn a primilor n termeni este dată de relaţia
S n = α q n + β , (∀)n ∈ *
,α , β ∈ *
,q∈ *
− {1} .
Demonstraţie Dacă şirul este o progresie goemetrică, atunci din
b b
propoziţia precedentă avem Sn = 1 q n + 1 = α q n + β , unde
q −1 1− q
b
α = 1 , β = −α . Reciproc, dacă Sn = α q n + β , atunci
q −1
bn +1 = Sn +1 − S n = (α q n +1 + β ) − (α q n + β ) = α q n (q − 1) , deci bn = α q n −1 (q − 1) ,
rezultă bn +1 = bn q , deci este o progresie geometrică.
2.2.9. Propoziţie Numerele a, b, c pozitive şi distincte sunt termeni (nu
neapărat consecutivi) ai unei progresii geometrice dacă şi numai dacă
lg b − lg a
∈ *.
lg c − lg b
Demonstraţie Dacă a, b, c sunt termenii unei progresii geometrice,
atunci fie a = bk , b = bm , c = bn şi
lg b − lg a lg b1q m −1 − lg b1q k −1 lg q m − k m − k
= n −1 m −1
= n−m
= ∈ * . Reciproc, din
lg c − lg b lg b1q − lg b1q lg q n−m
lg b − lg c k
= ∈ * rezultă succesiv (m + k ) lg b = m lg a + k lg c ; sau
lg c − lg b m
1
 c  k +m
bm+ k = a m c k . Fie b1 = a şi q =   . Atunci
a

28
k
k m 1
 c  k +m
bk +1 = b1q = a  
k
= c k + m a k + m = ( c k a m ) k + m = b şi
a
k +m
k +m  c  m+ k
bk + m +1 = b1q = a  = c . Prin urmare a, b, c sunt termeni ai unei progresii
a
geometrice b1 , bk +1 şi bk + m +1 .

2.3. Şiruri recurente

2.3.1. Noţiuni fundamentale

Se spune că un şir ( xn ) n∈N de numere reale este definit de o relaţie de


recurenţă dacă se cunoaşte o ecuaţie F (n, xn , xn−1 ,..., x0 ) = 0 care leagă termenul
general xn de termenii anteriori. În continuare ne vom referi doar la recurenţe
de ordinul k definite explicit.
2.3.1. Definiţie. Spunem că şirul ( xn ) n∈N este definit printr-o relaţie
explicită de recurenţă de ordinul k, k ∈ N * dacă:
xn+ k = f (n, xn+ k −1 ,..., xn+1 , xn ), ∀ n ∈ N (2.3.1)
unde f : N × R → R este o funcţie.
k

Vom numi soluţie generală a relaţiei de recurenţă (2.3.1) mulţimea


şirurilor ( xn ) n∈N care verifică relaţia de recurenţă.
Vom numi soluţie particulară a relaţiei de recurenţă (2.3.1) şirul
( xn ) n∈N care verifică relaţia de recurenţă şi
x0 = p0 , x1 = p1 ,..., xk −1 = pk −1 , unde p0 ,..., pk −1 ∈ R fixaţi
2.3.2. Cazuri particulare
(i) Dacă k=1 vom vorbi despre relaţii de recurenţă de ordinul 1.
(ii) Dacă k=2 vom vorbi despre relaţii de recurenţă de ordinul 2.
(iii) Dacă f este liniară, adică:
f (n, xn+ k −1 ,..., xn ) = an0 + a1n xn+ k −1 + ... + ann xn ,
şirurile (ani ) n∈N fiind date, atunci vorbim despre relaţii de recurenţă liniare.
Dacă an0 = 0, ∀ n ∈ N atunci relaţia de recurenţă se numeşte liniară şi
omogenă.
Dacă an0 = 0, ∀ n ∈ N şi şirurile (a ni ) n∈N , i = 1, n sunt constante, atunci
relaţia de recurenţă se numeşte liniară, omogenă şi cu coeficienţi constanţi.

29
2.3.3. Exemple. 1) xn+ 2 = n 2 xn+1 + nxn este o relaţie de recurenţă liniară
şi omogenă de ordinul 2 cu coeficienţi variabili.
2) xn+ 2 = 2 xn+1 + 3xn + n este o relaţie de recurenţă liniară, neomogenă
de ordinul 2.
3) xn = xn−1 xn−2 este o relaţie de recurenţă neliniară de ordinul 2.
În cazul relaţiilor de recurenţă de ordinul k avem următoarea teoremă de
existenţă şi unicitate.
2.3.4. Teoremă. Fie f : N × R k → R o funcţie. Atunci există un singur
şir ( xn ) n∈N astfel încât:
xn+ k = f (n, xn+ k −1 ,..., xn ), ∀ n ∈ N
x0 = p0 , x1 = p1 ,..., xk −1 = pk −1 , unde p0 , p1 ,..., pk −1 ∈ R fixaţi.
Demonstraţie. Fie E0 = { p0 }, E1 = { p1},..., Ek −1 = { pk −1} şi
E n = {x ∈ R | x = f (n, xn−1 ,..., x0 ); x0 ∈ E0 , x1 ∈ E1 ,..., xn−1 ∈ En−1}, n ≥ k .
Vom demonstra prin inducţie matematică propoziţia:
P (n) : cardEn = 1
P (0), P(1),..., P (k − 1) sunt adevărate.
Presupunem că P (n), P(n + 1),..., P(n + k − 1) sunt adevărate şi anume
E n = {xn }, En+1 = {xn+1},..., En+ k −1 = {xn+ k −1} . Atunci şi E n+ k are un singur
element xn+ k = f (n, xn+ k −1 ,..., xn ) .
Deci P (n) ∧ P(n + 1) ∧ ... ∧ P(n + k − 1) ⇒ P(n + k ) este adevărată pentru
orice n ∈ N . Rezultă din inducţia completă că mulţimea E n+ k posedă un singur
element. Şirul ( xn ) n∈N satisface condiţiile din enunţ, deci teorema este
demonstrată.

2.3.2. Recurenţe liniare de ordinul 1

În cele ce urmează vom considera recurenţe liniare de ordinul 1.


2.3.5. Definiţie. O relaţie de recurenţă de forma
xn+1 = an xn + bn , ∀ n ∈ N , (2.3.2)
unde (an ) n≥0 şi (bn ) n≥0 sunt şiruri de numere reale se numeşte relaţie de
recurenţă liniară de ordinul 1 cu coeficienţi variabili.
2.3.6. Observaţii. i) Dacă an = a, bn = b, ∀ n ∈ R obţinem o relaţie de
recurenţă liniară de ordinul 1 cu coeficienţi constanţi.

30
ii) Dacă bn = 0, an = a, ∀ n ∈ N obţinem o progresie geometrică şi forma
generală a şirului xn este xn = a n x0 , ∀ n ∈ N .
iii) Dacă bn = b, an = 1, ∀ n ∈ N obţinem o progresie aritmetică şi forma
generală a şirului xn este xn = x0 + nb, ∀ n ∈ N .
iv) Dacă an = a şi bn = f (n) , unde f : N → R obţinem
xn+1 = axn + f (n) , adică o relaţie de recurenţă liniară, neomogenă de ordinul 1.
În cazul unei recurenţe liniare de ordinul 1 se poate determina forma
generală a şirului. Vom da mai întâi forma generală a unui şir recurent liniar de
ordinul 1 cu coeficienţi constanţi.
2.3.7 Teoremă. Forma generală a şirului ( xn ) n≥0 dat prin relaţia de
recurenţă
xn+1 = axn + b, ∀ n ∈ N (2.3.3)
este:
xn = a n x0 + b(a n−1 + a n−2 + ... + 1) (2. 4.)
Demonstraţie. Vom folosi procedeul iterării directe. Dăm pe rând valori
lui n în relaţia de recurenţă (2. 3.) şi avem:
x1 = ax0 + b | ⋅a n−1
x2 = ax1 + b | ⋅a n− 2
..................
xn−1 = axn− 2 + b | ⋅a
xn = axn−1 + b
Însumând obţinem:
xn = a n x0 + b(1 + a + ... + a n−1 )
2.3.8. Observaţii. Uneori în probleme se folosesc şi alte metode care
conduc la determinarea termenului general pentru o recurenţă liniară de ordinul
1 cu coeficienţi constanţi şi anume:
- Reducerea la o ecuaţie omogenă: căutăm h ∈ R astfel încât
xn+1 + h = a ( xn + h), ∀ n ∈ N .
b
Obţinem h = (dacă a ≠ 1 ) şi notând y n = xn + h obţinem
a −1
 b  b
y n+1 = ay n , de unde y n = a n y 0 , deci xn = a n  x0 + − .
 a −1  a −1
- Utilizarea ecuaţiei omogene satisfăcute de ∆xn = xn − xn−1 : scriem relaţia de
recurenţă pentru n − 1 şi n:
xn−1 = axn− 2 + b

31
xn = axn−1 + b
Scăzând avem
∆xn = a∆xn−1 ,
de unde ∆xn = a n−1∆x1 , deci xn − xn−1 = a n−1 ( x1 − x0 ) . Dând valori lui n şi
însumând se obţine (2.3.4)
b
- Notând xn = a n u n obţinem u n+1 = u n + n+1 (a ≠ 0) . Dând valori lui n şi
a
 1 1 
însumând se obţine u n = u 0 + b 2 + ... + n  , de unde
a a 
xn = a x0 + b(a n−1 + ... + 1) .
n

2.3.9. Exemple. Să se determine forma generală a şirurilor definite prin:


1
a) xn+1 = − xn + 1, n ≥ 0, x0 = 0
2
b) xn+1 = 2 xn − 3, n ≥ 0, x0 = 2
c) x n +1 = −2 x n + 1, n ≥ 0, x 0 = 1
n
 1  2 2
Soluţie. a) xn =  −   −  +
 2  3 3
b) xn = −2 + 3
n

2 1
c) xn = (−2) n ⋅ +
3 3

În continuare vom da forma termenului general pentru o relaţie de


recurenţă liniară cu coeficienţi variabili.
2.3.10. Teoremă. Forma generală a şirului ( xn ) n≥0 dat prin relaţia de
recurenţă
(2.3.1) este:
 n −1 
xn+1 = a0 a1 ...an x0 +  ∑ bk ak +1ak + 2 ...an  + bn (2.3.5)
 k =0 
Demonstraţie. Vom folosi procedeul iterării directe. Dăm pe rând valori
lui n în relaţia de recurenţă (2. 1) şi avem:
x1 = a0 x0 + b0 | ⋅an an −1 ...a1
x1 = a1 x1 + b1 | ⋅an an−1 ...a2
x3 = a2 x2 + b2 | ⋅an an−1 ...a3

32
....................
xn = an −1 xn−1 + bn−1 | ⋅an
xn+1 = an xn + bn
Însumând obţinem
xn+1 = a0 a1 ...an x0 + b0 a1a2 ...an + b1a2 ...an + ... + bn−1an + bn
2.3.11. Corolar. Forma generală a şirului ( xn ) n≥0 dat prin relaţia de
recurenţă
xn+1 = axn + f (n), ∀ n ≥ 0 (2.3.6)
unde a ∈ R, f : N → R este
n −1
xn = a n x0 + ∑ f (k )a n− k −1 . (2.3.7)
k =0
2.3.12. Observaţie. Soluţia generală a relaţiei de recurenţă (2.3.6) este
suma dintre soluţia generală a relaţiei omogene şi o soluţie particulară a relaţiei
neomogene, adică: xn = y n + z n , unde
 y n+1 = ay n (solutie generala)

 z n+1 = az n + f (n), z 0 = p0 fixat (solutie particulara)
Dacă f (n) = α n P(n) unde α ∈ R şi P un polinom nenul, o soluţie
particulară are forma z n = α n Q(n) . Înlocuind în relaţia de recurenţă (2.3.6)
obţinem:
αQ(n + 1) − aQ(n) = P(n)
- dacă a = α atunci gradQ(n) = gradP(n) + 1
- dacă a ≠ α atunci gradQ(n) = gradP(n) .
Coeficienţii polinomului Q(n) se determină folosind metoda
coeficienţilor nedeterminaţi.
2.3.13. Exemple. Să se determine forma generală a şirului ( xn ) dat prin:
n 1
a) xn+1 = xn + , n ≥ 1, x1 = 0
n +1 n
n
2 1
b) xn+1 = xn +   n, n ≥ 0, x0 = 1
3 2
n
1 1
c) xn+1 = xn +   , n ≥ 0, x0 = 1 .
2 2
Soluţie. a) Aplicând teorema 2. 10. (sau folosind procedeul iterării
directe) obţinem:
xn+1 = a1 ...an x1 + b1a2 ...an + b2 a3 ...an + ... + bn−1an + bn

33
n 1
unde an = , bn = , n ≥ 1 deci
n +1 n
 3
2 n  13 n  1 n 1
xn+1 = 1 ⋅  ⋅ ⋅ ... ⋅  +  ⋅ ... ⋅  + ... + ⋅ +
3 4 n +1 2  4 n +1 n −1 n +1 n
1   1 1 
xn+1 =
 n + 1 + + ... + 
n +1   2 n 
b) O vom rezolva aplicând observaţia 2. 12. Avem xn = y n + z n , unde
n
2 2
y n+1 = y n , deci y n = c  , c ∈ R este o constantă ce se va determina
3 3
ulterior.
n
2 1
zn este o soluţie particulară a relaţiei deci z n+1 = z n +   n . Căutăm aşadar
3 2
n
1 1 2
z n =   Q(n) . Rezultă Q(n + 1) − Q(n) = n , Q(n) = αn + β . Înlocuind şi
2 2 3
n
1
egalând coeficienţii obţinem α = −6, β = 9 , deci z n =   (−6n + 9) .
2
n n
2 1
Deci xn = c  +   (−6n + 9) , x0 = 1 . Dând lui n valoarea 0
3 2
n n
27 27  2   1 
obţinem c = − , deci xn = −   +   (−6n + 9) .
4 4 3 2
c) Vom folosi corolarul 2.3.11 şi obţinem
n k n − k −1 n n −1 n −1
 1  n−1  1   1  1 1 1 1 
xn =   + ∑     =   + n  =   + n
 2  k =0  2   2  2 2 2 2 

2.3.3. Recurenţe liniare omogene de ordinul 2 cu coeficienţi constanţi

2.3.14. Definiţie. O relaţie de recurenţă de forma


xn+ 2 = axn+1 + bxn , ∀ n ∈ N, a, b ∈ R, b ≠ 0 (2. 8)
se numeşte relaţie de recurenţă liniară, omogenă, cu coeficienţi constanţi, de
ordinul 2.
Pentru a determina forma generală a şirului ( xn ) care verifică relaţia de
recurenţă (2. 8) vom folosi următoarele leme:

34
2.3.15. Lemă. Dacă şirurile (α n ) n∈N şi (β n ) n∈N satisfac condiţia (2. 8)
atunci şirul cu termenul general c1α n + c2β n satisface aceeaşi condiţie.
Demonstraţie. Deoarece α n+ 2 = aα n +1 + bα n şi β n+ 2 = aβ n+1 + bβ n ,
rezultă c1α n+ 2 + c2β n+ 2 = a(c1α n+1 + c2β n+1 ) + b(c1α n + c2β n ) .
2.3.16. Lemă. Dacă α este o rădăcină a ecuaţiei r 2 = ar + b , atunci şirul
(α ) n∈N verifică relaţia de recurenţă.
n

Demonstraţie. Deoarece α 2 = aα + b , înmulţind egalitatea cu α n


obţinem α n+ 2 = aα n +1 + bα n .
2.3.17. Definiţie. Ecuaţia
r 2 = ar + b (2.3.9)
se numeşte ecuaţia caracteristică ataşată relaţiei de recurenţă (2.3.8).
2.3.18. Teoremă. Dacă ecuaţia caracteristică r 2 = ar + b are două
rădăcini reale şi distincte r1 şi r2, atunci şirul care satisface egalitatea (2.3.8) are
termenul general de forma:
xn = c1r1n + c2 r2n , ∀ n ∈ N (2.3.10)
unde c1 şi c2 se determină în mod unic din condiţiile iniţiale x0 şi x1.
Demonstraţie. Din lema 2. 16. rezultă că r1n şi r2n verifică relaţia (2.3.8)
şi din lema 2.3.15. rezultă că xn = c1r1n + c2 r2n .
Pentru a determina c1 şi c2 avem de rezolvat sistemul
c1 + c2 = x0

c1r1 + c2 r2 = x1
care are soluţie unică întrucât r1 ≠ r2 .
2.3.19. Exemple. a) (şirul lui Fibonacci). Să se determine şirul ( Fn ) n∈N
definit prin:
Fn+ 2 = Fn+1 + Fn , ∀ n ∈ N
F0 = F1 = 1
b) (şirul lui Lucas). Să se determine şirul ( Ln ) n∈N definit prin:
Ln+ 2 = Ln+1 + Ln , ∀ n ∈ N
L0 = 2, L1 = 1 .
Soluţie. a) Ecuaţia caracteristică ataşată relaţiei de recurenţă este
1± 5
r 2 − r − 1 = 0 . r1, 2 = . Deci şirul are termenul general de forma
2
n n
1+ 5  1− 5 
Fn = c1   + c2 
  2  .

 2   
Din condiţiile iniţiale obţinem:
35
c1 + c2 = 1

 1+ 5 1− 5
c1 2 + c2 2 = 1
5 +1 5 −1
de unde c1 = şi c2 = .
2 5 2 5
1  1 + 5  1− 5  
n +1 n +1

Deci Fn =   −  
5  2   2  .
   
b) Procedând ca la a) constantele c1 şi c2 verifică:
c1 + c2 = 0

 1+ 5 1− 5
c1 2 + c2 2 = 1
n n
1+ 5  1− 5 
de unde c1 = c2 = 1 , deci Ln =   +
  2  .

 2   
2.3.20. Lemă. Dacă ecuaţia caracteristică r 2 = ar + b admite o rădăcină
dublă α, atunci şirul cu termenul general nα n satisface condiţia (2.3.8).
Demonstraţie. Înlocuind xn = nα n în relaţia de recurenţă (2.3.8) avem:
(n + 2)α n+ 2 = a(n + 1)α n+1 + bnα n ⇔
(n + 2)α 2 = a(n + 1)α + bn ⇔
n(α 2 − aα − b) + α(2α − a) = 0
a
care este adevărată întrucât α 2 − aα − b = 0 şi α = (întrucât ∆=0).
2
2.3.21. Teoremă. Dacă ecuaţia caracteristică r 2 = ar + b are o rădăcină
dublă α, atunci şirul care satisface egalitatea (2.3.8) are termenul general de forma:
xn = c1α n + c2 nα n , ∀ n ∈ N (2.3.11)
unde c1 şi c2 se determină în mod unic din condiţiile iniţiale x0 şi x1.
Demonstraţie. Din lemele 2.3.15., 2.3.16 şi 2.3.20 rezultă că şirul care
satisface egalitatea (2.3.8) are forma xn = c1α n + c2 nα n .
Pentru a determina c1 şi c2 avem de rezolvat sistemul
c1 = x0

c1α + c2 α = x1

36
c1 = x0

care are soluţia unică  x1 − x0 α întrucât α≠0 (pentru că b≠0).
c 2 =
α
2.3.22. Exemple. Să se determine termenul general al şirului ( xn ) n∈N
definit prin:
xn+ 2 = 4 xn+1 − 4 xn , ∀ n ∈ N
x0 = 1, x1 = 1 .
Soluţie. Ecuaţia caracteristică asociată relaţiei de recurenţă este:
r − 4r + 4 = 0 care are rădăcina dublă r1 = r2 = 2 . Rezultă că
2

xn = c1 ⋅ 2 n + c2 ⋅ n ⋅ 2 n . Din condiţiile iniţiale obţinem:


c1 = 1

2c1 + 2c2 = 1
1
deci c1 = 1 , c2 = − şi xn = 2 n−1 (2 − n) .
2
2.3.23. Teoremă. Dacă ecuaţia caracteristică r 2 = ar + b cu ∆<0 are
rădăcinile r1, 2 = r (cos t ± i sin t ) , atunci şirul care satisface condiţia (2.3.8) are
termenul general de forma:
xn = r n (c1 cos nt + c2 sin nt ), ∀ n ∈ N (2.3.12)
unde c1 şi c2 se determină în mod unic din condiţiile iniţiale x0 şi x1.
Demonstraţie. Conform lemei 2.3.16., şirul r1n satisface condiţia (2.3.8).
Înlocuind în (2.3.8) şi ţinând cont de formula lui Moivre:
r1n = r n (cos nt + i sin nt )
avem:
r n+ 2 [cos(n + 2)t + i sin(n + 2)t ] = ar n+1[cos(n + 1)t + i sin( n + 1)t ] +
+ br n (cos nt + i sin nt )
Separând părţile reale şi cele imaginare obţinem:
r n+ 2 cos(n + 2)t = ar n+1 cos(n + 1)t + br n cos nt
 n+2 n +1
r sin( n + 2)t = ar sin( n + 1)t + br sin nt
n

Înmulţind aceste egalităţi cu c1 şi c2 şi adunându-le deducem că şirul


xn = r (c1 cos nt + c2 sin nt ) satisface relaţia de recurenţă (2.3.8). Din condiţiile
n

iniţiale avem
c1 = x0

r (c1 cos t + c2 sin t ) = x1
de unde, întrucât r sin t ≠ 0 avem:

37
c1 = x0

c = x1 − rx0 cos t
 2 r sin t
2.3.24. Exemple. Să se determine forma generală a şirului ( xn ) n∈N
definit prin:
xn+ 2 = 2 2 xn+1 − 4 xn , ∀ n ∈ N, x0 = 1, x1 = 1
Soluţie. Ecuaţia caracteristică asociată r 2 − 2 2r + 4 = 0 are două
 π π
rădăcini complexe r1, 2 = 2 ± i 2 = 2 cos ± i sin  .
 4 4
 nπ nπ 
Atunci xn = 2 n  c1 cos + c2 sin  şi din condiţiile iniţiale obţinem
 4 4 
c1 = 1

2 c cos π + c sin π  = 1
  1 4
2
4

2 −2  nπ 2 −2 nπ 
de unde c1 = 1, c2 = . Deci xn = 2 n  cos + sin  .
2  4 2 4 
2.3.25. Observaţie. Mulţimea
S a ,b = {( xn ) n≥0 | xn + 2 = axn+1 + bxn , ∀ n ≥ 0} , a, b ∈ R
formează un subspaţiu vectorial de dimensiune 2 a spaţiului vectorial al şirurilor
reale. Dacă ecuaţia caracteristică r 2 = ar + b are:
- două rădăcini reale şi distincte r1 , r2 atunci mulţimea soluţiilor de bază
n n
{r1 , r2 } este bază a spaţiului vectorial
- două rădăcini reale egale r1 = r2 atunci mulţimea {r1n , nr1n } este bază a
spaţiului vectorial
- două rădăcini complexe r1, 2 = r (cos t ± i sin t ) atunci mulţimea
{r n cos nt , r n sin nt} este bază a spaţiului vectorial.

Bibliografie

• Gh. Andrei, Şiruri şi progresii, Ed. Paralela 45, pag 18-62


• Gh. Andrei, C. Caragea, I. Cucurezeanu, Probleme de algebră pentru
concursuri de admitere şi olimpiade şcolare, E.D.P., Bucureşti, 1993

38
Probleme rezolvate ( 2)

R2.3.1. Să se arate că elementele mulţimii M = {a, b, c} pot forma o progresie


aritmetică, dacă şi numai dacă ( b + c − 2a ) + ( c + a − 2b ) + ( a + b − 2c ) = 0
3 3 3

(Gh. Andrei)
Soluţie : Utilizăm identitatea
x + y + z − 3xyz = ( x + z + y ) ( x 2 + y 2 + z 2 − xy − yz − zx ) .
3 3 3
Luând
x = b + c − 2a , y = c + a − 2b, z = a + b − 2c , avem x + y + z = 0 , deci
x + y + z = 3 xyz , prin urmare ( b + c − 2a )( a + c − 2b )( a + b − 2c ) = 0 , ceea ce
3 3 3

înseamnă că elementele mulţimii M = {a, b, c} pot forma o progresie


aritmetică.
R2.3.2. Se consideră propoziţiile (α ) , ( β ) , ( γ )
(α ) x,y,z distincte, în progresie aritmetică
(β ) a, b, c distincte, în progresie geometrică, a, b, c ∈ ( 0, ∞ ) − {1}
(γ ) a yb z c x = a z c xc y .
Să se demonstreze că oricare două propoziţii inplică pe a treia.
(Gh. Andrei)
Soluţie :
i. Arătăm că (α ) şi ( β ) inplică ( γ ) . Putem presupune x > y > z şi
x − y = y − z = r > 0 , x − z = 2r > 0 , iar b 2 = ac . Rezultă lg a + lg c = 2 lg b , de
unde ( y − z ) lg a + ( x − y ) lg c = ( x − z ) lg b . Obţinem succesiv
y lg a + x lg c + z lg b = z lg a + x lg b + y lg c , sau lg ( a y b z c x ) = lg ( a z b x c y ) care
este echivalentă cu ( γ ) .
ii. Demonstrăm că (α ) şi (γ ) implică (β ). Din (γ ) rezultă
( y − z ) lg a + ( x − y ) lg c = ( x − z ) lg b , de unde r lg a + r lg c = 2r lg b deci
ac = b 2 .
iii. ( β ) şi ( γ ) implică (α ) . Din ( γ ) rezultă a 2 y b 2 z c 2 x = a 2 z b 2 x c 2 y sau
a 2 y c 2 x ( ac ) = a 2 z c 2 y ( ac ) , de unde a 2 y + z c 2 x + z = a 2 z + x c 2 y + x sau a 2 y − x − z = c 2 y − x − z
z x

care este echivalentă cu 2 y = x + z , adică ÷ x, y, z .


R2.3.3. Numerele x1 , x2 ,..., xn 2 formează o progresie aritmetică. Se ştie
că x1 + x2 + ... + xn = a, x12 + x22 + ... + xn2 = b 2 . Determinaţi acestă progresie.

39
Soluţie
Fie r raţia progresiei. Avem xk = x1 + ( k − 1) r şi din prima egalitate

rezultă
( x1 + xn ) n = a , sau 1
nx1 + rn ( n − 1) = a (1)
2 2
Pe de altă parte xk2 = x12 + 2 x1r ( k − 1) + r 2 p ( k − 1) şi din a doua relaţie din
2

n n n

∑x = nx12 + 2 x1r ∑ ( k + 1) + r 2 ∑ ( k − 1) = b 2 şi deci


2 2
enunţ, avem k
k =1 k =1 k =1

1
nx12 + x1rn ( n − 1) + r 2 ( n − 1) n ( 2n − 1) = b 2 (2)
6
Din (1), prin ridicare la pătrat şi împărţire cu n,
1 2 a2
avem nx1 + x1rn ( n − 1) + r ( n − 1) n ( 2n − 1) =
2
(3)
4 n
r 2 n ( n − 1)
2
b2n − a 2 2 3 (b2n − a 2 )
Din (2) şi (3) avem = , de unde r = ± .
12 n n n2 − 1
Din (1) rezultă x1 şi progresia este determinată.
1 1 1 n
R2.3.4 Determinţi inegalitatea + + ... + < , unde
a1a2 a3 a4 a2 n −1a2 n a0 a2 n
a0 , a1 ,..., an sunt termenii unei progresii aritmetice crescătoare cu termeni
pozitivi.
(Cardinal nr. 2/1998 )
1 1 1
Soluţie : Considerăm sumele S1 = + + ... + şi
a1a2 a3 a4 a2 n −1a2 n
1 1 1
S2 = + + ... + . Fie r raţia progresiei. Ţinând seama că
a0 a1 a2 a3 a2 n − 2 a2 n −1
a1 − a0 = a2 − a1 = a3 − a2 = ... = a2 n − a2 n −1 = r şi că
r a −a 1 1
= k +1 k = − , suma S1 + S 2 se scrie după reducerea termenilor,
ak ak +1 ak ak +1 ak ak +1
1 1 1  2n
S1 + S 2 =  − = . Cum progresia are termeni pozitivi şi este
r  a0 a2 n  a0 a2 n
1 1 1 1 1 1
crescătoare, avem < , < ,..., < şi rezultă
a1a2 a0 a1 a3 a4 a2 a3 a2 n −1a2 n a2 n − 2 a2 n −1
2n n
S1 < S 2 , deci 2 S1 < S1 + S 2 = , prin urmare S1 < .
a0 a2 n a0 a2n

40
R2.3.5. Fie a, b, c trei numere naturale cu ( a, b, c ) = 1 .Dacă a, b,
c sunt termeni, nu neapărat consecutivi , ai unei progresii aritmetice, atunci
a , b , c sunt numere naturale.
(Ion Cucurezeanu)
Soluţie : Dacă r este raţia progresiei, atunci c − a = nr ,
b − a = mr , cu m, n ∈ * . Eliminând r între cele două relaţii, obţinem
m c − n b = (m − n) a (1) , de unde prin ridicare la pătrat, avem
m 2 c + n 2b − 2mn bc = ( m − n ) a
2
(2) . Cum menbrul întâi din (2) trebuie să
fie număr natural, rezultă b = B 2 d , c = C 2 d , unde d = ( b, c ) , B, C ∈ *
Din (1) rezultă ( mC − nB ) d = ( m − n ) a ( 3)
Din (3), după eventualele simplificări se scrie A d = D a , cu ( A, D ) = 1
sau A2 d = D 2 a .Din ( a , b, c ) = 1 , rezultă ( a, d ) = 1 şi cum ( A, D ) = 1 , din
A2 d = D 2 a , obţinem a = A2 , d = D 2 , deoarece din a A2 , A2 a şi a, A, numere
naturale, avem a = A2 .Din a = A2 , b = B 2 D 2 , c = C 2 D 2 , rezultă
a , b , c ∈ *.
R2.3.6. Dacă şirul numerelor naturale 1, 2,3,... se împarte în câteva
progresii aritmetice, atunci în una (cel puţin) din aceste progresii, primul termen
se divide cu raţia.
(A.V. Kelarev, Kvant 1/1985)
Soluţie : Notăm prin a1 , a2 ,..., an primii termeni ai progresiilor în care se
împarte şirul numerelor naturale şi prin d1 , d 2 ,..., d n raţiile lor. Produsul acestor
raţii se află într-una din progresii, deci există i, cu 1 ≤ i ≤ n şi un anume k, astfel
încât d1 ⋅ d 2 ⋅ ... ⋅ d n = ai + kdi . Din această egalitate rezultă că di divide pe ai .
R2.3.7. Într-o progresie geometrică avem S p = a, S2 p = b . Să se
determine Skp în funcţie de a, b, k ( a, b ∈ , a ≠ 0, b ≠ 2a, k , p ∈ * )
(Ovidiu Pop, RMT. Nr1/1996)
q −1 p
q2 p −1
Soluţie : Avem S p = α = a şi S p = = b , unde α este
q −1 q −1
b
primul termen. Prin înpărţirea celor două relaţii, deducem q p + 1 = , deci
a

41
b
qp = − 1 . Apoi,
a
a  b   a 2  b  
k k
q kp − 1 a
S kp = α = p ( q − 1) =
kp
 − 1 − 1 =  − 1 − 1 .
q −1 q −1 b  b − 2a  a 
− 2  a  
a
R2.3.8. Într-o progresie geometrică cu termeni pozitivi, produsul
primilor m termeni este egal cu produsul primilor n termeni ( m ≠ n ) . Să se
demonstreze că produsul primilor m+n termeni este egal cu 1.
Soluţie : Fie progresia (bn ) n≥1 şi m > n . Din b1 ⋅ b2 ⋅ ... ⋅ bm = b1 ⋅ b2 ⋅ ... ⋅ bn ,
rezultă bn +1 ⋅ ... ⋅ bm = 1 , deci
( bn+1 ⋅ bn+ 2 ⋅ ... ⋅ bm ) = ( bn+1bm )( bn+ 2bm−1 ) ... ( bmbn+1 ) = ( b1bm+ n ) = 1 , de unde
2 m+ n

b1bm + n = 1 . Dar ( b1b2 ...bm + n ) = ( b1bm + n )( b2bm + n −1 ) ... ( bm + nb1 ) = ( b1bm + n ) = 1 , prin
2 m+ n

urmare b1b2 ...bm + n = 1 .


R2.3.9. Fie ( xn ) n ≥1 un şir de numere reale strict pozitive şi distincte. Să
se arate că ( xn ) n ≥1 este o progresie aritmetică dacă şi numai dacă
n −1
1 x (x − x )
( x22 − x12 )∑ = 2 n 1 , ∀n ≥ 2
k =1 xk + xk +1 xn
(N. Papacu)

Soluţie
Presupunem că este verificată relaţia din enunţ
n −1
1 x x −x x −x

k =1 xk + xk +1
= 2 2 2 ⋅ n 1 = A n 1 (1)
x2 − x1 xn xn
n
1 x −x
Pentru n → n + 1 , avem ∑x
k =1 + xk +1
= A n +1 1 (2)
xn +1
k

1 x −x
Scăzând (1) din (2), obţinem = Ax1 n +1 n (3)
xn + xn +1 xn xn +1
1 x −x
Avem şi = Ax1 n n −1 (4). Împărţind membru cu membru
xn −1 + xn xn −1 xn
xn −1 + xn xn +1 − xn xn −1 x2 − x2 x
relaţiile (3) şi (4), avem = ⋅ , de unde n2 n −21 = n −1 ,
xn + xn +1 xn − xn −1 xn +1 xn +1 − xn xn +1
ceea ce este echivalent cu ( xn2 − xn +1 xn −1 )( xn +1 + xn −1 ) = 0. Rezultă

42
xn2 = xn +1 xn −1∀n ∈ ∗ , deci şirul formează o progresie geometrică. Reciproc,
dacă şirul formează o progresie geometrică, atunci xk +1 = xk q şi prin calcul
x1 (q n −1 − 1)
direct ambii membri sunt egali cu .
q n−2

R2.3.10. Să se arate că orice progresie aritmetică formată din numere


naturale conţine termenii unei progresii geometrice.

Soluţie
Fie (an ) n ≥1 o progresie aritmetică formată din numere naturale şi r ∈ ∗
raţia progresiei. Progresia se scrie a1 , a1 + r , a1 + 2r ,..., a1 + kr , a1 + a1r = a1 (1 + r ) .
Printre aceşti termeni se găsesc şi termenii a1 + ra1 (2 + r ) = a1 (1 + r ) 2 = aa1 (2 + r ) +1
şi
a1 + a1 (3 + 3r + 3r 2 )r = a1 (1 + r )3 = aa (3+3r + r 2 ) +1 . Deducem că orice număr
1

de forma a1 (1 + r ) este termen al progresiei aritmetice şi anume aa (1+ r )k − a +1 .


k
1 1

Aşadar şirul a1 , a1 (1 + r ), a1 (1 + r ) 2 ,... este o progresie geometrică.

R2.3.11. Fie şirul ( xn ) n ≥1 , cu x1 = 1 , x2 = 2 , definit prin xn5+1 = xn3+ 2 ⋅ xn2 . Să


se determine termenul general xn .

Soluţie
Logaritmăm relaţia în baza 2 şi obţinem:
5log 2 xn +1 = 3log 2 xn + 2 + 2 log 2 xn , log 2 x1 = 0 log 2 2 = 1 . Cu notaţia
yn = log 2 xn , avem 5 yn +1 = 3 yn + 2 + 2 yn , cu y1 = 0 , y2 = 1 , sau
2
3( yn + 2 − yn +1 ) = 2( yn + 2 − yn ) . Cu substituţia zn +1 = zn , deci zn este o progresie
3
n −1
2 2
cu raţia q = , deci zn = z , q n −1 =   . Din relaţia zk = yk −1 − yk , rezultă
3 3
n
1 − qn   2 n 
∑ zk = yn+1 − y1 , deci yn+1 = z1 1− q
= 3 1 −    . Din log 2 xn = yn , rezultă
k =1   3  
n−1
2
1− 
x n = 2 yn = 8 3
.

43
3. Numere complexe în algebră

3.1. Proprietăţi generale

Din teoria numerelor complexe presupunem cunoscute noţiunile de bază,


studiate în manualele şcolare. Fără a intra în detalii, trecem în revistă câteva
dintre acestea.
Fie C = { z = x + iy x, y ∈ R } corpul numerelor complexe înzestrat cu
binecunoscutele operaţii de adunare şi înmulţire. Dacă z = x+iy ∈ C, x se
numeşte partea reală a numărului complex z şi se notează cu Re z , iar y
coeficientul părţii imaginare şi se notează Im z .

X.3.1.1. Modulul numărului complex z = x + iy reprezintă numărul real, notat


z = x2 + y2 .
Proprietăţi ale modulului:
1) z ≥ 0 , ∀ z ∈ C
z =0 ⇔ z =0
2) z = − z , ∀ z∈ C
3) - z ≤ Re z ≤ z şi − z ≤ Im z ≤ z , ∀ z ∈ C
4) z1 ⋅ z 2 = z1 ⋅ z 2 , ∀ z1 , z 2 ∈ C
z1 z1
5) = , ∀ z1 , z 2 ∈ C , z 2 ≠ 0
z2 z2
6) z1 + z 2 ≤ z1 + z 2 , ∀ z1 , z 2 ∈ C , cu egalitate dacă şi
numai dacă ∃ t ≥ 0: z2 = t z1.
7) z 1 − z 2 ≤ z 1 − z 2 ≤ z 1 + z 2 , ∀ z 1 , z 2 ∈ C
3.1.2. Conjugatul numărului complex z = x+iy este numărul complex :
z = x − iy.
3.1.3. Proprietăţi :
1) z = z ⇔ z ∈ R ;
2) z = z , ∀ z ∈ C ;
2
3) z = z ⋅ z, ∀ z ∈C ;
4) z 1 + z 2 = z 1 + z 2 , ∀ z 1 , z 2 ∈ C ;
5) z 1 ⋅ z 2 = z 1 ⋅ z 2 , ∀ z 1 , z 2 ∈ C ;

44
 z  z
6)  1  = 1 , ∀ z 1 , z 2 ∈ C , z 2 ≠ 0 ;
 z2  z2
z+z z−z
7) Re z = ; Im z = , ∀ z ∈C .
2 2i

Bibliografie

1. Alexandru P., Brânzei D., Gorgotă V., Ulmeanu S., Mateamtica în


concursuriel şcolare, Editura Paralela 45, Piteşti, 1999
2. Andrica D., Bişboacă N., Numere complexe. Probleme rezolvate din
manualele alternative, Editura Millenium, Alba Iulia, 2000
3. Andrica D., Bişboacă N., Numere Complexe de la…a…la…z, Editura
Millenium, Alba Iulia, 2001
4. Andrei Gh., Caragea C., Cucurezeanu I., Bordea Gh., Probleme de algebră
pentru concursuri de admitere şi olimpiade şcolare, E.D.P., Bucureşti, 1993
5. Becheanu M. şi colaboratori, Olimpiade de matematică 1990-1996, clasele
IX-X, Editura Gil, Zalău, 1997
6. Nicula V., Numere complexe, Probleme şi exerciţii pentru clasa a X-a,
Editura Scorpion 7, Bucureşti, 1993
7. Tămâian T., Probleme selectate din reviste şcolare, Editura Cub Press,
Baia Mare, 2002

45
Probleme rezolvate (3)

R3.2.1 Fie x,y,z trei numere complexe cu proprietăţile: x = y = z şi


xyz = xy + yz + zx = 1. Calculaţi x + y + z.
Soluţie :
3
Avem 1 = xyz = x ⋅ y ⋅ z = x . Deci x = y = z = 1.
1 1 1
Rezultă x ≠ 0 şi x =
. Analog y = , z = . Avem:
x y z
xy + yz + zx 1 1 1
1= = + + = z + x + y , deci x + y + z = 1, de unde x+y+z =
xyz z x y
1.

1
R3.2.2 Fie a ∈ R, z ∈ C, astfel încât a = z + . Să se determine cea mai
z
mare şi cea mai mică valoare a lui z .
Soluţie :
Deoarece a ≥ 0, avem:

a = z+
2 1
2
 1  1 2
=  z +  z +  = z +
z2 + z 1
2

+ 2 =
4
(
z + z + z − 2 z +1 )2 2

2 2
z  z  z z z z

4
⇒ z − z 2 (a 2 + 2) + 1 = − ( z + z ) ≤ 0
 a 2 + 2 − a 4 + 4a 2 a 2 + 2 + a 4 + 4a 2 
⇒ z ∈ , .
 2 2 
Aplicând formula radicalilor compuşi rezultă că :
− a + a2 + 4 a + a2 + 4  a + a2 + 4
z ∈ ,  . Deci max z = , min
 2 2  2
− a + a2 + 4 a + a2 + 4
z = şi se ating pentru z = ± i.
2 2

R3.2.3 Fie z1 , z 2 , z 3 ∈ C * , distincte două câte două şi cu modulele egale.


Ştiind
1 1 1
că numerele z 1 + 2
, z2 + , z3 + sunt reale, să se determine z1.
z1 z1 z 2 z1 z 3

46
Soluţie :
1 a a a
Fie a = ∈ C . Atunci : z1 + , z 2 + , z3 + ∈R .
z1 z1 z2 z3
a a
deci : z k + = z k + , k = 1, 2, 3 ⇒ z k2 z k + a z k = z k2 z k + a z k ,
zk zk
k=1,2,3 Deci: z k r 2 + a z k = z k r 2 + a z k , k = 1, 2, 3 ; unde
r = z1 = z 2 = z 3 .
Înmulţind cu z k , obţinem:
z r + a r 2 = r 4 + a z k2
2
k
2
⇒ z k2 (r 2 − a ) = r 2 (r 2 − a 2 ).
r 2 (r 2 − a 2 )
Dacă r 2 − a ≠ 0, atunci z k2 = , k = 1, 2, 3.
r2 − a
Deci z12 = z 22 = z 32 . Din z12 = z 22 şi z1 ≠ z 2, rezultă că z1 = − z 2. Analog
z1 = − z 3 . Ar rezulta că z 2 = z 3 , fals.
1 1
Deci r 2 = a . Atunci r 2 = a ⇒ = r2 ⇒ = r 2 ⇒ r 3 = 1 . Deci z1 = 1.
z1 z1

R3.2.4 Fie z1, z 2 , z 3 ∈ C ,


z 1 = z 2 = z 3 = 1, z + z + z 2
1
2
2
2
3 = 0 şi z 1 + z 2 + z 3 ≠ 0 . Să se arate
că z + z + z − 3 z 1 z 2 z 3 = 4 .
3
1
3
2
3
3

Soluţie :
z13 + z 23 + z 33 − 3 z1 z 2 z 3 = z1 + z 2 + z 3 ⋅ z12 + z 22 + z 32 − z1 z 2 − z1 z 3 − z 2 z 3 =
= z1 + z 2 + z 3 ⋅ z1 z 2 + z 2 z 3 + z1 z 3 .
Dar
1 1 1 z1 z 2 + z 2 z 3 + z1 z 3
z1 + z 2 + z 3 = z1 + z 2 + z 3 = + + = =
z1 z 2 z 3 z1 z 2 z 3
= z1 z 2 + z 2 z 3 + z1 z 3 ; ( z1 + z 2 + z 3 ) 2 = z12 + z 22 + z 32 + 2 ( z1 z 2 + z1 z 3 + z 2 z 3 ) .
Cum z12 + z 22 + z 32 = 0 , obţinem:
2
z 1 + z 2 + z 3 = 2 z1 z 2 + z 2 z 3 + z 1 z 3 = 2 z1 + z 2 + z 3 .
Obţinem: z1 + z 2 + z 3 = 2 = z1 z 2 + z 2 z 3 + z1 z 3 .

47
R3.2.5 Dacă z ∈ C , a ∈ C , z = 1, n ∈ Ν * , să se arate că:
2n


k =1
zk + a ≥ n z −1 .

Soluţie :
Pentru oricare numere complexe x şi y avem:
x + a + y + a = x + a + − y − a ≥ ( x + a ) + ( − y − a ) = x − y . Atunci:

( )( )
2n

∑z
k =1
k
+ a = z 2 n + a + z 2 n −1 + a + z 2 n − 2 + a + z 2 n −3 + a + ... +

( z2 + a +
+ z + a ) ≥ z 2 n − z 2 n −1 + z 2 n − 2 − z 2 n −3 + ... + z 2 − z = z 2n−1 ⋅ z − 1 +
z 2n−3 ⋅ z −1 +
+ z 2 n −5 ⋅ z − 1 + ... + z ⋅ z − 1 = n z − 1 .

R3.2.6 a) Fie z1 , z 2 , z 3 ∈ C * astfel încât z1 = z 2 = z 3 şi z1 + z 3 = 2z 2 . Să


se arate că z1 = z 2 = z 3 .
b) Fie z1 , z 2 , z 3 ∈ C astfel încât z1 + z 3 = 2z 2 şi z ∈C ,
z ∉ {z1 , z 2 , z 3 }. Să se arate că dacă z − z1 + 2 z + z 2 + z − z 3 = 4 z , atunci
z − z1
> 0.
z − z3
Soluţie :
a)
2 z 2 = z1 + z 3 ≤ z1 + z 3 = 2 z 2 ⇒ z1 + z 3 = z 1 + z 3 ⇒ z3 =αz1 ,
cu α > 0 ⇒ z 3 = α z 1 ⇒ α = 1 ⇒ z 1 = z 2 = z 3 .
b) 4 z = z − z1 + 2 z + z 2 + z − z 3 ≥ z − z1 + 2 z + 2 z 2 + z − z 3 =
4 z ⇒ z − z1 + z + z 2 + z + z 2 + z − z 3 =
( z − z1 ) + ( z + z 2 ) + ( z + z 2 ) + ( z − z 3 ) ⇒ z − z1 = α ( z + z 2 ) , α > 0 şi
z − z1 α
z − z 3 = ( z + z 2 ), β > 0, deci: = > 0.
z − z3 β

48
R3.2.7 Fie z1 , z 2 ∈ C * astfel încât z 1 + z 2 = z 1 = z 2 . Să se calculeze
z2
.
z1
Soluţie :
z2 z2
Fie = x + iy, x, y ∈ R . Atunci z1 = z 2 implică = 1 , adică
z1 z1
z2
= 1 , de unde (1 + x ) + y 2 = 1 .
2
x 2 + y 2 = 1 , iar z1 + z 2 = 1 implică 1 +
z1
 x 2 + y 2 = 1 1 3
Deci:  . Obţinem x = − , y = ± . Există două soluţii:
(1 + x ) + y = 1
2 2
2 2
z2 1 3 z 1 3
= − +i şi 2 = − − i .
z1 2 2 z1 2 2

R3.2.8 Dacă z ∈ C cu z 4 + 2 = z 2 , atunci z ≤ 2 . În ce caz avem


egalitate?

Soluţie :
(1)
Avem z 4 = ( z 4 + 2) − 2 ≤ z 4 + 2 + 2 = z 2 + 2 . Deci
4 2 2 2
z − z − 2 ≤ 0 , adică ( z + 1) ( z − 2) ≤ 0 , deci z ≤ 2 . Egalitate în
2
(1) avem dacă ∃ λ > 0 astfel încât z 4 + 2 = −2 λ . Rezultă că − 2λ = z ,
2
adică z = 2 λ ≤ 2, cu egalitate pentru λ = 1. Obţinem z 4 = −4 cu rădăcinile
± (1 ± i ) .

R3.2.9 Fie k, n ∈ N * şi z1 , z 2 ,..., z n ∈ C * cu acelaşi modul astfel încât


1 1 1
z1k + z 2k + ... + z nk = 0 . Demonstraţi că k
+ k + ... + k = 0 .
z1 z 2 zn

Soluţie :
Fie z 1 = z 2 = ... = z n = r > 0 şi z 1k + z 2k + ... + z nk = 0 . Avem:
1 1 1 z1k z 2k z nk
+ + ... + = + + ... + =
z1k z 2k z nk z1k ⋅ z1k z 2k ⋅ z 2k z nk ⋅ z nk

49
z1k z 2k z nk
2k
+ 2k
+ ... + 2k
=
z1 z2 zn

=
z1
z1k
2k
+
z2
z 2k
2k
+ ... +
z nk
zn
2k
=
r
1
2k
(z
k
1 )
+ z 2k + ... + z nk =

r
1 k
2k
(
z1 + z 2k + ... + z nk = 0 . )
R3.2.9 Fie z ∈ C , z = 1 . Să se arate că:
n 1 + z + 1 + z 2 + ... + 1 + z 2 n +1 ≥ 2 n , n ∈ N * .

Soluţie :
Avem:
n 1 + z + 1 + z 2 + ... + 1 + z 2n+1 = n 1 + z + 1 + z 2 + 1 + z 3 + ( 1 + z 4 + ( )
) (
+ 1 + z 5 + ... + 1 + z 2 n + 1 + z 2 n +1 )
≥ n 1 + z + 1 + z 2 − 1 − z 3 + 1 + z 4 − 1 − z 5 + ... +
+ 1 + z 2n − 1 − z 2n+1 = n 1 + z + z 2 ⋅ 1 − z + z 4 ⋅ 1 − z + ... + z 2n ⋅ 1 − z = n 1 + z + n 1 − z =
= n (1 + z + 1 − z )≥ n 1 + z − 1 + z = 2 n.

50
4. Aplicaţii ale numerelor complexe în geometrie

În această temă vom utiliza numerele complexe pentru rezolvarea şi


generalizarea unor probleme de geometrie. Deşi metoda vectorială şi metoda
numerelor complexe sunt echivalente, fiecare dintre ele rezolvă cu uşurinţă
anumite probleme şi în acelaşi timp creează, în limbajul lor specific, noi
probleme.

4.1.1 Amintim câteva rezultate, care vor fi utile în cele ce urmează. Vom nota
cu M(z) punctul M de afix z.
4.1.2 Distanţa dintre punctele M1(z1) şi M2(z2) este M1M2 = z1 − z 2 .

4.1.3 Afixul punctului M care împarte segmentul [M1M2] în raportul k,


z − kz 2
adică MM 1 = k MM 2 este z = 1 , unde M(z), M1(z1), M2(z2).
1− k
4.1.4 Consecinţă . Afixul mijlocului M al segmentului [M1M2] este
z + z2
z= 1 ; Afixul g al centrului de greutate G al triunghiului M1M2M3 este
2
z + z 2 + z3
g= 1 ; patrulaterul M1M2M3M4 este paralelogram dacă şi numai
3
dacă z1+z3 = z2+z4, unde Mi(zi), i = 1, 2, 3, 4.

4.1.5 Condiţia de coliniaritate : Punctele M1(z1), M2(z2), M3(z3) sunt


coliniare dacă şi numai dacă există k1, k2, k3 ∈ R cu k1+ k2+ k3 = 0 şi
k1 z1 + k 2 z 2 + k 3 z 3 = 0 .
Demonstraţie : Dacă M1 , M2 , M3 sunt coliniare, atunci există
z − kz 3
k ∈ R cu M 2 M 1 = k M 2 M 3 . Deci z 2 = 1 , adică z1 − (1 − k ) z 2 − kz 3 = 0 .
1− k
Pentru k1 = 1, k2 = k-1, k3 = -k obţinem concluzia. Reciproc, din
k1 z1 + k 2 z 2 + k 3 z 3 = 0 cu k2= -k1-k3, obţinem k1 ( z1 − z 2 ) = −k 3 ( z 3 − z 2 ) . Pentru
k3 z − kz 3
k=− obţinem z 2 = 1 , adică M1 , M2 , M3 sunt coliniare.
k1 1− k

4.1.6 Măsurara unghiului orientat ∠ M 1OM 2 , în sens trigonometric,


(semidreapta OM1 se roteşte în sens trigonometric peste semidreapta OM2),

51
z2
faţă de un reper cu originea în O este: µ (∠ M 1OM 2 ) = arg , unde z1, z2 sunt
z1
afixele punctelor M1, respectiv M2.
4.1.7 Consecinţă : Dacă M1(z1), M2(z2), M3(z3), atunci măsura unghiului
orientat (în sens trigonometric) ∠ M 1M 2 M 3 este:
z3 − z 2
µ (∠ M 1 M 2 M 3 ) = arg .
z1 − z 2

M3
y
M1

z3 – z2 Translatăm M2 în originea O şi aplicăm


M2 4.1.5 .

z1 – z2
x

O
Demonstraţie :
z3 − z 2
4.1.8 Consecinţă : Dacă M1(z1), M2(z2), M3(z3) şi = ρ ε , unde ρ > 0,
z1 − z 2
M2 M3
ε = cos α + i sin α , cu α ∈ [0, 2π ) , atunci =ρ
M1M2
şi µ(∠M1M2 M3 ) = min(α, 2π −α) .

4.1.9 Formula rotaţiei în comple Dacă M3(z3) se obţine printr-o rotaţie cu


centrul în M2(z2) şi unghi α ∈ [0, 2π ) , a punctului M1(z1), atunci :
z 3 = z 2 + ( z1 − z 2 )ε , unde ε = cos α + i sin α , dacă rotaţia se efectuează în sens
trigonometric sau ε = cos(2π − α ) + i sin (2π − α ) , dacă rotaţia se efectuează în
sens invers trigonometric.

4.1.10 Consecinţă : Triunghiul ABC este echilateral dacă şi numai dacă


π π
c = a + (b − a )ε , unde ε = cos + i sin , dacă ∆ ABC este orientat în sens
3 3
5π 5π
trigonometric, sau ε = cos + i sin , dacă ∆ ABC este orientat în sens
3 3
invers trigonometric.

52
4.1.11 Unghiul a două drepte. Dacă M1(z1), M2(z2), M3(z3), M4(z4) sunt
puncte distincte în plan, diferite de origine, atunci măsura unghiului orientat (în
sens trigonometric) al dreptelor M1M2 şi M3M4 este :
z −z
µ (∠(M 1 M 2 , M 3 M 4 ) ) = arg 2 1 .
z 4 − z3
z3 − z1
4.1.12 Consecinţă : Dacă = ρ ε, unde ρ > 0, ε = cos α + i sin α ,
z4 − z3
MM
α ∈ [0, 2π ) , atunci: 1 2 = ρ şi µ (∠(M 1 M 2 , M 3 M 4 ) ) = min (α , 2π − α ) .
M3 M4
z − z1
4.1.13 Consecinţă : M 1 M 2 ⊥ M 3 M 4 ⇔ 2 ∈ iR * .
z 4 − z3
z − z1
M 1 M 2  M 3 M 4 ⇔ 2 ∈ R* .
z 4 − z3

4.1.14 Punctele M1(z1), M2(z2), M3(z3), M4(z4), distincte, sunt conciclice dacă
şi numai dacă raportul anarmonic al afixelor z1, z2, z3, z4 este real, adică:
z −z
(z1 , z 2 , z 3 , z 4 ) = 3 1 : z 4 − z1 ∈ R * .
z3 − z 2 z 4 − z 2
Demonstraţie :
Cazul I: Dacă M1 şi M2 sunt de aceeaşi parte a dreptei M3M4 avem:
z − z3 z − z4
arg 1 = arg 1 .
z 2 − z3 z2 − z4
Cazul II: Când M1 şi M2 sunt separate de dreapta M3M4 avem:
z − z3 z −z
arg 1 − arg 1 4 = π .
z2 − z3 z2 − z4

4.1.15 Triunghiuri asemenea . Triunghiurile A1A2A3 şi A1' A2' A3' , la fel


orientate, sunt asemenea, în această ordine, dacă şi numai dacă
a 2 − a1 a 2' − a1'
= .
a3 − a1 a3' − a1'

53
Demonstraţie: Avem ∆ A1 A 2 A 3 ~ ∆ A1' A 2' A 3'
 A1 A2 A1' A2'
 =
⇔  A1 A3 A1' A3' ⇔
∠ A A A ≡ ∠ A ' A ' A '
 3 1 2 3 1 2

a −a a 2' − a1'
 2 1
= '
 a3 − a1 a3 − a1' a 2 − a1 a 2' − a1'
 ⇔ = .
 a 2 − a1 a 2' − a1' a3 − a1 a3' − a1'
arg = arg
 a3 − a1 a3' − a1'

4.1.16 Observaţie: Triunghiurile A1A2A3 şi A1' A2' A3' , la fel orientate, sunt
asemenea, dacă şi numai dacă: a1' (a 2 − a3 ) + a 2' (a3 − a1 ) + a3' (a1 − a 2 ) = 0 .
4.1.17 Observaţie: Triunghiurile A1A2A3 şi A1' A2' A3' , invers orientate, sunt
a 2 − a1 a 2' − a1'
asemenea în această ordine, dacă şi numai dacă: = .
a3 − a1 a3' − a1'
Demonstraţie : Se consideră triunghiul M1M2M3 simetric cu A1' A2' A3' faţă de O
Triunghiul M1M2M3 are afixele vârfurilor a1' , a 2' , a3' şi este la fel orientat cu
triunghiul A1A2A3. Folosim 4.14, obţinem relaţia cerută.

4.1.18 Consecinţă : Triunghiul A1A2A3 este echilateral dacă şi numai dacă:


a12 + a 22 + a32 = a1 a 2 + a 2 a3 + a3 a1 .
Demonstraţie: Triunghiul A1A2A3 este echilateral
⇔ ∆ A1 A 2 A 3 ~ ∆ A 2 A3 A1 ⇔
( )
a 2 a 2 − a3 + a3 (a3 − a1 ) + a1 (a1 − a 2 ) = 0 ⇔ a12 + a 22 + a32 = a1 a 2 + a 2 a3 + a3 a1
.
4.1.19 Aria unui triunghi. Dacă a1, a2, a3 sunt afixele vârfurilor triunghiului
A1A2A3, notat în sens trigonometric, atunci
1
(
σ [A1 A2 A3 ] = Im a1 a 2 + a 2 a3 + a3 a1 .
2
)

54
Fără a restrânge generalitatea
problemei, putem considera originea y
sistemului ortogonal de axe în
interiorul triunghiului. A1
Fie a1 = r1 (cosθ 1 + i sin θ 2 )
a 2 = r2 (cosθ 2 + i sin θ 2 )
a3 = r3 (cosθ 3 + i sin θ 3 ) .
Atunci:
a1a2 + a2 a3 + a3 a1 = r1r2 [cos(θ 2 − θ 1 ) +
O x
+ i sin (θ 2 − θ 1 )] + r2 r3 [cos (θ 3 − θ 2 ) + A2

+ i sin (θ 3 − θ 2 )] + r1 r3 [cos (θ 1 − θ 3 ) +
A3
i sin (θ 1 − θ 3 )] . Deci:

Demonstraţie :
1
2
( )1
2
1
2
1
Im a1 a 2 + a 2 a 3 + a3 a1 = r1 r2 sin (θ 2 − θ 1 ) + r2 r3 sin (θ 3 − θ 2 ) + r1 r3 sin (θ 1 − θ 3 ) =
2

= σ [A1OA2 ] + σ [A2 OA3 ] + σ [A3 OA1 ] = σ [A1 A2 A3 ] .

4.1.20 Observaţie : Formula se poate extinde pentru un poligon conve Dacă


A1A2…An, n ≥ 3 este un poligon convex, notat în sens trigonometric, iar a1,
a2,…, an sunt afixele vârfurilor, atunci:
1
( )
σ [A1 A2 ... An ] = Im a1 a 2 + a 2 a 3 + .... + a n −1 a n + a n a1
2
(Formula lui Kiril
Docev). Demonstraţie prin inducţie (vezi [3] ).

4.1.21 Afixul ortocentrului unui triunghi. Faţă de un reper cartezian cu


originea O în centrul cercului circumscris triunghiului ABC, afixul
ortocentrului H al triunghiului ABC, este: h = a + b + c , unde H(h), A(a), B(b),
C(c). Demonstraţie :
A

Fie O1 simetricul lui O faţă de BC. Atunci


. O
AHO1O este paralelogram. Rezultă:
H
a + o1 = h + o . Obţinem : h = a + o1 = a + b + c .
B C
. O1
55
4.1.22 Consecinţă : Faţă de un reper cu originea în centrul cercului
circumscris triunghiului ABC, centrul ω al cercului lui Euler al triunghiului
a+b+c
ABC este : ω= , unde ω (ω ) .
2
Demonstraţie : Ω este mijlocul segemntului [OH].

4.1.23 Caracterizarea triunghiului dreptunghic . Triunghiul ABC înscris în


cercul C(O, R) este dreptunghic dacă şi numai dacă a + b + c = R , unde A(a),
B(b), C(c).
Demonstraţie : Dacă triunghiul ABC este dreptunghic cu unghiul drept în A,
atunci B şi C sunt diametral opuse, deci b = −c , de unde a + b + c = a = R .
2
Reciproc dacă a + b + c = R , atunci a+b+c = R 2 , adică
 2
R2 R2 
(a + b + c ) R ++  = R 2 , deci
 a b c 
(a + b + c )  1 + 1 + 1  = 1 ⇔ ⇔ (a + b )(b + c )(c + a ) = 0 adică două
a b c
din punctele A, B, C sunt diametral opuse.

Bibliografie

1. Andrei Gh., Caragea C., Cucurezeanu I., Bordea Gh., Probleme de algebră
pentru concursurile de admitere şi olimpiade şcolare, E.D.P., Bucureşti,
1993
2. Andrica D., Bişboacă N., Numere complexe de la…a…la…z, Aditura
Millenium, Alba Iulia, 2001
3. Andrica D., Varga C., Văcăreţu D., Teme de geometrie, Editura Promedia
Plus, Cluj Napoca, 1997
4. Cocea C., 200 de probleme din geometria triunghiului echilateral, Editura
Gh. Asachi, Iaşi, 1992
5. Dincă M., Chiriţă M., Numere complexe în matematica de liceu, Editura All
Educational, Bucureşti, 1996
6. Nicula V., Numere complexe. Probleme şi exerciţii pentru clasa a X-a,
Editura Scorpion 7, Bucureşti, 1993

56
Probleme rezolvate (4)
R4.2.1 Fie ABCDE un pentagon complex, iar M, N, P, Q, X, Y respectiv
mijloacele segmentelor (BC), (CD), (DE), (EA), (MP), (NQ). Să se arate că
XY  AB.
Soluţie :
Notăm cu literele mici corespunzătoare, afixele vârfurilor. Avem:
b + c
m = ,
2
c+d d +e e+a m+ p b+c+d +e
n= , p= , q= , x= = ,
2 2 2 2 4
a+d +c+e b−a XY 1
y= . Atunci: x − y = . Deci = şi XY AB.
4 2 AB 4
R4.2.2 Dacă pe laturile unui patrulater oarecare ABCD construim în
exterior pătrate de centre O1, O2, O3, O4 , atunci dreptele O1O3 şi O2O4 sunt
perpendiculare.
Soluţie :
Fie O1, O2, O3, O4 centrele pătratelor construite pe (AD), (DC), (CB),
respectiv (BA). Atunci O1 este transformatul punctului D printr-o rotaţie de
π
centru mijlocul segmentului (AD) şi unghi (vârfurile patrulaterului ABCD
2
fiind notate în sens trigonometric). Deci:
a+d  a + d  a + d + i(d − a ) c + d + i (c − d )
o1 = + d − i = . Analog o2 = ,
2  2  2 2
b + c + i(b − c ) a + b + i (a − b )
o3 = , o4 = . Avem:
2 2
a+d −b−c d −a−b+c
o1 − o3 = +i ,
2 2
c+d −a−b c−d −a+b o1 − o3
o2 − o4 = +i . Deci = −i . Obţinem
2 2 o2 − o4
O1O3=O2O4 şi O1O3 ⊥ O2 O4 .

R4.2.3 Se dă un triunghi ABC şi în interiorul său se consideră triunghiul


A ' B ' C ' asemenea cu triunghiul dat şi având aceeaşi orientare (adică vârfurile
celor două triunghiuri sunt notate în acelaşi sens de rotaţie). Fie A '' , B '' , C ''

( )( )( )
aparţinând segmentelor AA ' , BB ' , CC ' astfel încât:
AA ''
=
BB ''
=
CC ''
.
A '' A ' B '' B ' C '' C '

57
Să se arate că ∆A '' B '' C '' ~ ∆ABC .
Soluţie :
AA '' BB '' CC ''
Fie = == λ . Notăm cu litere mici afixele corespunzătoare
A '' A '
B '' B ' C '' C '
a − λa ' b − λb ' '' c − λc '
vârfurilor. Atunci: a = ,b = , c =
'' ''
. Deoarece
1− λ 1− λ 1− λ
∆ABC ~ ∆ A ' B ' C ' rezultă : a ' (b − c ) + b ' (c − a ) + c ' (a − b ) = 0 . Se verifică că:
a ' ' (b − c ) + b '' (c − a ) + c '' (a − b ) = 0 , adică ∆A '' B '' C '' ~ ∆ABC .

R4.2.4 Pe laturile patrulaterului convex ABCD se construiesc în exterior


triunghiurile echilaterale ABM, BCN, CDP, DAQ. Să se arate că patrulaterele
ABCD şi MNPQ au acelaşi centru de greutate.
Soluţie :
Notăm vârfurile patrulaterului în sens trigonometric. Atunci:
m = b + (a − b )ε , n = c + (b − c )ε , p = d + (c − d )ε , q = a + (d − a )ε , unde
π π
ε = cos + i sin . Adunând aceste relaţii, obţinem: m+n+p+q = a+b+c+d,
3 3
deci patrulaterele ABCD şi MNPQ au acelaşi centru de greutate.
R4.2.5 Dacă pe laturile triunghiului ABC construim în exterior triunghiurile
asemenea AC ' B, BA ' C , CB ' A , atunci triunghiurile ABC şi A ' B ' C ' au acelaşi
centru de greutate.
AB ' BC ' CA '
Soluţie : Fie = = = r şi
AC AB BC
( ) ( ) ( )
α = µ ∠ ABC ' = µ ∠ BCA ' = µ ∠ CAB ' . Atunci, folosind 4.1.8, avem:
b ' = a + (c − a )rε , c ' = b + (a − b )rε , a ' = c + (b − c )rε , unde
ε = cos α + i sin α . Adunând aceste relaţii obţinem a ' + b ' + c ' = a + b + c ,
adică cele două triunghiuri au acelaşi centru de greutate.
Observaţie : Dacă triunghiurile construite în exterior sunt echilaterale,
obţinem cunoscuta problemă a lui Toricelli.
R4.2.6 Pe laturile triunghiului ABC se construiesc în exterior triunghiurile
echilaterale ABC ' , CAB ' , BCA ' . Să se arate că centrele de greutate ale acestor
triunghiuri formează un triunghi echilateral.
Soluţie :
Fie G1, G2, G3 centrele de greutate ale triunghiurilor ABC ' , CAB ' , BCA ' .
Notăm cu litere mici corespunzătoare, afixele vârfurilor. Atunci:

58
c ' = b + (a − b )ε , a ' = c + (b − c )ε , b ' = a + (c − a )ε , unde
π π a + 2b + (a − b )ε c + 2a + (c − a)ε
ε = cos + i sin ; g1 = , g2 = ,
3 3 3 3
b + 2c + (b − c)ε
g1 = . Obţinem:
3
b + 2c + (b − c)ε 2 a − c − b 2c − a − b 2
g 3 + (g 2 − g 3 )ε = + ⋅ε + ⋅ ε = g1 ,
3 3 3
deoarece ε 2 = ε − 1 . Deci g 3 + (g 2 − g 1 )ε = 0 , adică ∆G1G2 G3 este
echilateral.

R4.2.7 Se consideră în plan un triunghi A1A2A3 şi un punct P0. Se defineşte


As = As −3 , ∀ s ≥ 4, s ∈ N şi se construieşte un şir de puncte P0, P1, …, astfel
încât Pk+1 este imaginea punctului Pk prin rotaţia de centru Ak+1 şi de unghi
1200, în sensul arcelor de ceasornic, k=0, 1, 2, … . Dacă P1986 = P0, atunci
triunghiul A1A2A3 este echilateral.
Soluţie :
Avem: A1=A4=A7=…; A2=A5=A8=…; A3=A6=A9=… . Dar Pk+1=
= R A k +1 , 120 o
(Pk ) , deci pk +1 = ak +1 + ( pk − ak )ε , unde
2π 2π
ε = cos + i sin .
3 3
[
Obţinem : p k + 1 = (1 − ε ) a k + 1 + a k ε + ... + a 1 ε k
]. Din 0 = P = P ,
0 1986

obţinem: 0 = (1 − ε )[a 1986 + a1985ε + ... + a1ε 1985


] = (1 − ε )[a + a ε + a ε ]⋅ 662 .
3 2 1
2

Deci a3 + a 2 ε + a1ε = 0 . 2
Cum ε + ε +1 = 0, 2
obţinem:
a 3 = a 1 + ε (a 2 − a 1 ) , adică A1A2A3 este echilateral.

R4.2.8 Fie A1A2A3A4 un patrulater inscriptibil. Se notează cu H1, H2, H3, H4


ortocentrele triunghiurilor A2A3A4, A3A4A1, A4A1A2, A1A2A3. Să se arate că
patrulaterele A1A2A3A4 şi H1H2H3H4 sunt congruente.
Soluţie :
Avem: h1 = a2 + a3 + a4 , h2 = a3 + a4 + a1 , h3 = a4 + a1 + a2 ,
h4 = a1 + a2 + a3 .
Atunci: H 1 H 2 = a1 − a2 = A1 A2 şi
h3 − h2 a − a3 a − a2
µ (∠ H 1 H 2 H 3 ) = arg = arg 2 = = arg 3 = µ (∠ A1 A2 A3 ) .
h1 − h2 a 2 − a1 a1 − a 2
Analog celelalte.

59
R4.2.9 Fie z1, z2, z3 numere complexe distincte, având acelaşi modul r.
1 1 1 1
Arătaţi că: + + ≥ 2 .
z1 − z 2 ⋅ z1 − z 3 z 2 − z1 ⋅ z 2 − z 3 z 3 − z1 ⋅ z 3 − z 2 r
Soluţie :
Considerăm un triunghi care are afixele z1, z2, z3 şi fie originea axelor de
coordonate în centrul cercului circumscris triunghiului. Notăm
z 1 − z 2 = c , z 2 − z 3 = a , z1 − z 3 = b şi r = z k = R , k = 1, 2, 3 .
Inegalitatea devine:
1 1 1 1 abc 4 RS
+ + ≥ 2 ⇔ a+b+c ≥ 2 ⇔ 2p ≥ ⇔ R p ≥ 2S ⇔
bc ab ac R R R2
⇔ R p ≥ 2 ρ p ⇔ R ≥ 2 ρ , unde ρ raza cercului înscris. (inegalitatea lui
Euler).
R4.2.10 Fie z1, z2, …, zn afixele vârfurilor A1, A2, …, An ale unui poligon
înscris în cercul cu centrul în origine şi se rază r. considerăm
z + z 2 + ... + z n
g= 1 .
n
2 2 2 2
a) Arătaţi că g − z1 + g − z 2 + ... + g − z n + n g = nr 2 .
b) Demonstraţi inegalitatea: g − z1 + g − z 2 + ... + g − z n ≤ nr .
c) Deduceţi că apoi în orice triunghi are loc inegalitatea:
9
m a + mb + m c ≤ R .
2
Soluţie :
a) Avem:
( )
n n n n n

∑ g − zk = ∑ (g − zk ) g − zk = ∑ g g − ∑ gz k − ∑ g z k +
2

k =1 k =1 k =1 k =1 k =1
n
+ ∑ zk zk = n g
2 2
− g ng − gn g + nR 2 = nR 2 − n g .
k =1
2 2
 n   n   n 2 
n
 n
 ∑ g − z k  =  ∑ g − z k ⋅1 ≤  ∑ g − z k  ∑12  = n∑ g − z k
2
b) ≤
 k =1   k =1   k =1  k =1  k =1

≤ nr n = n r .
2 2 2

c) Din punctul b) deducem: GA+GB+GC ≤ 3R, adică


2 2 2 9
ma + mb + mc ≤ 3R ⇒ ma + mb + mc ≤ R .
3 3 3 2

60
R4.2.11 Dacă z1, z2, z3, z4 ∈ C * sunt distincte două câte două şi
z1 + z 2 + z 3 + z 4 = 0 şi z1 = z 2 = z 3 = z 4 = 1 , atunci sunt afixele vârfurilor
unui dreptunghi.
Soluţie :
Din z1 + z 2 + z 3 = − z 4 rezultă z 1 + z 2 + z 3 = − z 4 = 1 . Folosind relaţia
4.1.22 deducem că ∆Z 1 Z 2 Z 3 este dreptunghic, unde Z i ( z i ) , i =1, 2, 3, 4.
Analog pentru celelalte unghiuri.

R4.2.12 Afixele z1, z2, z3 ale vârfurilor triunghiului A1A2A3 verifică


condiţiile:
a) z1 = z 2 = z 3 = 1 ;
2 2
b) z1 + z 2 + z 2 + z 3 2 + z 3 + z1 = 3 .
Demonstraţi că A1A2A3 este triunghi echilateral.
Soluţie :
Din b)
( ) ( ) ( )
obţinem: (z1 + z 2 ) z1 + z 2 + (z 2 + z3 ) z 2 + z3 + (z3 + z1 ) z3 + z1 = 3 ⇒
(z1 + z 2 + z 3 )(z 1 + z 2 + z 3 ) = 0 ⇒ z1 + z 2 + z 3
2
= 0 . Deci OH=0, unde O
reprezintă centrul cercului circumscris, iar H ortocentrul triunghiului A1A2A3.
Din OH=0 rezultă O=H, adică ortocentrul triunghiului coincide cu
centrul cercului circumscris. Deducem că A1A2A3 este triunghi echilateral.

R4.2.13 Fie ABCD un paralelogram şi M un punct în planul său. Să se arate


că: MA ⋅ MC + MB ⋅ MD ≥ AB ⋅ BC .
Soluţie :
Fie z afixul punctului M, iar a, b, c, d afixele punctelor A, B, C, D.
Folosind faptul că a+c=b+d, obţinem:
(a − b )(b − c ) = (z − b )(z − d ) − (z − a )(z − c ) . Trecând la module rezultă:
(a − b)(b − c) = (z − b)(z − d ) − (z − a)(z − c) ≤ (z − b)(z − d ) + (z − a) ⋅
⋅ ( z − c ) = z − b ⋅ z − d + z − a ⋅ z − c , adică
AB ⋅ BC ≤ MB ⋅ MD + MA ⋅ MC .

R4.2.14 Pe laturile triunghiului A1A2A3 considerăm punctele M1 ∈ (A2A3),


M2 ∈ (A1A3), M3 ∈ (A1A2) astfel încât M 1 A2 =λ 1 M 1 A3 ; M 2 A3 =λ 2 M 2 A1 ;
M 3 A1 =λ 3 M 3 A2 . Atunci aria triunghiului M1M2M3 este :

61
1 − λ1λ 2 λ 3
σ [M 1 M 2 M 3 ] = σ [A A A ] .
(1 − λ1 )(1 − λ 2 )(1 − λ3 ) 1 2 3
Soluţie :
Notăm cu literele mici corespunzătoare afixele punctelor. Atunci:
a1 − λ1 a3 a − λ 2 a1 a − λ3 a 2
m1 = , m2 = 3 , m3 = 1 . Deci:
1 − λ1 1 − λ2 1 − λ3
1
[
σ [M1M 2 M 3 ] = Im m1m2 + m2 m3 + m3 m1 = Im
2
] 1
2
 1−λλ 1 2λ3
 1 − λ1λ 2 λ3
 ( )
1 2 + a2a3 + a3a1  =
aa σ [A A A ].
(1 − λ1 )(1 − λ 2 )(1 − λ3 ) 1 2 3
(1−λ1 )(1−λ2 )(1−λ3 ) 
Observaţie 1 : Pentru σ [M 1 M 2 M 3 ] = 0 , regăsim teorema lui Menelaus:
λ1λ 2 λ3 = 1 .
Observaţie 2 . Dacă M1, M2, M3 sunt picioarele bisectoarelor interioare ale unui
2abc
triunghi ABC, atunci σ [M 1 M 2 M 3 ] = σ [ABC ] , unde a, b, c
(a + b )(b + c )(c + a )
sunt lungimile laturilor.

R4.2.15 Se consideră pentagonul inscriptibil ABCDE. Notăm cu H1, H2, H3,


H4, H5 ortocentrele triunghiurilor ABC, BCD, CDE, DEA, EAB şi cu M1, M2,
M3, M4, M5 mijloacele laturilor DE, EA, AB, BC şi respectiv CD. Să se arate că
dreptele H1M1, H2M2, H3M3, H4M4 şi H5M5 sunt concurente.
Soluţie :
Alegem un reper cu originea O în centrul cercului circumscris
pentagonului. Dacă afixele punctelor A, B, C, D, E sunt a, b, c, d, respectiv e, se
d +e
ştie că afixul punctului H1 este h1=a+b+c, iar afixul lui M1 este m1= şi
2
analoagele. Un punct P de afix p aparţine dreptei H1M1 dacă şi numai dacă
d +e
există t1 real astfel încât p = (1 − t1 )(a + b + c ) + t1 . Analog P ∈ H2M2 dacă
2
e+a
şi numai dacă există t2 ∈ R, astfel încât p = (1 − t 2 )(b + c + d ) + t 2
2
2 a+b+c+d +e
ş.a.m.d. Pentru t1 = avem p = ∈ HiMi, oricare ar fi i = 1, 5 ,
3 3
deci dreptele sunt concurente.

62
5. Ecuaţii în C

5.1 Nu ne propunem să epuizăm problematica ecuaţiilor peste corpul


numerelor complexe. Vom scoate în evidenţă numai câteva aspecte legate de
rădăcinile de ordinul n ale unui număr complex, modulele rădăcinilor unei
ecuaţii, etc., aşa cum intervin în multe probleme de concurs. În acest scop,
reamintim:

5.1.1 Forma trigonometrică a unui număr complex: dacă z = x+iy ∈ C ,


atunci z = r (cos t + i sin t ), unde r = x 2 + y 2 se numeşte modulul numărului
complex z şi se notează z , iar t se numeşte argumentul (redus) al lui z. Acesta
se notează cu arg z şi reprezintă mulţimea soluţiilor sistemului:
 x
cos t = r
 , unde 0 ≤ t ≤ 2π , pentru z ≠ 0 .
sin t = y
 r

5.1.2 Rădăcină de ordinul n a unui număr complex


Dacă a ∈ C , n ∈ N , n ≥ 2 , atunci orice număr complex z care verifică
ecuaţia z n = a se numeşte rădăcina de ordinul n a lui a.
Pentru a = r (cos t + i sin t), r > 0, t ∈ [0,2π ) , avem rădăcinile de ordinul
n:
 t + 2kπ t + 2kπ 
z k = n r  cos + i sin , k = 0, n − 1 .
 n n 
În particular, dacă a=1, rădăcinile ecuaţiei z n = 1, n ∈ N , n ≥ 3 se
numesc rădăcinile de ordinul n ale unităţii şi se notează cu ε k , k = 0 , n − 1 .
2 kπ
2kπ 2kπ not n i
Deci ε k = cos + i sin = e , k = 0, n − 1 .
n n
Imaginile geometrice ale numerelor complexe ε k sunt vârfurile unui poligon
regulat cu n laturi, înscris în C(O,1).

5.1.3 Se numeşte rădăcină primitivă a ecuaţiei binome z n = 1, fiecare


rădăcină a ecuaţiei care nu este rădăcină nici a unei ecuaţii binome de grad mai
mic decât n.

63
5.1.4 Proprietăţi :
a) Fiecare rădăcină a ecuaţiei binome x n = 1 este de asemenea rădăcină a
fiecărei ecuaţii x q = 1 , pentru care n/q.
b) Rădăcinile comune ale ecuaţiilor binome x m = 1 şi x n = 1 sunt şi
rădăcinile ecuaţiei binome x d = 1 , unde d = (m,n) este c.m.m.d.c. al
numerelor m şi n.
2 kπ
i
c) Rădăcinile primitive ale ecuaţiei binome x = 1 , sunt date de x k = e
m m
, în
care (k,m) =1, 0 ≤ k ≤ m .
Demonstraţie
a) Este imediată.
2 pπ
i
b) Fie xp = e m
, p = 0, m − 1 , rădăcină a ecuaţiei xm = 1 şi
2 qπ
i
xq = e n
, q = 0, n − 1 rădăcină e ecuaţiei x n = 1 . Condiţia necesară şi
suficientă ca x p = x q este ca x p = x q şi ca arg x p = arg x q . Prima relaţie este
satisfăcută deoarece x p = x q =1. A doua are loc dacă există r ∈ Z astfel încât
2 pπ 2qπ p q
să avem = + 2rπ sau − = r.
m n m n
Dacă d = (m,n), atunci există m’, n’ ∈ N în aşa fel încât m = m’ d, n = n’
d, cu (m’, n’) =1. Ultima ecuaţie devine n’p - m’q = m’n’r d şi de aici rezultă că
m’/ n’p şi cum (m’, n’) = 1, rezultă că m’/p. Adică există p’ ∈ N în aşa fel
2 pπ 2 p ' m 'π 2 p 'π
încât p = p’m’. Deci arg x p = = = şi deci xp este
m m'd d
rădăcină a ecuaţiei x d = 1 , unde d = (m,n).
Reciproc, fiecare rădăcină a ecuaţiei binome x d = 1 este conform
proprietăţii a) şi rădăcină comună a ecuaţiilor x m = 1 şi x n = 1 , deoarece d/m şi
d/n.
c) Trebuie să găsim ecuaţia binomă x p = 1 , de gradul cel mai mic, care
admite rădăcina xk. Din x kp = 1 , deducem că există k ' ∈ Z astfel încât
2kpπ kp
= 2k 'π , adică = k' ∈Z .
m m
Dacă d = (k,m), atunci există k’,m’ astfel încât
k = k d , m = m d , (k , m ) = 1 . Înlocuind în ultima relaţie obţinem
' ' ' '

k ' pd k ' p
'
= ' ∈ Z şi cum k ' şi m ' sunt prime între ele, rezultă m ' / p . Deci cea
md m

64
m
mai mică valoare a lui p este p = m ' şi înlocuind în m = m ' d obţinem p = .
d
Rezultă că dacă xk este rădăcină a ecuaţiei binome x m = 1 , ecuaţia binomă de
m
gradul cel mai mic pe care o verifică rădăcina xk, este de gradul p = , unde
d
d=(k,m).
Dacă, în plus, xk este rădăcină primitivă a ecuaţiei binome x m = 1 ,
atunci aceasta este ecuaţia binomă de gradul cel mai mic care are pe xk
m
rădăcină. Adică trebuie să avem m = , d = (k,m). Deducem că trebuie d =
d
(k,m) = 1, adică k şi m sunt prime între ele.

5.1.5 Observaţie
Dacă (m,n)=1, ecuaţiile binome x n = 1 şi x m = 1 au numai x=1 rădăcină
comună.

5.1.6 Propoziţie
Dacă x este rădăcina primitivă a ecuaţiei binome x n = 1 , atunci
rădăcinile ecuaţiei sunt: x r , x r +1 , ... , x r + n −1 , ∀ r ∈ N .
Demonstraţie
Într-adevăr, x r + h , h = 0, n − 1 este rădăcină a ecuaţiei binome x n = 1 ,
( )
n
deoarece x r + h = e 2( r + h ) π i = 1 . Rămâne să arătăm că oricare două dintre cele n
numere α r ,α r +1 , ... ,α r + n −1 sunt distincte.
Să presupunem prin absurd că, pentru r + h1 ≠ r + h2 (cu h1 > h2 ) , am
( )
avea α r + h1 = α r + h2 . Atunci: α r + h1 α h1 − h2 − 1 = 0 şi deci, deoarece α r + h2 ≠ 0 , am
avea α h1 − h2 = 1 . Dar h1 − h2 < n şi deci α ar fi rădăcină a ecuaţiei binome
x h1 − h2 = 1 , de grad h1 − h2 < n . Contadicţie cu α este rădăcină primitivă a
ecuaţiei binome x n = 1 .

65
Bibliografie

1. Andrica D., Bişboacă N., Numere complexe de la a…la…z, Editura


Millenium, Alba Iulia 2001
2. Andrica D., Muşuroia N., O metodă de obţinere a unor identităţi
remarcabile, G.M. 1, 1996, pag. 13-18
3. Andrei Gh., Caragea C., Cucurezeanu I., Bordea Gh., Probleme de algebră
pentru concursuri de admitere şi olimpiade şcolare, E.D.P., Bucureşti ,1993
4. Bălună M. şi colectiv, Zece lecţii alese de matematică elementară, S.S.M.R,
1998
5. Becheanu M. şi colaboratori, Olimpiade de matematică, IX-X, 19960-1996,
Editura Gil, Zalău, 1997
6. Gorgotă V., Şerdean I., Ulmeanu S., Matematica în concursurile şcoalre,
IX-XII, Editura Paralela 45, 2002
7. Nicula V., Numere complexe, Probleme şi exerciţii pentru clasa a X-a,
Editura Scorpion 7, Bucureşti, 1993
8. Tămâian T., Probleme selectate din reviste şcoalre, Editura Cub Press,
Baia Mare, 2002

66
Probleme rezolvate (5)

R5.2.1 Fie n>2 un număr natural şi z ∈C \ {1} astfel încât z n = 1 .


2
1) Să se arate că 1 − z > .
n −1
2) Să se arate că pentru orice k ∈ Z , k nedivizibil cu n, are loc:
kπ 1
sin > .
n n −1
Soluţie :
( )
1) z n − 1 = ( z − 1) z n −1 + z n − 2 + ... + z + 1 , deci z satisface ecuaţia:
( ) (
z n −1 + z n − 2 + ... + z + 1 = 0 . Rezultă: z n −1 − 1 + z n − 2 − 1 + ... + ( z − 1) = −n şi )
[ ]
( z − 1) z n − 2 + 2 z n −3 + ... + (n − 1) = − n . Trecând la module obţinem:
n = z − 1 ⋅ z n − 2 + 2 z n −3 + 3 z n − 4 + ... + (n − 1) ≤ 1 − z ⋅ z ( n−2
+2 z
n −3
+ ... + (n − 1) )
n(n − 1) 2
, deci: n ≤ 1 − z ⋅ (1 + 2 + ... + (n − 1) ) = 1 − z ⋅ . Rezultă: 1 − z ≥ .
2 n −1
(Egali- tate numai dacă imaginile geometrice ale numerelor ar fi coliniare).
 2kπ 2kπ  kπ  kπ kπ 
2) 1 − z = 1 −  cos + i sin  = 2 i sin  cos + i sin  .
 n n  n  n n 

Rezultă că 1 − z = 2 sin . Folosind punctul 1), obţinem concluzia.
n

R5.2.2 Într-un cerc de rază 1 se înscrie un poligon regulat A1A2…An.


demonstraţi că dacă P este un punct pe cercul circumscris poligonului, atunci :
PA12 + PA22 + ... + PAn2 = 2n .
Soluţie :
Fie Ak(zk), unde z k , k = 1, n sunt soluţiile ecuaţiei z n = 1 . Atunci:

∑ (z − z k ) (z − ) ∑ (z z + z )
n n n n

∑ ∑
2
PA k2 = z − zk = zk = k z k − zk z − zz k =
k =1 k =1 k =1 k =1

n n n
− z∑ zk − z ∑ z k + ∑
2 2
=n z zk .
k =1 k =1 k =1

67
n n
Dar ∑z
k =1
k = 0, din relaţiile lui Viète, deci şi ∑z
k =1
k = 0 . Obţinem :
n


2
PA 2
k = n z + n = 2n .
k =1

R5.2.3 Fie ε o rădăcină primitivă a ecuaţiei x n − 1 = 0 , n ≥ 2 şi z un


număr complex astfel încât z − ε k
≤ 1 , pentru orice k ∈ {0,1,2,..., n − 1} .
Să se arate că z ≤ 1 .
Soluţie :
Evident
1− ε n
1 + ε + ε 2 + ... + ε n−1 =
1− ε
= 0. Atunci (z − 1) + ( z − ε ) + z − ε 2 + ... + ( )
+ (z − ε n −1 ) = nz . Trecând la module, avem :
(
n z = ( z − 1) + ( z − ε ) + z − ε 2 + ... + )
(z − ε ) ≤ z − 1 + z − ε
n −1
+ ... + z − ε n −1 ≤ n . Deci z ≤ 1 .

R5.2.4 Într-un cerc se înscriu două poligoane regulate, unul cu 1982 laturi,
altul cu 2973 laturi. Ştiind că au vârfuri comune, să se afle numărul lor.
Soluţie :
Numărul vârfurilor comune este egal cu numărul soluţiilor comune ale
ecuaţiilor binome: z 1982 = 1 şi z 2973 = 1 . Din 5.1.4 obţinem că acest număr este
d = ( 1982, 2973 ) = 991 .

R5.2.5 Fie A1A2…An un poligon regulat înscris în cercul C de rază egală cu


n
1. Să se determine max ∏ PAk , P ∈ C.
k =1
Soluţie :
Fie Ak(zk) unde z k , k = 1, n sunt soluţiile ecuaţiei z n = 1 . Avem
n
egalitatea: z n − 1 = ∏ ( z − z k ) . Considerând z afixul lui P şi trecând la module
k =1
n n
în relaţia anterioară, avem: ∏ z−z
k =1
k = z n − 1 ⇔ ∏ PAk = z n − 1 . Dar
k =1
n
z n − 1 ≤ z n + 1 = 2. Deci max ∏ PAk = 2 . Pentru zn = -1 are loc egalitatea.
k =1

68
R5.2.6 Un număr par de persoane sunt aşezate în jurul unei mese circulare.
După o pauză, aceleaşi persoane se reaşează la masă ocupând poziţii arbitrare.
Să se arate că există cel puţin două persoane astfel încât numărul persoanelor
aşezate între ele a rămas neschimbat. Rămâne proprietatea adevărată pentru un
număr impar de persoane?
Soluţie :
Presupunem că persoanele ocupă vârfurile unui poligon regulat
A0…A2n-1 cu 2n laturi înscris într-un cerc de centru O, vârfurile fiind notate
astfel încât să parcurgem cercul în sens trigonometric. Alegând axa Ox astfel
încât să treacă prin vârful A0, putem asocia fiecărui vârf Ak numărul complex zk

de argument , 0 ≤ k ≤ 2n . Considerăm că rearanjarea vârfurilor poligonului
n
se face prin rotirea fiecărui vârf Ak cu un unghi ϕ k de măsură
π  π 
lk ⋅  l k ∈ N , l k ⋅ ≤ 2π  , în sens trigonometric. Dacă două unghiuri
n n 
ϕ i , ϕ j sunt egale, concluzia problemei rezultă imediat. În caz contrar rezultă
 π 2π π
că unghiurile ϕ k parcurg toată mulţimea 0, , , ... , (2n − 1)  . Deoarece
 n n n
după rotaţie se obţin vârfurile aceluiaşi poligon regulat, rezultă că suma
unghiurilor de rotaţie este un număr real de forma 2mπ . Deci are loc egalitatea:
2 n −1 2 n −1
π
∑ϕk =
k =0
∑n
j=0
= ( 2 n − 1)π = 2 m π . Deci contradicţie.
j⋅

În cazul n impar proprietatea nu rămâne adevărată. Considerând cazul a


1 2 3 4 5 
5 persoane notate cu numerele 1, 2, 3, 4, 5 şi permutatrea σ =   se
1 3 5 2 4 
observă că pentru (i, j) cu 1 ≤ i ≤ j ≤ 5 avem j − i ≠ σ ( j ) − σ (i ) .

R5.2.7 Fie ecuaţia az 2 + bz + c = 0 , unde a, b, c ∈ C * cu a = b = c . Să se


5 −1 5 +1
arate că ≤ zk ≤ , k = 1, 2 , unde z1, z2 sunt rădăcinile ecuaţiei.
2 2
Soluţie :
Avem: az 2 = − bz − c = bz + c ≤ b ⋅ z + c . Rezultă
 1+ 5   1− 5 
− z − 1 ≤ 0 . Deci  z −   z −  ≤ 0 . Rezultă
2
z  
 2   2 

69
1+ 5
z ≤ .
2
2
Analog c = az 2 + bz ≤ a ⋅ z + b ⋅ z . Obţinem
2 5 −1
z + z − 1 ≥ 0 şi de aici z ≥ .
2

R5.2.8 Fie ecuaţia az 2 + bz + c = 0 , a, b, c ∈ C * . Să se demonstreze


că :
b2 b2
a) Dacă z1 şi z2 sunt rădăcinile ecuaţiei şi z 1 = z 2 , atunci ∈ R şi ≤4
ac ac
b2 b
b) Dacă 0 < ≤ 4 şi α este una dintre rădăcinile ecuaţiei, atunci ≤2 α .
ac a
Soluţie :
a) Fie z1 = r (cos α + i sin α ), z 2 = r (cos β + i sin β ) cu r > 0 şi
α , β ∈ [0, 2π ) . Din relaţiile lui Viète avem :
b
− = z1 + z 2 = r [ (cos α + cos β ) + i (sin α + sin β ) ] =
a
α−β  α+β α+β
= 2 r cos  cos + i sin  (1) şi
2  2 2 
c
= z1 ⋅ z 2 = r 2 ( cos (α + β ) + + i sin(α + β ) ) (2).
a
Din (1) avem
α−β
( cos(α + β ) + i sin(α + β ) ) = 4 c cos2 α − β ,
2
b ( 2)

2
= 4 r 2 cos2 de unde
a 2 a 2
b2 2 α −β b2
= 4 cos ∈ R şi ≤ 4.
ac 2 ac
4ac
− b ±ib −1
b2
b) Avem z1, 2 = . Atunci
2a
b 4ac
z1, 2 = ⋅ − 1 ± ib −1 =
2 a b2
1 b 4ac 1 b 4ac 1 b
= ⋅ 1+ 2 −1 = ≥ .
2 a b 2 a b2 2 a

70
1
R5.2.9 Fie z = + ω + ω 2 + ... + ω n , unde
2
2π 2π
ω = cos + i sin , n ∈ N *.
2n + 1 2n + 1
Să se demonstreze că:
a) Im z 2 k = 0, Re z 2 k +1 = 0 , pentru orice k ∈ N .
b) (2 z + 1) + (2 z − 1)
2 n +1 2 n +1
= 0.
Soluţie :
a) ( )
Evident ω 2 n +1 = 1 . Atunci: 0 = 1 − ω 2 n +1 = (1 − ω ) 1 + ω + ω 2 + ... + ω 2 n .
Rezultă:
(
1 + ω + ω 2 + ... + ω 2 n = 0 ⇔ 1 + ω + ω 2 + ... + ω n ω + .... + ω n = 0 . )
−1
Rezultă: ω + ω 2 + ... + ω n = . Deci:
1+ ω n
2πn 2πn
cos + i sin −1
1 1 ω −1n
1 n + 1 n + 1 i πn
z= − = = = tg .
2 1+ω n
( 2 ω +1 2
n
) cos
2πn
+ i sin
2πn
+1
2 2n + 1
n +1 n +1
i 2k πn
Atunci z 2 k = 2 k ⋅ tg 2 k ∈R, deci Im z 2 k = 0;
2 2 n + 1
i 2 k +1 πn
z 2 k +1 = 2 k +1 ⋅ tg 2 k +1 este pur imaginar, deci Re z 2 k +1 = 0 .
2 2n + 1
b)
2 n +1 2 n +1

( 2 z + 1 ) + ( 2 z − 1 ) =  i tg πn + 1 +  i tg πn − 1 =
2 n +1 2 n +1

 2n + 1   2n + 1 

=
(cos α + i sin α ) − (cos α − i sin α ) = 2 i sin ( 2n + 1 )α = 0, unde α = πn .
2 n +1 2 n +1

cos 2 n +1 α cos 2 n +1 α 2n + 1

R5.2.10 Rezolvaţi în mulţimea numerelor complexe sistemul:


 x ( x − y )( x − z ) = 3

 y ( y − x )( y − z ) = 3 .
 z ( z − x )( z − y ) = 3

Soluţie :
Observăm că numerele x, y, z sunt nenule şi diferite. Scăzând între ele
( )
primele două ecuaţii avem: (x − y ) x 2 + y 2 − xz − yz şi cum x ≠ y, rezultă că

71
x2 + y 2 = xz + yz ( 1 ) . Analog obţinem y 2 + z 2 = yz + zx ( 2 ) şi
z + x = zy + xy ( 3 ). Adunând relaţiile (1), (2) şi (3), obţinem
2 2

x 2 + y 2 + z 2 = xy + yz + zx ( 4 ) . Scăzând din relaţia (4) relaţia (1), obţinem


z 2 = xy ( 5 ) şi analog x 2 = zx ( 6 ) şi y 2 = xz ( 7 ) . Din relaţiile (6) şi (7)
avem (x − y )( x + y ) = z ( y − x ) , adică (x − y )( x + y + z ) = 0 . Cum x ≠ y ,
obţinem x+y+z = 0. Înlocuind în (1) obţinem x3=1 şi analog y3=1, z3=1.
Obţinem mulţimea soluţiilor
{( )( )( )( )( )( )}
S = 1, ε , ε , 1, ε , ε , ε ,1, ε , ε , ε ,1 , ε , ε ,1 , ε ,1, ε unde ε = 1, ε ≠ 1 .
2 2 2 2 2 2 3

72
6. Metoda vectorială în rezolvarea problemelor de algebră

O cale elegantă de rezolvare a unor probleme de algebră, mai ales a unor


inegalităţi, este metoda vectorială, care foloseşte în principal produsul scalar a
doi vectori şi proprietăţile acestuia.

6.1. Produsul scalar a doi vetori.


Definiţie Produsul scalar a doi vectori a , b este numărul notat
a ⋅ b = a ⋅ b ⋅ cos ϕ , unde ϕ este unghiul vectorilor a , b .
π
1) dacă ϕ ∈ [0,  , atunci produsul scalar al vectorilor a , b este un
2
număr pozitiv;
π
2 ) dacă ϕ = , atunci produsul scalar este nul;
2
π
3) dacă ϕ ∈  , π ) , atunci produsul scalar este număr negativ.
2

6.1.1. Proprietăţi ale produsului scalar.


P1. Produsul scalar este comutativ: a · b = b · a
P2. Doi vectori nenuli sunt perpendiculari dacă şi numai dacă produsul
lor scalar este nul a ≠ 0 , b ≠ 0 , a ⋅ b = 0 dacă şi numai dacă a ⊥ b
P3. Produsul scalar a doi vectori de acelaşi sens este egal cu produsul
modulelor lor
P4. Produsul scalar a doi vectori este egal cu mărimea unuia dintre
vectori înmulţită cu proiecţia celuilalt pe el.
P5. Produsul scalar este distributiv faţă de adunarea vectorilor
( )
a ⋅ b + c = a ⋅b + a ⋅c
(a + b)⋅ c = a ⋅ c + b ⋅ c
6.1.2. Produsul scalar în plan
Fie a = x1 i + y1 j , b = x 2 i + y 2 j , doi vectori in plan.
Atunci a ⋅ b = x1 x 2 + y1 y 2 .
Putem calcula unghiul a doi vectori a = ( x1 , y1 ) , b = ( x 2 , y 2 ) . Avem
a ⋅b x 1 x 2 + y1 y 2
cos ϕ = = .
a⋅b x12 + y12 ⋅ x 22 + y 22

73
De aici deducem că vectorii a şi b sunt perpendiculari dacă şi numai dacă
x1 x 2 + y1 y 2 = 0 .

6.1.3. Produsul scalar în spaţiu


Fie a = ( x1 , y1 , z1 ) , b = ( x 2 , y 2 , z 2 ) doi vectori în spaţiu.
Atunci 1) a ⋅ b = x1 x 2 + y1 y 2 + z1 z 2
2) a = x12 + y12 + z12
x1 x 2 + y1 y 2 + z1 z 2
3) cos ϕ =
x + y12 + z12 ⋅ x 22 + y 22 + z 22
2
1

6.2. Aplicaţii ale produsului scalar Consideraţii teoretice

Considerăm în reperul cartezian XOY , vectorii


a = a1 i + a 2 j = (a1 , a 2 ) , b = b1 i + b2 j
Vom nota în continuare produsul scalar a · b =a1b1 + a2b2,
a a 
sub forma unui tablou:  1 2  = a1b1 + a 2 b2 .
b1 b2 

6.2.1.Definiţie Cuplurile (a1,a2) , (b1,b2) au aceeaşi monotonie dacă


a1 ≤ a 2 , b1 ≤ b2 sau a 2 ≤ a1 , b2 ≤ b1 2 , ai , bi > 0 , i = 1,2 deci cel mai mare
dintre numerele a1, a2 se află în tablou deasupra celui mai mare dintre b1 şi b2 .

6.2.2. Propoziţie Fie (a1,a2), (b1,b2) cupluri cu aceeaşi monotonie


a a  a a 
Atunci  1 2  ≥  1 2 
b1 b2  b1 b2 
Demonstraţie
a a  a a 
Avem  1 2  -  1 2  = a1b1 + a 2 b2 − a1b2 − a 2 b1 = (a1 − a 2 )(b1 − b2 ) ≥ 0
b1 b2  b2 b2 

74
6.2.3. Definiţie Tripletele (a1 , a 2 , a3 ) , (b1 , b2 , b3 ) au aceeaşi monotonie
dacă a1 ≤ a 2 ≤ a3 , b1 ≤ b2 ≤ b3 sau a1 ≥ a 2 ≥ a3 , b1 ≥ b2 ≥ b3 , a1 , bi > 0 ,
i = 1,3 .

6.2.4. Propoziţie Fie (a1 , a 2 , a3 ) , (b1 , b2 , b3 ) două triplete de numere


( )
reale pozitive, având aceeaşi monotonie, iar b1' , b2' , b3' o permutare a numerelor
a a a3  a1 a 2 a3 
b1 , b2 , b3 . Atunci  1 2  ≥  ' ' ' .
b1 b2 b3  b 1 b 2 b3 
Demonstraţie
a1 a 2 a3 
Trebuie să arătăm că din cele 6 numere de forma  ' ' '  , cel mai mare
b1 b2 b3 
a a a3 
număr este  1 2  ( )
.Dacă b1' , b2' , b3' ≠ (b1 , b2 , b3 ) , atunci (∃) k, l, 1 ≤ k ≤ l ≤ 3
b1 b2 b3 
astfel încât (a k , al ) , (bk , bl ) să nu aibă aceeaşi monotonie. Din proprietate
a k al 
6.2.2. rezultă că schimbând locurile numerelor bk' , bl' , numărul  ' '  se mărşte
bk bl 
a1 a 2 a3 
şi deci  ' ' '  se măreşte. Continuând raţionamentul, ajungem la concluzia
b1 b2 b3 

a1 a 2 a3 
b b b  este cel mai mare număr.
 1 2 3

6.2.5. Observaţie (Generalizare)


Numim vector n-dimensional sau vector cu n
componente, a = (a1, a2 ,...., an ) ∈ R+ n

n ≥ 2.
Considerând vectorii n-dimensionali
a = (a1 , a 2 ,....a n ) , b = (b1 , b2 ,....bn ) ,…, z = ( z1 , z 2 ,....z n ) ,definim numărul

75
a1 a 2 .... a n 
b b .... b 
 1 2 n 
= a1b1 ....z1 + a 2 b2 ....z 2 + .... + a n bn ....z n .
.... ........ .... 
 
 z1 z 2 .... z n 
Dacă (a1 , a 2 ,...., a n ), (b1 , b2 ,...., bn ),...., ( z1 , z 2 ,...., z n ) au aceeaşi monotonie,
atunci
a1 a 2 .... a n  a1 a 2 .... a n 
' ' '

b b .... b   ' ' '



 1 2 n ≥ b 1 b2 .... b n .
.... ........ ....  .... ........ .....
   ' 
 z1 z 2 .... z n   z1 z 2 .... z n 
' '

6.3. Alte inegalităţi deduse folosind produsul scalar

R6.4.1. Dacă a>0, b>0, c>0, să se arate că


( )
 1 1 1
a 3 + b 3 + c 3  + +  ≥ (a + b + c )
2

a b c
Soluţie
(
Considerăm vectorii v1 = a a , b b , c c şi v 2 =  , )
 1 1 1 
, 
 a b c
1 1 1
Avem v1 = a 3 + b 3 + c 3 , v 2 = + + , v1 ⋅ v 2 = a + b + c
a b c
Inegalitatea v1 ⋅ v 2 ≤ v1 ⋅ v 2 este echivalentă cu inegalitatea cerută.

R6.4.2. Dacă a,b ∈ [− 1,1] să se arate că


2
a +b
1− a2 + 1− b2 ≤ 2 1−  
 2 
Soluţie Considerăm vectorii v1 = (1,1) , v 2 = ( 1− a , 2
)
1 − b 2 . Relaţia
v1 ⋅ v 2 ≤ v1 ⋅ v 2 se scrie 1 − a 2 + 1 − b 2 ≤ 2 2 − a 2 − b 2 (1)

76
2
a +b
Trebuie să arătăm că (
2 2−a −b 2 2
) ≤ 2 1−   (2), inegalitate
 2 
echivalentă cu 4 − 2a 2 − 2b 2 ≤ 4 − (a + b ) dacă şi numai dacă (a − b ) ≥ 0 .
2 2

Din (1) şi (2) obţinem relaţia din enunţ.


Observaţii
0
1 Inegalitatea (1) este mai “tare” decât inegalitatea cerută.
2 0 Avem egalitate pentru a=b= λ , λ ∈ R şi λ ≤ 1 .
30 Inegalitatea admite şi o generalizare:
Dacă xi ≤ 1, xi ∈ R, i = 1, n , atunci
2
n
 n 

i =1
1 − x ≤ n −  ∑ xi 
2
i
 i =1 
2

R6.4.3. Să se demonstreze că
n
4 + n 9 + n 16 n +1 6 + n +1 8 + n +1 12
>
n
6 + n 8 + n 12 n +1 4 + n +1 9 + n +1 16

Soluţie Numerele 4=2 2 , 9=3 2 , 16=4 2 , 6=3 ⋅ 2 , 8= 2 ⋅ 4 şi 12= 3 ⋅ 4 , ne


sugerează considerarea următorilor vectori:
(
v1 = n 2 , n 3 , n 4 , )
v2 = n 3, n 4 , n 2 , (
v3 = n +1 2 , n +1 3 , n +1 4 , ) ( )
v4 =( n +1
)
3 , n +1 4 , n +1 2 .
Inegalitatea v1 ⋅ v 2 ≤ v1 ⋅ v 2 se scrie n
2 ⋅n 3 + n 3⋅n 4 + n 4 ⋅n 2 ≤

≤ ( 2 ) + ( 3) + ( 4 ) ⋅ ( 3) + ( 4 ) + ( 2 )
n
2
n
2
n
2
n
2
n
2
n
2
,
sau n
6 + 12 + 8 ≤ 4 + 9 + 16 .
n n n n n

n
4 + n 9 + n 16
Deoarece nu putem avea egalitate, deducem > 1 (1)
n
6 + n 12 + n 8
Folosind un raţionament similar pentru vectorii v3 şi v 4 obţinem
n +1
6 + n +1 8 + n +1 12
< 1 (2). Inegalităţile (1) şi (2) rezolvă complet problema.
n +1
4 + n +1 9 + n +1 16

R6.4.4. Să se arate că pentru numerele pozitive fixate a,b,c,x,y,z , are


loc inegalitatea

77
ax + by + cz + (a 2
)( 2
)
(a + b + c )(x + y + z )
+ b2 + c2 x2 + y2 + z 2 ≥
3
(V. Dubrovski, V.Cârtoaje, Kvanr nr 4/1990)

Soluţie Expresia ax + by + cz reprezintă produsul scalar dintre vectorii


u = (a, b, c ) şi v = ( x, y, z ) , iar a 2 + b 2 + c 2 , x 2 + y 2 + z 2 reprezintă
lungimile acestor vectori. Să vedem ce semnificaţie are membrul drept al
inegalităţii cerute. Dacă notăm w = (1,1,1) atunci a + b + c = u ⋅ w , iar
x + y + z = v ⋅ w . Cu aceste notaţii inegalitatea de demonstrat devine:
2
u ⋅ v + u ⋅ v ≥ u ⋅ w ⋅ v ⋅ w. (∗ )
3

Dacă u = (0,0,0 ) sau v = (0,0,0 ) , inegalitatea ( ∗ ) este evident satisfăcută,


aşadar putem presupune u > 0, v > 0 . Cum w = 3 , împărţim ( ∗ ) cu
u ⋅v u ⋅v v⋅w
+1 ≥ 2 ⋅ , sau cos α + 1 ≥ 2 cos β cos χ , unde α , β , χ sunt
u⋅v u⋅v v⋅w
unghiurile dintre u şi v , u şi w respectiv v şi w
(FIG 6.1).

Cerinţa s-a redus la o problemă de geometrie în spaţiu. Fiind dat un


triedru cu unghiurile plane de la vârf egale cu α , β , χ să fie demonstreze că are
loc inegalitatea : cos α +1 ≥ 2 cos β cos χ .
Este cunoscut că α ≤ β + χ , de unde cos α ≥ cos (β + χ ) , prin urmare avem:
2cos β cos χ = cos (β + χ ) +cos (β − χ ) ≤ cos α +1, ceea ce trebuia demonstrat.

78
Să vedem când avem egalitate. Din raţionamentul făcut anterior, observăm
că pentru a obţine egalitate trebuie ca β − χ = 0 şi α = β + χ , deci β = χ şi
α = 2 β (FIG…).
Acestea conduc la faptul că vectorii u, v, w sunt coplanari şi
( ) ( )
< u , v =< v, w . Să observăm că

Fig. 6.2
w
proiecţia lui v pe dreapta suport a lui w este vectorul v' = λ , unde
w
v⋅w x+ y+z x+ y+z
λ = v cos β = v ⋅ = , deci v' = ⋅ w (FIG…)
v⋅w 3 3

Prin urmare coordonatele vectorului v" simetricul lui v faţă de w vor fi

Fig. 6.3
Prin urmare coordonatele vectorului v " simetricul lui v faţă de w vor fi
 2( y + z ) − x 2( x + z ) − y 2( x + y ) − z 
v" =  , ,  . Cum u şi v" sunt vectori
 3 3 3 

79
a b c
coliniari, rezultă = = , care este condiţia ca
2( y + z ) − x 2( x + z ) − y 2( x + y ) − z
inegalitatea să devină egalitate.

6.4. Sisteme de ecuaţii

R6.5.1. Să se rezolve sistemul:


 2 2
9 y z + 4 x 2 z 2 + 25 x 2 y 2 = 16 x 2 y 2 z 2
 2
x + 4 y + z = 9
2 2

 15
 x − y 3 + z 15 =
 2

9 4 25
Soluţie Prima ecuaţie este echivalentă cu 2
+ 2 + 2 = 16 . Această
x y z
 3 2 5
scriere ne sugerează considerarea următorilor vectori: a =  , ,  ,
x y z
3 2 5
b = ( x, 2 y , z ) . Avem a ⋅b = ⋅ x + ⋅ 2 y + ⋅ z = 12 , iar
x y z
9 4 25
a⋅b = 2
+ 2 + 2 ⋅ x 2 + 4 y 2 + z 2 , deci a ⋅ b = a ⋅ b . Rezultă că unghiul
x y z
vectorilor este nul, vectorii sunt deci coliniari şi prin urmare coordonatele lor
3 1 5
sunt proporţionale. Obţinem 2 = 2 = 2 , de unde
x y z
3 4 5 12 9 3 15
2
= 2
= 2 = , deci x 2 = , y 2 = , z 2 = . Ţinând cont de ecuaţia
x 4y z 9 4 4 4
 3 3 15   3 3 15 
a treia, avem soluţiile  , , ;  − ,−
  , .
2 2 2  2 2 2 

R6.5.2. Să se arate că sistemul:


 x + y 4 + z 4 = 1
4

 2 nu are soluţii reale.


 x + y 2 + z 2 = 7

80
Soluţii Considerăm vectorii a = (x 2 , y 2 , z 2 ) , b = (1,1,2 ) . Inegalitatea
(a ⋅ b) ≤ a 2
⋅b
2
este echivalentă cu
(x ) ( 2
)( )
+ y 2 + 2 z 2 ≤ x 4 + y 4 + z 4 12 + 12 + 2 2 , adică 7 ≤ 6 , absurd. Aşadar
2

sistemul nu are soluţii reale.


R6.5.3. Să se rezolve sistemul de ecuaţii:
 x 2 + y 2 + y(x + z ) = 0
 2
 x + x + y + 2 yz = 0
 2
3 x + 8 y + 8 xy + 8 y = 2 x + 4 z + 2
2 2

(Matematica V Şcole 5/1984)


Soluţie Scrierea sistemului sub forma echivalentă
 x( x + y ) + y ( y + z ) = 0

 x( x + 1) + y (2 z + 1) = 0 (*)

4( x + y ) + 4( y + z ) = ( x + 1) + (2 z + 1)
2 2 2 2

ne sugerează considerarea următorilor vectori a = ( x, y ) , b = ( x + y, y + z ) ,


c = ( x + 1,2 z + 1) . Din (*) deducem a ⋅ b = 0 , a ⋅ c = 0 , 4b 2 = c 2 . Dacă a = 0 ,
1
rezultă x = y = 0 şi z = − . Dacă a ≠ 0 , avem c = ±2b şi vectorii b şi c sunt
2
coliniari. Din ( x + 1,2 z + 1) = (2 x + 2 y,2 y + 2 z ) rezultă x+2y=1 şi 2y=1, deci
 1 1
soluţia  0, ,−  . Din (x + 1,2 z + 1) = −(2 x + 2 y,2 y + 2 z ) rezultă 3x+2y=1 şi
 2 2
1− 2y −1− 2y
4 z + 2 y = −1 , de unde x = , y= care înlocuite în prima ecuaţie
3 4
1
ne conduce la 10 y 2 − 13 y + 4 = 0 cu soluţia convenabilă y = . Soluţiile
2
 1  1 1
ecuaţiei sunt  0,0,−  şi  0, ,−  .
 2  2 2

Bibliografie

• M. Ganga, Manual pentru clasa a X-a M1, Ed. Mathpress, Ploieşti, 2002
• V.Tudor, Probleme de algebră cu rezolvări ingenioase, Ed. Carminis,
Piteşti, 1999 pag 64-84

81
Probleme rezolvate (6)

R6.3.1. Fie a, b > 0 . Să se arate că a 3 + b 3 ≥ a 2 b + b 2 a .


Soluţie
Cuplurile (a,b), (a2,b2) au aceeaşi monotonie, deci
a b  a b 
 2 2 ≥  2 2
dacă şi numai dacă a 3 + b 3 ≥ a 2 b + b 2 a
a b  b a 

R6.3.2. Dacă a, b, c > 0 , atunci are loc inegalitatea


a +b +c ≥ a b+b c+c a.
3 3 3 2 2 2

Soluţie Tripletele (a,b,c), (a2,b2,c2) au aceeaşi monotonie, deci


a b c  a b c 
 2 2 2
≥  2 2 2  dacă şi numai dacă a 3 + b 3 + c 3 ≥ a 2 b + b 2 c + c 2 a .
a b c  c a b 

b b c 3
R6.3.3. Dacă a,b,c>0, atunci + + ≥ .
b+c c+a a+b 2
 1 1 1 
Solutie Considerăm tripletele (a,b,c);  , ,  cu aceeaşi
b+c c+a a +b
 1 1 1   1 1 1 
monotonie şi tripletele  , , ,  , ,  sau
c+a a+b b+c a+b b+c c+a
a b c  a b c 
 1 1 
1 ≥  1 1 1  dacă şi numai dacă

b + c c + a a + b  c + a a + b b + c 
a b c a b c
+ + ≥ + + şi
b+c c+a a+b c+a a+b b+c
a b c  a b c 
 1 1  ≥
1   1  1 1  dacă şi numai dacă

b + c c + a a + b   a + b b + c c + a 
a b c a b c
+ + ≥ + +
b+c c+a a+b a+b c+b a+c
Adunând cele două inegalităţi, obţinem inegalitatea cerută.

82
a1 + a 2 + .... + a n n
R6.3.4. Dacă ai ≥ 0 , i= 1, n , atunci ≥ a1 ⋅ a 2 ⋅ .... ⋅ a n
n
(inegalitatea dintre media aritmetică şi cea geometrică)
Soluţie Considerăm vectorul n-dimensional x = ( x1 , x 2 ,...., x n ) , atunci

 x1 x 2 .... x n   x1 x 2 .... x n 
 x x .... x   x x .... x 
n 1 
 1 2 ≥ 2 3 dacă şi numai dacă
.... ........ ....  .... ........ .... 
   
 x1 x 2 x n   x n x1 .... x n −1 
x1n + x 2n + ...... x nn ≥ n ⋅ x1 ⋅ x 2 ⋅ ..... ⋅ x n
Luând ai = xin , i = 1, n , obţinem inegalitatea cerută.

R6.3.5. (Inegalitatea lui Cebâşev)


Să se arate că dacă 0 ≤ a1 ≤ a 2 ≤ .... ≤ a n , 0 ≤ b1 ≤ b2 ≤ .... ≤ bn ,
1
Atunci a1b1 + a 2 b2 + .... + a n bn ≥ (a1 + a 2 + .... + a n )(b1 + b2 + .... + bn )
n
Soluţie
Considerăm vectorii n-dimensionali de aceeaşi monotonie
a =(a1,a2,….,an), b =(b1,b2,….,bn). Adunând inegalităţile
a1 a 2 .... a n  a1 a 2 .... a n 
b b .... b  ≥ b b .... b 
 1 2 n   1 2 n 

a1 a 2 .... a n  a1 a 2 .... a n 


b b .... b  ≥ b b .... b 
 1 2 n   2 3 1 

…………………………………….
a1 a 2 .... a n  a1 a 2 .... a n 
b b .... b  ≥ b b .... b  , obţinem
 1 2 n   n 1 n −1 

n(a1b1+a2b2+….+anbn) ≥ (a1+a2+….+an)(b1+b2+….+bn).

R6.3.6. Fie a1,a2,….,an>0 şi S=a1+a2+….+an. Să se arate că


a1 a2 an n
+ + .... + ≥
S − a1 S − a 2 S − an n − 1

83
Soluţie
Considerăm n-uplurile cu aceeaşi monotonie (a1 , a 2 ,...., a n ) ,
 1 1 1 
 , ,....,  , atunci avem inegalităţile
 S − a1 S − a 2 S − a n 
a1 a2 .... a n  a1 a2 .... a n 
 1 1  ≥  1 1 
 .... 1   1 ....
 S − a1 S − a 2 S − a n   S − a 2 S − a3 S − a1 
a1 a2 .... a n  a1 a2 .... a n 
 1 1 
1 ≥  1  1 .... 1 
 ....
 S − a1 S − a 2 S − a n   S − a3 S − a 4 S − a 2 
……………………………………………………………………..
a1 a2 .... a n  a1 a2 .... a n 
 1 1 
1  ≥  1  1 1 
 .... ....
 S − a1 S − a 2 S − a n   S − a n S − a1 S − a n −1 
Prin adunarea inegalităţilor obţinem.
 
(n − 1) a1 + a 2 + .... + a n  ≥
 S − a1 S − a 2 S − an 
a 2 + a 3 + .... + a n a1 + a3 + .... + a n a + a 2 + .... + a n −1
+ + .... + 1 =n
S − a1 S − a2 S − an

R6.3.7. Să se arate că dacă a,b,c,d>0, atunci a3+b3+c3+d3 ≥ a2b+b2c+c2d+d2a.


Soluţie
Considerăm triplete (a 2 , b 2 , c 2 , d 2 ) , (a, b, c, d ) cu aceeaşi monotonie,
a b c d  a b c d 
atunci  2 2 2 2  ≥  2 2 2
a b c d  d a b c 
2

a2 b2 c2 a+b+c
R6.3.8. Fie a,b,c>0. Demonstraţi că + + ≥
b+c a+c a+b 2

(C. 1952, G.M. 7-8/1998)


 1 1 1 
Soluţie Tripletele (a 2 , b 2 , c 2 ) şi  , ,  au aceeaşi
b+c c + a a +b
monotonie.

84
Aplicând rezultatul din propoziţia 6.2.4. avem inegalităţile
a 2 b 2 c2  a 2 b2 c2 
   
 1 1 1  ≥  1 1 1 
 b + c c + a a + b   c + a a + b b + c 
a 2 b2 c2  a 2 b2 c2 
   
 1 1 1  ≥  1 1 1 
 b + c c + a a + b   a + b b + c c + a 
Adunând membru cu membru aceste inegalităţi , obţinem
 a2 b2 c2  a2 + b2 b2 + c2 c2 + a2
2 + +  ≥ + + ≥
b+c c+a a +b a+b b+c c+a
1
≥ (a + b ) + 1 (b + c ) + 1 (c + a ) = a + b + c
2 2 2
Comentariu Prezentăm în continuare o altă soluţie a problemei, bazată pe
inegalităţi cunoscute. Presupunem a ≥ b ≥ c , deci a 2 ≥ b 2 ≥ c 2 şi
1 1 1
≥ ≥ .
b+c c+a a+b
a2 1
Din inegalitatea Cebâşev avem 3 ∑ ≥ ∑ a2 ⋅ ∑ (1)
b+c b+c
Din inegalitatea Cauchy-Buniakowsky-Schwarz, rezultă
3 ∑ a ≥ (a + b + c ) (2)
2 2

Din inegalitatea dintre media aritmetică şi cea geometrică, avem


1 9 1
∑ a + b ≥ 2 ⋅ a + b + c (3)
Din (1),(2) şi (3) rezultă inegalitatea cerută.

85
7. Ecuaţii exponenţiale şi logaritmice nestandard

De cele mai multe ori, problemele propuse la concursurile de


matematică nu se încadrează într-un anumit tipar. Rezolvarea lor presupune din
partea competitorilor, pe lângă o bună stăpânire a aparatului matematic, şi
abilitate deosebită care să permită “spargerea” problemei.
Noţiunile “problemă standard”, “problemă nestandard” sunt relative.
Orice problemă a cărei rezolvare nu este cunoscută, poate reprezenta, la un
moment dat, o problemă nestandard.
Vom numi problemă nestandard o problemă a cărei rezolvare nu se
bazează pe un algoritm cunoscut. Nu există metode generale de rezolvare a
acestor probleme. Vom încerca să indicăm câteva direcţii de abordare a
acestora. Tehnicile utilizate apelează la: studiul monotoniei, studiul convexităţii
unor funcţii, inegalităţi clasice, etc.

7.1 Utilizarea monotoniei unor funcţii

7.1.1 Teoremă : Dacă funcţia f este strict monotonă pe intervalul I, iar c


este o constantă reală, atunci ecuaţia f ( x) = c are pe intervalul I cel mult o
soluţie.
Demonstraţie : Fie f funcţie strict crescătoare. Presupunem că ecuaţia
f ( x) = c are cel puţin două soluţii diferite x1 , x 2 pe intervalul I. Fie x1 < x 2 .
Din f strict crescătoare rezultă f ( x1 ) < f ( x 2 ) . Contradicţie cu
f ( x1 ) = f ( x 2 ) = c .

7.1.2 Teoremă : Dacă funcţiile f şi g sunt monotone pe intervalul I, de


monotonii diferite, cel puţin una dintre ele fiind strict monotonă, atunci ecuaţia
f ( x) = g ( x) are cel mult o soluţie pe intervalul I.
Demonstraţie : Fie f strict crescătoare, iar g descrescătoare pe intervalul I.
Presupunem că există cel puţin două soluţii diferite x1 , x 2 , din intervalul I, ale
ecuaţiei f ( x) = g ( x) . Fie x1 < x 2 . Atunci f ( x1 ) = g ( x1 ) ≥ g ( x 2 ) = f ( x 2 ) .
Contradicţie cu f funcţie strict crescătoare pe intervalul I.

Amintim câteva rezultate cunoscute din teoria funcţiilor:


7.1.3 Propoziţie : Fie f , g : A ⊂ R → R .
a) Dacă f şi g sunt funcţii strict crescătoare (descrescătoare) pe A, atunci
f + g este o funcţie strict crescătoare (descrescătoare) pe A.

86
b) Dacă f , g : A → (0, ∞ ) sunt strict crescătoare (descrescătoare), atunci
f ⋅ g este o funcţie strict crescătoare (descrescătoare).

7.1.4 Propoziţie : Fie f : A → B, g : B → C .


a) Dacă f, g sunt strict crescătoare, atunci g o f este strict crescătoare.
b) Dacă f, g sunt strict descrescătoare, atunci g o f este strict crescătoare.
c) Dacă f, g sunt strict monotone, dar de monotonii diferite, atunci g o f
este strict descrescătoare.

7.2. Utilizarea inegalităţilor şi rezolvarea anumitor ecuaţii exponenţiale şi


logaritmice

Vom pune în evidenţă alte câteva direcţii de abordare a ecuaţiilor


exponenţiale şi logaritmice nestandard, direcţii ce se bazează pe inegalităţi:
metoda constantei separatoare, metoda utilizării inegalităţilor clasice, inegalităţi
deduse din studiul convexităţii anumitor funcţii, etc.
I. Metoda constantei separatoare sau metoda minimaximului, se
bazează în principal pe evaluarea ambilor membri ai ecuaţiei.
Fie dată ecuaţia f ( x) = g ( x) , x ∈ I ⊂ R (1). Să admitem că se
cunoaşte că f ( x) ≤ A, iar g ( x) ≥ A , pentru orice x ∈ I . Este evident că ecuaţia
 f ( x) = A
(1) are soluţii dacă şi numai dacă sistemul de ecuaţii  , x ∈ I este
 g ( x) = A
compatibil. Evident partea dificilă o reprezintă determinarea constantei A. Nu
sunt reguli generale. În principiu se utilizează proprietăţile funcţiilor f şi g.
Următorul exemplu este ilustrativ în acest sens:

x2 + x
R7.2.1 Să se rezolve ecuaţia: 2 cos 2 = 2 x + 2−x .
6
x +x
2
Soluţie: Avem inegalităţile: 2 cos 2 ≤ 2 şi 2 x + 2 − x ≥ 2 . Deci egalitate
6
 2 x
2
+ x
 cos =1
avem dacă  6 . Din a doua ecuaţie se obţine x = 0, soluţie
2 x + 2 − x = 2

unică, care verifică şi prima ecuaţie. Ecuaţia dată are soluţia unică x = 0.

II. Utilizarea inegalităţilor clasice : Se folosesc inegalităţile: mediilor,


Cauchy-Buniakovski-Schwarz, Bernoulli, etc., dar în mod special interesează

87
situaţia în care avem egalitate în aceste inegalităţi. Amintim, fără a le
demonstra, aceste inegalităţi.

7.2.2. Inegalitatea mediilor : Dacă a1 , a 2 ,..., a n sunt n numere reale strict


pozitive, (n ∈ N , n ≥ 2 ) , atunci:
H n (a1 , a 2 ,..., a n ) ≤ Gn (a1 , a 2 ,..., a n ) ≤ An (a1 , a 2 ,..., a n ) , cu egalitate
n
dacă a1 = a 2 = ... = a n , unde: H n (a1 , a 2 ,..., a n ) = ,
1 1 1
+ + ... +
a1 a 2 an
G n (a 1 , a 2 ,..., a n ) = n a 1 a 2 ... a n ,
a1 + a 2 + ... + a n
An (a1 , a 2 ,..., a n ) = .
n

7.2.3. Inegalitatea Cauchy-Buniakovski-Schwarz: Dacă a i , bi ∈ R , i = 1, n ,


2
 n   n  n 
atunci:  ∑ a i b i  ≤  ∑ a i2   ∑ b i2  , cu egalitate dacă ai b j = a j bi ,
 i =1   i =1   i=2 
i ≠ j , i, j = 1, n .

7.2.4. Inegalitatea lui Bernoulli : Dacă a ∈ R , a > − 1, n ∈ N , atunci:


(1 + a )n ≥ 1 + na .
Ilustrăm această metodă prin următoarea problemă:

R7.2.5 Să se rezolve ecuaţia: 2log 3x + 3log 4x + ...+ nlog n+1x + (n +1)log 2x = n


Soluţie :
logbc
Folosind identitatea a = clogba şi inegalitatea dintre media aritmetică
şi geometrică, avem:

nx = x log 32 + x log 43 + ... + x log n+1n + x log 2( n+1) ≥ n x log 32+log 43+...+logn+1 n+log 2( n+1) ≥
n

n
n n log 3 2 ⋅ log 4 3 ⋅ ... ⋅ log n +1n ⋅ log 2 ( n +1 )
≥n x = n n x n = nx (1).
Am folosit faptul că:
lg2 lg3 lgn lg(n +1)
log3 2 ⋅log43⋅...⋅ logn+1 n⋅log2(n +1) = ⋅ ⋅ ...⋅ ⋅ =1
lg3 lg4 lg(n +1) lg2
.

88
Egalitatea în (1) are loc pentru x = 1.

III. Utilizarea convexităţii :

7.2.2. Definiţie : O funcţie f : I ⊆ R → R este strict convexă pe intervalul I


dacă ∀ x1 , x 2 ∈ I , ∀ λ ∈ [0,1] are loc inegalitatea:
f (λx1 + (1 − λ )x 2 ) < λf ( x1 ) + (1 − λ ) f ( x 2 ) (2).
Dacă inegalitatea este de sens contrar, funcţia se numeşte strict concavă.
Amintim că funcţia exponenţială f a : R → (0, ∞ ) , f a ( x) = a x , unde
a > 0, a ≠ 1 este strict convexă, iar funcţia logaritmică g a : (0, ∞ ) → R,
g a ( x) = log a x este strict concavă pentru a > 1 şi strict convexă pentru
0 < a < 1. Folosind inegalitatea (2) putem arăta că anumite ecuaţii nu mai au şi
alte soluţii.

R7.2.6 Problemă rezolvată: Să se rezolve ecuaţia:


5 x + 7 x + 11x = 6 x + 8 x + 10 x , x ∈ N .
Soluţie :
Evident x = 0 şi x = 1 sunt soluţii. Arătăm că ecuaţia nu are alte soluţii.
Funcţia f : [0, ∞ ) → R, f (t ) = t n , unde n natural, n ≥ 2 este strict convexă.
n
an + bn  a + b 
Atunci, din (2), obţinem: >  , pentru a, b > 0, a ≠ b .
2  2 
x
5x + 7 x  5 + 7 
Deci : >  =6 , x≥2
x

2  2 
x
7 x + 11x  7 + 11 
>  =9 , x≥2
x

2  2 
x
11x + 5 x  11 + 5 
>  = 8 , x ≥ 2.
x

2  2 
Adunând aceste relaţii, obţinem: 5 x + 7 x + 11x > 6 x + 8 x + 9 x , pentru
x ≥ 2 , deci ecuaţia nu poate avea alte soluţii.

Un aspect legat de folosirea convexităţii în rezolvarea acestor tipuri de


probleme este cuprins în următoarea propoziţie:
7.2.3. Propoziţie : Dacă funcţia f : I → R este strict convexă pe intervalul I,
iar g : I → R este o funcţie liniară, atunci ecuaţia f ( x) = g ( x) are cel mult
două soluţii pe intervalul I .

89
Demonstraţie : Admitem că există cel puţin trei soluţii diferite x1 , x 2 , x3 . Fie
x3 ∈ ( x1 , x 2 ) . Atunci există λ ∈ (0 ,1) astfel încât: x 3 = λ x1 + (1 − λ ) x 2 . Dar
f ( x 3 ) = g ( x 3 ) = λg (x1 ) + (1 − λ )g ( x 2 ) = λf (x1 ) + (1 − λ ) f (x 2 ) . Contradicţie cu f
strict convexă.
Evident, concluzia propoziţiei se păstrează dacă f este strict concavă.

R7.2.8 Să se rezolve ecuaţia: 10 3 (1999 x + 2) = 1x + 2 x + ... + 2000 x .


Soluţie :
Se verifică uşor că x = 0 şi x = 1 sunt soluţii. Cum funcţia f : R → R,
f ( x ) = 10 3 (1999 x + 2 ) este liniară, iar funcţia g : R → R,
g ( x) = 1 + 2 + ... + 2000 este strict convexă (sumă de funcţii strict convexe
x x x

este o funcţie strict convexă, după cum se poate verifica, folosind 7.2.5),
deducem că ecuaţia f ( x) = g ( x) nu poate să aibă alte soluţii.

Bibliografie

1. Andrei Gh., Cucurezeanu I., Caragea C., Bordea Gh., probleme de algebră
pentru concursuri de admitere şi olimpiade şcolare, clasa a X-a, E.D.P.,
Bucureşti, 1999
2. Becheanu M. şi colaboratori, Olimpiade de matematice 1990-1996, clasa
IX-X, Editura Gil, Zalău, 1997
3. Berinde V., Explorare, investigare şi descoperire în matematică, Efemeride,
2001
4. Ganga M., Ecuaţii şi inecuaţii, Editura Mathpress, Ploieşti, 1998
5. Gorgotă V., Şerdean I., Ulmeanu S., Matematica în concursurile şcolare,
2002, IX-XII, Editura Paralela 45, 2002
6. Nanu I., Tutescu L., Ecuaţii nestandard, aditura Apoloo, Craiova, 1994
7. Suceveanu V., Copaceanu R., Metode nestandard de rezolvare a ecuaţiilor,
Foaie matematică, Nr. 3 şi Nr. 4, 1999, Chişinău
8. Tămâian T., Probleme selectate din reviste selecte, editura Cub Press, Baia
Mare, 2002

90
Probleme rezolvate (7.1)

R7.2.1 Să se rezolve ecuaţia: 10 x + 11x + 12 x = 13 x + 14 x .


Soluţie :
Împărţim ambii membri cu 13 x . Ecuaţia devine:
x x x x
 10   11   12   14 
  +   +   = 1+   .
 13   13   13   13 
x x x
 10   11   12 
Funcţia f : R → R , f ( x) =   +   +   este strict descrescătoare;
 13   13   13 
x
 14 
funcţia g : R → R , g ( x) = 1 +   este strict crescătoare. Atunci ecuaţia
 13 
f ( x) = g ( x) are cel mult o soluţie. Cum x = 2 verifică ecuaţia, deducem că
aceasta este unică.

(
R7.2.2 Rezolvaţi în R+* ecuaţia: log 2 1 + x = log 3 x . )
Soluţie :
(
Notăm log 3 x = y . Ecuaţia devine: log 2 1 + 3 y = y sau 1 + 3 y = 2 y ,)
y y
 1   3   1   3 
y y

de unde   +  = 1 . Funcţia f : R → R , f ( y ) =   +  este


 2   2   2   2 
strict descrescătoare şi f (2) = 1 . Deci y = 2 este unica soluţie. Obţinem că
x = 9 este unica soluţie a ecuaţiei date.

R7.2.3 Să se rezolve ecuaţia: log 6 x + x ( log34


)
+ xlog35 = log3 x .
log x log x log x
Soluţie : Pentru x > 0 , ecuaţia se transcrie: 3 3 + 4 3 + 5 3 = 6 3 .
log x

Pentru ecuaţia 3z + 4 z + 5z = 6 z , împărţim cu 6 z şi obţinem:


z z z z z z
1 2 5 1 2 5
+
      + = 1 . Funcţia f : R → R , f ( z ) = + +
      este
2 3 6 2 3 6
strict descrescătoare pe R şi f (3) = 1 . Deci z = 3 este unica soluţie. Obţinem că
x = 9 este unica soluţie a ecuaţiei date.

R7.2.4 Să se rezolve ecuaţia: (a + b ) − a x − b x = 2 (ab ) , unde a > 1, b > 1 .


x x

2
 x x

Soluţie: Ecuaţia se mai scrie: (a + b)
x
=  a 2
+b 2 
 sau echivalent:
 
2
 x 2
  2x x
 x x x x
 (a + b ) 2  = 
 a + b 2 
 . Obţinem (a + b ) 2 = a 2
+ b 2
. Împărţind cu (a + b ) 2,
   

91
x x
x x
 a 2  b 2
rezultă:   +  = 1 . Funcţia f : R → R , f ( x) =  a   b 2
2
+ 
a+b a+b a+b a+b
este strict descrescătoare (sumă de funcţii strict descrescătoare) şi f (2) = 1 .
Deci x = 2 este unica soluţie.

R7.2.5 Să se rezolve în (0, ∞ ) ecuaţia: x = a


x x+a2
,
unde a ∈ N , a > 1 .
Soluţie :
Logaritmând în baza a, obţinem: x log a x = x + a 2 sau echivalent
a2
log a x = 1 + . Funcţia f : (0, ∞ ) → R , f ( x) = log a x este strict crescătoare,
x
a2
iar funcţia g : (0, ∞ ) → R, g ( x) = 1 + este strict descrescătoare. Din T 7.1.2
x
deducem că ecuaţia are cel mult o soluţie. Dar x = a 2 este soluţie. Deci x = a 2
este unica soluţie.

R7.2.6 Să se rezolve: x n ( x − 1) + lg x = 0, n ∈ N * .
Soluţie :
xn+1
n+1
Ecuaţia se transcrie: x − x + lg n = 0 . Adică x n+1 + lg x n+1 = x n + lg x n
n

x
(1). Funcţia f : (0, ∞ ) → R , f ( x) = x + lg x este strict crescătoare, deci
injectivă. Ecuaţia (1) devine f (x n +1 ) = f (x n ) . Rezultă x n +1 = x n . Cum x > 0 ,
obţinem x = 1.

R7.2.7 Să se rezolve ecuaţia: (x + a)log b − (x + b)log a = b − a ,


a b

unde a > 1, b > 1 .


Soluţie :
Cum a log b c = c log b a , ecuaţia devine: b
log a( x+a )
− alog b( x+b) = b − a
log ( x+a ) log ( x+b )
sau b a + x +a = x +b+ a b sau
b log a ( x + a ) + a log a ( x + a ) = b log b ( x + b ) + a log b ( x + b ) . Funcţia f :R → R ,
f ( x ) = b x + a x este injectivă. Obţinem: log a ( x + a ) = log b ( x + b ) . Fie
t = log a ( x + a) . Atunci: b t − a t = b − a . Dacă a = b , atunci orice x
∈ (− a, ∞ ) este soluţie.
t t
a b−a a b−a
Dacă a < b : 1 =   + t . Cum funcţia g : R → R , g (t ) =   + t
b b b b
este strict descrescătoare, obţinem t = 1, adică x = 0 este soluţie unică. Dacă
a > b , analog.

92
1 1
x+ x+
R7.2.8 Rezolvaţi ecuaţia: 4 x
+9 x
= 275 .
Soluţie :
1 1
x+ x+
Pentru x < 0, 4 x
+9 x
< 2 , deci ecuaţia nu are soluţii.
1
x+
Fie x > 0 . Considerăm funcţia f a : (0, ∞ ) → R, f a ( x) = a
, a > 1 . Atunci x

1
fa = g o h , unde h : (0, ∞ ) → [2, ∞ ), h( x) = x + , iar
x
g : [2, ∞ ) → R, g ( x) = a x . Se constată că h este strict descrescătoare pe (0,1]
şi strict crescătoare pe [1, ∞ ) , iar g este strict crescătoare. Atunci, din propoziţia
7.1.4, deducem că funcţia f a este strict descrescătoare pe (0,1] şi strict
crescătoare pe [1, ∞ ) . Funcţia
1 1
x+ x+
F : (0, ∞ ) → R, F ( x) = 4 + 9 = f 4 ( x) + f 9 ( x) este strict descrescătoare
x x

pe (0,1] şi strict crescătoare pe [1, ∞ ) , din P 7.1.3. În concluzie, ecuaţia:


F(x)=275 are cel mult câte o soluţie pe intervalele (0,1] , respectiv [1, ∞ ) . Dar
1 1
x = ∈ (0,1] şi x = 2 ∈ [1, ∞ ) sunt soluţii. Deci x = şi x = 2 sunt singurele
2 2
soluţii ale ecuaţiei date.

 3 x − 2 y = 23
R7.2.9 Rezolvaţi sistemul:  .
 log 3 x + log 2 y = 2
Soluţie :
(
Din prima ecuaţie: x = log 3 2 y + 23 , iar din a doua: x = 3 )
2−log 2 y
.
(
Obţinem ecuaţia: log 3 2 + 23 = 3
y
)
2 − log 2 y
. Cum funcţiile din cei doi membri ai
ecuaţiei sunt de monotonii diferite, iar y = 2 verifică ecuaţia, obţinem că y = 2
x = 3
este soluţie unică. Soluţia sistemului este:  .
y = 2

3 x + 4 y = 5 x

R7.2.10 Rezolvaţi în R sistemul: 3 y + 4 z = 5 y .
 z
3 + 4 = 5
x z

Soluţie :
Adunăm ecuaţiile sistemului şi rezultă:
( ) ( ) (
3x + 4 x − 5x + 3 y + 4 y − 5 y + 3z + 4 z − 5z = 0 )
(*) . Ştim, folosind
monotonia, că ecuaţia 3 + 4 = 5 are singura soluţie x = 2. Deci x = y = z = 2
x x x

este soluţie a sistemului. Vom arăta că este soluţie unică. Admitem că ar exista

93
 5  x 
o soluţie cu x > 2 . Din prima ecuaţie rezultă că 4 = 3   − 1 . Dar funcţia
y x

 3  
 5  x

f : [0, ∞ ) → R, f ( x) = 3 x   − 1 fiind strict crescătoare (ca produs de
 3  
funcţii pozitive strict crescătoare, P 7.1.3), deducem că x > 2 ⇒ f ( x) > 16 ,
adică 4 y > 16, deci y > 2 . Analog obţinem z > 2 . Dar pentru
x > 2, y > 2, z > 2 , avem 3 + 4 −5 < 0,
x x x
3 + 4y − 5y < 0,
y

3 + 4 − 5 < 0 . Contradicţie cu (*). Asemănător se tratează şi cazurile


z z z

x ∈ (0,2 ), respectiv x ≤ 0 .

Probleme rezolvate (7.2)

( )
R7.4.1 Rezolvaţi ecuaţia: log 3 8 + 2 x − x 2 = 2 x −1 + 21− x .
Soluţie :
Avem: 8 + 2 x − x 2 ≤ 9 , pentru orice x real. Atunci log 3 8 + 2 x − x 2 ≤ 2 , ( )
pentru orice x ∈ (− 2,4 ) . Dar 2 x −1
+2 1− x
≥2 2 x −1
⋅2 1− x
= 2 . Ecuaţia dată este
(
log 8 + 2 x − x
echivalentă cu sistemul:  x −13 1− x
2
)= 2 . Se verifică că x = 1 este unica
2 + 2 = 2
soluţie.

R7.4.2 Să se rezolve în R ecuaţia: log 2 2 x 2 + 2 = ( ) x2 +1


x2 + 2
.
Soluţie :
Se verifică că x = 0 este
soluţie a ecuaţiei. Cum
( 1
)
2 x 2 + 2 ≥ 2 , ∀ x ∈ R , avem log 2 2 x 2 + 2 ≥ , ∀ x ∈ R . Vom arăta că
2
x +1 1
2
≤ , ∀ x ∈ R . Avem echivalent 2 x2 +1 ≤ x2 + 2 sau
x +2
2
2
4 x 2 + 4 ≤ x 4 + 4 x 2 + 4 , echivalent cu 0 ≤ x 4 , ∀ x ∈ R . Am arătat că
x2 +1 1
x +2
2
( )
≤ ≤ log 2 2 x 2 + 2 , ∀ x ∈ R . Atunci ecuaţia dată este echivalentă
2
 x2 +1 1
 2 =
cu sistemul:  x + 2 2 .
(
log 2 x 2 + 2 = 1
 2 2
)
94
Obţinem x = 0, unica soluţie.
log x
R7.4.3 Să se rezolve ecuaţia: 2 3 + 3 x = 4 .
log 2

Soluţie :
Notăm log3 x = t, x > 0, x ≠ 1. Atunci:

( )
log 3 2 1 1 1
3 log x 2
=3 log 3 x
= 3 log 3 2 log x
3 = 2 t . Ecuaţia devine: 2 t + 2 t = 4 . Aplicând
1 1
t+
inegalitatea mediilor, avem: 4 = 2 + 2 ≥ ≥ 2 2 t t t
≥ 4 , cu egalitate dacă t =
1. Obţinem x = 3, soluţia ecuaţiei date.

R7.4.4 Să se rezolve ecuaţia: 16 x + 81x + 625 x = 60 x + 90 x + 150 x .


Soluţie :
Notăm 2 x = a, 3 x = b, 5 x = c . Ecuaţia devine:
a + b + c = a bc + ab c + abc .
4 4 4 2 2 2
Dar
a 4 + b 4 + c 4 ≥ a 2 b 2 + b 2 c 2 + c 2 a 2 ≥ a 2 bc + ab 2 c + abc 2 , cu egalitate pentru
a = b = c . Obţinem x = 0 singura soluţie a ecuaţiei date.
R7.4.5 Să se rezolve ecuaţia:
( −x −x
)( −x
x + 1 + 2 + ... + n 1 + 2 + ... + n = n , unde n ∈ N , n ≥ 2 .
2n x x x 2 *
)
Soluţie :
Aplicând inegalitatea mediilor avem:
n 1+ 2x +...+ nx
≤ sau echivalent:
1 1 n
1+ x +...+ x
2 n
( )( )
1x + 2 x + ... + n x 1− x + 2− x + ... + n− x ≥ n2 . Cum x 2 n ≥ 0, ∀ x ∈ R , egalitate evem
doar pentru x = 0.
R7.4.6 Să se rezolve ecuaţia:
a + b + b + c + c + a = 2a + 2b + 2c .
x x x x x x x x x

Soluţie :
Aplicând inegalitatea lui Cauchy-Buniakovski-Schwarz obţinem:
a +bx + bx + cx + cx + ax ≤ 3 ⋅ 2 ax +bx + cx .
x
( ) Deci:
2a x + 2b x + 2c x ≤ ≤ 2 ⋅ 3(a x + b x + c x ) . Ridicând la pătrat obţinem:
ax +bx +cx ≥ axbx + bxcx + cxax , cu egalitate numai dacă a x = b x = c x . Dacă
a = b = c , atunci orice x real este soluţie. Dacă a ≠ b sau b ≠ c sau c ≠ a ,
atunci x = 0, unica soluţie.

R7.4.7 Să se rezolve ecuaţia: 3 2 x +1 − x ⋅ 3 x +1 − 3 x − 6 x 2 − 7 x − 2 = 0 .

95
Soluţie :
Ecuaţia se scrie în formă echivalentă:
( ) ( )
3 ⋅ 3 − (3x + 1) ⋅ 3x − 6x 2 + 7x + 2 = 0 . Obţinem:
x 2

3 x + 1 ± 81x 2 + 90 x + 25 2
3x = , adică: 3 x = 2 x + 1 sau 3 x = − x − . Ecuaţia
6 3
3 x = 2 x + 1 are soluţiile x1 = 0 şi x 2 = 1 şi nu mai are alte soluţii, deoarece
graficul unei funcţii strict convexe şi o dreaptă au cel mult două puncte distincte
2
comune (propoziţia 7.3.7). Ecuaţia 3 x = − x − are soluţia x = −1 , unică,
3
deoarece membrul stâng este o funcţie strict crescătoare, iar membrul stâng o
funcţie strict descrescătoare.
În concluzie, ecuaţia dată are soluţiile: x1 = 0 , x 2 = 1 şi x3 = −1 .

R7.4.8 a) Să se demonstreze că dacă f : R → R+ este o funcţie convexă,


atunci funcţia g : R → R+ , g ( x) = f n ( x) este convexă pentru orice n ∈ N * .
n n
 4 
x 
2
b) Să se rezolve în R ecuaţia:  2 x +  +  4 x +  = 3 n 3 n + 2 n ,
x
( )

unde n ∈ N este fixat.
*

Soluţie :
a) Deoarece f este convexă, atunci f (tx1 + (1 − t )x 2 ) ≤ tf (x1 ) + (1 − t ) f x 2 , ( )
oricare ar fi x 1 , x 2 ∈ R , t ∈ [0 ,1 ] . Atunci f 2 (tx 1 + (1 − t )x 2 ) ≤
( )
≤ t 2 f 2 ( x1 ) + 2t (1 − t ) f x 1 f ( x 2 ) + (1 − t ) f 2 ( x 2 ) , deoarece f este pozitivă. Deci
2

f 2 este convexă.
Demonstrăm prin inducţie după n natural, n ≥ 2 că f n este convexă.
Presupunând că pentru k ∈ N , k ≥ 2 , f k este convexă, avem pentru
x1 , x 2 ∈ R şi t ∈ [0 ,1 ] că:
f k +1 (tx 1 + (1 − t )x 2 ) = f k (tx 1 + (1 − t )x 2 )⋅ f (tx 1 + (1 − t )x 2 ) ≤
[ ]
≤ tf k (x1 ) + (1 − t) f k (x2 ) ⋅ f (tx1 + (1 − t)x2 ) ≤
[ ]
tf k (x1 ) + (1− t) f k (x2 ) ⋅ [tf (x1 ) + (1− t ) f (x2 )] =
( )
= t 2 f k +1 ( x1 ) + t (1 − t ) f ( x1 ) f ( x 2 ) f k −1 ( x1 ) + f k −1 ( x 2 ) + (1 − t ) f ( x 2 ) ⋅ f k ( x 2 ) ≤
2

≤ tf k +1 ( x1 ) + (1 − t ) f k +1 ( x 2 ) , ultima inegalitate fiind echivalentă cu :


( )[ ( )]
t 2 − t f k +1 ( x1 ) + f ( x1 ) f ( x 2 ) f k −1 ( x1 ) + f k − 2 (x 2 ) + f k +1 ( x 2 ) ≤ 0 , care este
evident adevărată.
Am demonstrat astfel că f k +1 este convexă şi folosind metoda inducţiei
matematice, rezultă că f n este convexă pentru orice n natural, n ≥ 2 .

96
1
b) Se observă că dacă x este soluţie, atunci este soluţie, pentru n ∈ N * .
x
Dacă n este par, dacă x este soluţie şi – x este soluţie, iar dacă n este impar,
ecuaţia poate avea numai soluţii pozitive. Este suficient să determinăm soluţiile
strict pozitive. Considerăm funcţiile f1 , f 2 , f 3 , f 4 : R + → R + ,
4 2
f1 ( x ) = 2 x, f 2 ( x) = , f 3 ( x) = 4 x, f 4 ( x) = , care sunt funcţii
x x
convexe pe (0, ∞ ) . Atunci f1 + f 2 şi f 3 + f 4 sunt funcţii convexe. Din punctul
a) obţinem că ( f1 + f 2 ) şi ( f 3 + f 4 ) sunt funcţii convexe, deci
n n

f = ( f1 + f 2 ) + ( f 3 + f 4 ) este convexă. Cum pentru o funcţie convexă


n n

neconstantă f, ecuaţia f ( x) = k , unde k este o constantă reală, are cel mult


1
două soluţii, iar x1 = 2 şi x 2 = sunt soluţiile ecuaţiei date, putem
2
1
concluziona: dacă n este par, ecuaţia are soluţiile ± 2 şi ± , iar dacă n este
2
1
impar, ecuaţia are soluţiile 2 şi .
2
R7.4.9 Să se rezolve ecuaţia:
(n − 1)n (n + 2) (n − 1)
a 2 x + a 3 x + ... + a nx + = ax , unde
2 2
a > 0 , a ≠ 1, n ∈ N * , n ≥ 2 .
Soluţie :
Pentru orice x real avem: a 2 x + 1 ≥ 2 a x , a 3 x + 2 ≥ 3 a x ,…,
a + n − 1 ≥ na x conform inegalităţii mediilor. Prin adunare se obţine:
nx

n(n − 1) (n − 1) (n + 2)
a 2 x + a 3 x + ... + a nx + ≥ ax , cu egalitate dacă şi numai
2 2
dacă a 2 x = a 3 x = ... = a nx = 1 , adică x = 0.

1 1  π
R7.4.10 Să se rezolve ecuaţia: 2x
+ 2x
= x , a ∈  0,  .
sin a cos a  2
Soluţie :
Se aduce ecuaţia la forma: (1 + tg a ) + (1 + ctg a ) = x
2 x 2 x
(1).
f (x) = (1 + tg a ) + (1 + ctg a ) − x
x x
Dacă x < 0 , funcţia 2 2
este strict
crescătoare, iar ecuaţia f ( x) = 0 are cel mult o soluţie. Se verifică că x = −1
este soluţie. Dacă x ∈ (0,1) , membrul stâng al ecuaţiei (1) este supraunitar, iar
cel drept este subunitar. Deci ecuaţia nu are soluţii pentru x ∈ (0,1) . Dacă x ≥ 1 ,
atunci x ≥ [x ] şi conform inegalităţii lui Bernoulli avem:

97
(1 + tg a ) + (1 + ctg a ) ≥ (1 + tg a )[ ] + (1 + ctg a )[ ] > 2 + [x ](tg
2 x 2 x 2 x 2 x 2
a + ctg 2 a ≥)
≥ 2 + 2 [x ] > 2 x > x . Deci singura soluţie este x = −1 .

R7.4.11 Rezolvaţi în mulţimea [0, ∞ ) ecuaţia:


3 x + 4 x + 5 x + 10 x + 14 x + 21x = 2 x +1 + 3 x ⋅ 2 x +1 + 2 ⋅ 35 x .
Soluţie :
Ecuaţia se scrie: 3x + 4 x + 5x + 2 x ⋅ 5x + 2 x ⋅ 7 x + 3x ⋅ 7 x = 2(2 x + 3x ⋅ 2 x + 5x ⋅ 7 x )
sau 3x + 2 x ⋅ 2 x + 5 x + 2 x ⋅ 5 x + 2 x ⋅ 7 x + 3x ⋅ 7 x + 2 x + 2 x ⋅ 3x + 5 x ⋅ 7 x = 3(2 x + 2 x ⋅ 3x + 5 x ⋅ 7 x )
( )( ) (
sau 2 x + 3 x + 5 x 1 + 2 x + 7 x = 3 1⋅ 2 x + 2 x ⋅ 3 x + 5 x ⋅ 7 x . Deoarece x ≥ 0, )
avem 2 x ≤ 3 x ≤ 5 x şi 1 ≤ 2 x ≤ 7 x . Aplicând inegalitatea lui Cebâşev, obţinem:
( )( ) (
2 x + 3 x + 5 x 1 + 2 x + 7 x ≤ 3 2 x + 2 x ⋅ 3x + 5 x ⋅ 7 x , ) cu egalitate pentru
2 = 3 = 5 şi 1 = 2 = 7 , deci x = 0.
x x x x x

98
8. Probleme de numărare

8.1. Consideraţii teoretice şi interpretări ale formulelor uzuale din


combinatorică

Există o mare varietate de probleme care se pot încadra în această temă.


Pentru rezolvarea acestora, este necesar să reţinem următoarele:
I. Dacă A şi B sunt două mulţimi finite şi notăm
B A = { f f : A → B func þie} , atunci
A
BA = B ,
(am notat X numărul elementelor mulţimii X).
Acest rezultat se demonstrează prin inducţie după m = A . Dăm în
continuare unele interpretări utile ale acestui rezultat.
1) Numărul submulţimilor unei mulţimi M având n elemente, n ∈ ,
M
este P ( M ) = 2 . Aceasta rezultă din faptul că numărul cerut este egal cu
numărul funcţiilor f : M → {0,1} .
2) În câte moduri poate fi pavată o alee de lungime n (n ∈ * ) şi lăţime
1 cu plăci pătrate de 1× 1 , folosindu-se plăci de n culori. Numărul cerut este
m n , deoarece fiecărei poziţii de placă din cele n trebuie să-i atribuim o culoare
din cele m.
3) Câte cuvinte de m litere pot fi făcute cu un alfabet ce conţine n
simboluri? Numărul cerut este m n şi este egal cu numărul funcţiilor f : A → B ,
A = n − numărul de simboluri şi B = m − numărul literelor dintr-un cuvânt.
4) Câte numere naturale de n cifre se pot forma folosind k cifre fixate,
k ∈ {1, 2,K ,10} ? Dacă nici o cifră din cele k nu este 0, putem forma k n
numere deoarece orice cifră din număr poate fi aleasă în k moduri. Dacă printre
cele k cifre se află şi cifra 0, prima cifră a numărului poate fi aleasă în ( k − 1 )
moduri şi orice altă cifră a numărului poate fi aleasă în k moduri. Aşadar,
numărul de numere în acest caz este (k − 1) ⋅ k n −1 .
5) Câte numere naturale au n cifre în scrierea lor în baza k (k ≥ 2) ?
Folosind 4) deducem că numărul cerut este (k − 1) ⋅ k n −1 .
6) În câte moduri pot fi împărţite n obiecte la m persoane? A face o
astfel de împărţire revine la a stabili un destinatar pentru fiecare obiect, deci a
defini o funcţie de la mulţimea obiectelor la mulţimea persoanelor privite ca şi
destinatari. Obţinem că numărul cerut este m n .

99
II. Numărul submulţimilor ordonate cu k elemente ale unei mulţimi cu n
elemente (k , n ∈ , k ≤ n) este
not. n!
Ank = = n(n − 1)(n − 2)L (n − k + 1) ,
(n − k )!
şi se citeşte aranjamente de n luate câte k, unde p ! = 1⋅ 2 ⋅ 3 ⋅K ⋅ p .
Punem în evidenţă unele interpretări ale numerelor Ank .
1) Dacă A şi B sunt mulţimi finite cu A = k ≤ B = n , atunci numărul
funcţiilor injective f : A → B este egal cu Ank . Într-adevăr, pentru a defini o
funcţie f : A → B avem nevoie de valorile f (a1 ), f (a2 ),K , f (ak ) care vor
forma o submulţime ordonată cu k elemente a lui B. Deci numărul funcţiilor
injective de la A la B este egal cu numărul submulţimilor ordonate cu k
elemente ale lui B, în total Ank .
2) Numărul cuvintelor formate cu k litere distincte, folosind un alfabet
cu n simboluri este tot Ank , k ≤ n .
3) Numărul modurilor de pavare a unei alei 1× k cu plăci alese de culori
diferite din n culori date este Ank , k ≤ n .
4) Dacă A este o mulţime finită şi nevidă, atunci numărul funcţiilor
injective (surjective, bijective) f : A → A este Ann = n ! . Facem observaţia că o
funcţie bijectivă f : A → A , A finită, se mai numeşte şi permutare a mulţimii A.
Numărul permutărilor unei mulţimi cu n elemente este egal cu n ! .
III. Dacă A este o mulţime cu n elemente, n ∈ * , atunci numărul
submulţimilor lui A având fiecare k elemente (k fixat, k ≤ n ) este:
Ak n! n(n − 1) ⋅K ⋅ (n − k + 1)
Cnk = n = = ,
k ! k !(n − k )! k!
şi se citeşte combinări de n elemente luate câte k. Redăm în continuare câteva
aplicaţii semnificative.
1) Care este numărul funcţiilor f :{1, 2,K , n} → {0,1} şi având
n
proprietatea ∑ f (i) = k ? A defini o astfel de funcţie presupune a reţine exact k
i =1
elemente din domeniul de definiţie şi a asocia fiecăruia valoarea 1. Aşadar,
numărul cerut este egal cu numărul de submulţimi cu k elemente ale domeniului
de definiţie, adică cu Cnk . Pentru k < 0 sau k > n , sau k ∈ [0, n] \ , nu avem
astfel de funcţii.
2) Numărul drumurilor laticeale de lungime minimă care unesc punctul
O(0, 0) cu punctul B(m, n) (m, n ∈ ) este Cmm+ n . Într-adevăr, lungimea minimă

100
a unui astfel de drum este m + n , singurele deplasări fiind de forma
( p, q) a ( p + 1, q) sau ( p, q) a ( p, q + 1) . Din cei n + m paşi de lungime unu
avem de făcut m paşi orizontali şi n paşi verticali, ordinea efectuării lor fiind
arbitrară. Numărul drumurilor laticeale cerut este egal cu numărul de paşi pe
orizontală, ceea ce se poate face în Cmm+ n moduri.
3) Numărul funcţiilor strict crescătoare f :{1, 2,K , k} → {1, 2,K , n} ,
k ≤ n , este egal cu Cnk . Aceasta rezultă din faptul că fiecare funcţie strict
crescătoare este determinată de k ! funcţii injective prin ordonarea crescătoare a
valorilor sale. Aşadar, numărul cerut este
Num ã rul func þiilor injective Ank
N= = = Cnk .
k! k!
4) Numărul funcţiilor crescătoare f :{1, 2,K , k} → {0,1, 2,K , n − k} este
k
Cn . Motivăm în continuare acest lucru.
Fie f o funcţie care îndeplineşte condiţiile din ipoteză. Atunci, funcţia
g :{1, 2,K , k} → {1, 2,K , n} , g (i ) = i + f (i ) , (∀) i = 1, k este strict crescătoare.
Într-adevăr, g este corect definită şi dacă presupunem 1 ≤ i1 < i2 ≤ k , atunci
f (i1 ) ≤ f (i2 ) , deci i1 + f (i1 ) < i2 + f (i2 ) , adică g (i1 ) < g (i2 ) .
Reciproc, dacă g :{1, 2,K , k} → {1, 2,K , n} , g (i ) = i + f (i ) , (∀) i = 1, k ,
este o funcţie strict crescătoare atunci i + f (i ) ∈ {1, 2,K , n} pentru (∀) i = 1, k ,
de unde f (i ) ∈ {0,1,K , n − k} pentru orice i = 1, k şi
1 < 2 < K < k ⇒ g (1) < g (2) < K < g (k ) ⇒
⇒ 1 + f (1) < 2 + f (2) < 3 + f (3) < K < k + f (k ) ,
de unde utilizând faptul că f (i ) ∈ , i = 1, k , obţinem
f (1) ≤ f (2) ≤ K ≤ f (k ) .
Aşadar, funcţia f este crescătoare.
Ca urmare, mulţimea funcţiilor f cerute este în bijecţie cu mulţimea
funcţiilor g. A defini o funcţie g revine la a alege un şir 1 ≤ b1 < b2 < K < bk ≤ n
({b1 , b2 ,K , bk } ⊂ {1, 2,K , n}) şi acesta se poate face în Cnk moduri (vezi şi
problema precedentă). Ca urmare, numărul cerut este Cnk .
Mai facem observaţia că numărul funcţiilor crescătoare f : A → B ,
A = n , B = m este Cmn + n −1 .
5) Numărul modurilor de descompunere a numărului natural n în sumă
de k numere naturale nenule, a1 + a2 + K + an , în care contează ordinea

101
numerelor a1 , a2 ,K , an este Cnk−−11 . Pentru a motiva aceasta, este suficient să ne
imaginăm că intervalul [0, n] de lungime n, trebuie să-l partiţionăm în k
subintervale. Aceasta se poate face alegând cele k − 1 capete dintre numerele
1, 2,K , n − 1 , alegere ce se poate face în Cnk−−11 moduri.
6) Numărul modurilor de pavare a unei alei de lungime n şi lăţime 1 cu
plăci 1× 1 dintre care k albe şi n − k negre este Cnk . Într-adevăr, numărul cerut
este egal cu numărul de alegeri a k poziţii din cele n poziţii în care să punem
plăci albe şi acesta este Cnk .
IV. Dacă n = p1α1 ⋅ p2α 2 ⋅K ⋅ pαk k reprezintă descompunerea în factori primi
a numărului natural n ( p1 , p2 ,K , pk sunt numere prime iar α1 ,α 2 ,K ,α k ∈ *
),
atunci numărul divizorilor naturali ai lui n este:
(α1 + 1)(α 2 + 1) ⋅K ⋅ (α k + 1) .

Bibliografie

1. Dorin Andrica, Eugen Jecan, Teste de matematică, Editura GIL, Zalău


2. Dan Brânzei, Vasile Gorgota, Sorin Ulmeanu, Concursuri interjudeţene de
matematică, Editura Paralela 45, 1999
3. Ovidiu Cojocaru, Matematică, Concursul interjudeţean “Spiru Haret – Gh.
Vrânceanu” 1985-1986, Editura Paralela 45
4. Mircea Ganga, Probleme elementare de matematică, Editura Mathpress,
2003
5. Adrian Ghioca, Acad. Nicolae Teodorescu, Culegere de probleme,
Bucureşti, 1987
6. Laurenţiu Panaitopol, Dinu Şerbănescu, Probleme de teoria numerelor şi
combinatorică pentru juniori, Editura GIL, 2003
7. Acad. Nicolae Teodorescu şi alţii, Culegere de probleme, S.S.M.R., vol.I,
Bucureşti
8. Ion Tomescu, Introducere în combinatorică, Editura Tehnică, 1972
9. * * *, Colecţiile revistelor G.M. şi R.M.T

102
Probleme rezolvate (8)

R8.2.1. a) Să se determine numărul de moduri în care 2n persoane pot fi


împărţite în n grupuri de câte 2 persoane.
b) Să se arate că ( (m ⋅ n)!) se divide cu (n !) m +1 ⋅ (m !) n +1 , pentru orice
2

n, m ∈ *
.
2
Soluţie. a) Prima pereche poate fi aleasă în C2n moduri, a doua în C22n − 2
moduri etc. Cum ordinea alegerii perechilor nu contează, numărul căutat va fi:
1 2 1 (2n)! (2n − 2)! 4! 2! (2n)!
C2 n ⋅ C22n − 2 ⋅K ⋅ C42 ⋅ C22 = ⋅ ⋅ ⋅K ⋅ ⋅ = n .
n! n ! 2!(2n − 2)! 2!(2n − 4)! 2!⋅ 2! 2!⋅ 0! 2 ⋅ n !
b) Generalizând punctul a) obţinem că numărul de moduri în care m ⋅ n
persoane pot fi împărţite în n grupe de câte m persoane este
(m ⋅ n)! (m ⋅ n)!
, deci ∈
(m !) ⋅ n !
n
(m !) n ⋅ n !
(1)
Analog se obţine
(m ⋅ n)!

(n !) m ⋅ m !
(2)
Din (1) şi (2) prin înmulţirea celor două numere naturale obţinem:
( (m ⋅ n)!) ∈
2

.
( n !) ⋅ ( m !)
m +1 n +1

R8.2.2. Avem la dispoziţie 2n persoane (n ∈ * ) care trebuie repartizate în


două cluburi, fiecare club constând din câte n membri. În fiecare club este ales
un preşedinte şi un vicepreşedinte. În câte moduri se poate face aceasta.
2
Soluţie. Există C2n posibilităţi de alegere a repartizării pe cluburi.
Pentru fiecare alegere există An2 posibilităţi de alegere a unui preşedinte şi a
unui vicepreşedinte pentru primul club, iar aceştia, odată aleşi, există An2
posibilităţi de alegere a unui preşedinte şi a unui vicepreşedinte pentru al doilea
(2n)!
club. În total există C2nn ⋅ An2 ⋅ An2 = posibilităţi de grupare.
((n − 2)!) 2
R8.2.3. Fie A o mulţime cu n elemente şi B o mulţime cu m elemente,
n, m ∈ * , n ≤ m . Să se arate că numărul funcţiilor nemonotone definite pe A şi
cu valori în B este m n − 2Cmn + n −1 + m .

103
Soluţie. Numărul total al funcţiilor f : A → B este m n .
Numărul funcţiilor strict crescătoare este egal cu Cmn , iar al celor
descrescătoare coincide cu numărul combinărilor cu repetiţie al unei mulţimi cu
(m + n − 1)!
m elemente luate câte n, adică cu Cmn + n −1 = . Observăm că numărul
n !(m − 1)!
funcţiilor crescătoare este egal cu numărul celor descrescătoare şi de asemenea
că există m funcţii constante care au fost numărate de două ori (o dată printre
cele crescătoare şi apoi printre cele descrescătoare. Ca urmare, numărul
funcţiilor monotone este 2Cmn + n −1 − m , în consecinţă, numărul cerut este
m n − 2Cmn + n −1 + m .
R8.2.4. Să se determine numărul de numere cu 7 cifre care nu încep şi nu se
termină cu cifra 1.
Soluţie. Fie mulţimile A = {1, 2,3, 4,5, 6, 7} , B = {0,1, 2,3, 4,5, 6, 7,8,9} .
Problema este echivalentă cu a determina numărul aplicaţiilor f : A → B cu
proprietăţile:
f (1) ≠ 0, f (1) ≠ 1, f (7) ≠ 1
(1)
Pentru f (1) există 8 posibilităţi de alegere, iar pentru f (7) există 9 posibilităţi
de alegere. Fie i ∈ {2,3, 4,5, 6} . Pentru f (i ) există 10 posibilităţi de alegere.
Rezultă că numărul funcţiilor f cu proprietatea (1) este 8 ⋅ 9 ⋅105 = 72 ⋅105 .
R8.2.5. Fie n > 0 un număr natural. Să se determine numărul polinoamelor
P( X ) cu coeficienţi 0, 1, 2 sau 3 cu proprietatea P (2) = n .
Soluţie. Fie P ( X ) = a0 + a1 X + K + ak X k un polinom cu coeficienţi în
a 
mulţimea {0,1, 2,3} astfel încât P (2) = n . Atunci, punând bi =  i  rezultă că
2
n
numărul m = b0 + b1 ⋅ 2 + K + bk ⋅ 2k satisface inegalităţile 0 ≤ m ≤ .
2
Mai mult, egalitatea de mai sus dă reprezentarea binară a lui m.
Aplicaţia P ( X ) a m defineşte o funcţie de la mulţimea de polinoame dată la
n
mulţimea numerelor întregi şi nenegative mai mici sau egale cu . Vom arăta
2
că această funcţie este bijectivă.
Pentru a proba injectivitatea, fie
m = a0 + a1 ⋅ 2 + K + ak ⋅ 2k = a0′ + a1′ ⋅ 2 + K + ak′ ⋅ 2k ,

104
 a   a′ 
astfel încât  i  =  i  , pentru orice i = 0,1,K , k .
2 2
 a   a′ 
Atunci 2 (a0 − a0′ ) şi  0  =  0  implică a0 = a0′ . În continuare, se reduce a0,
2 2
se împarte cu 2 şi se procedează prin inducţie.
Surjectivitatea rezultă astfel:
Din m = b0 + b1 ⋅ 2 + K + bk ⋅ 2k , bi ∈ {0,1} , se definesc numerele ai astfel
a 
încât ai ∈ {0,1, 2,3},  i  = bi şi 2i +1 n − a0 − a1 ⋅ 2 − K − ai ⋅ 2i . În concluzie,
2
n
numărul polinoamelor date este   + 1 .
2
R8.2.6. Câte numere naturale există printre numerele
1⋅ m 2 ⋅ m p⋅m
, ,K , , p, m, n ∈ , n ≠ 0 .
n n n
Soluţie. Fie (m, n) = d . Rezultă că m = m1 ⋅ d , n = n1 ⋅ d cu (m1 , n1 ) = 1 .
Obţinem numerele:
1⋅ m1 m m
, 2 ⋅ 1 ,K , p ⋅ 1 cu (m1 , n1 ) = 1
n1 n1 n1
(1)
 p
Între numerele de la (1) există   numere naturale căci (m1 , n1 ) = 1 . Cum
 n1 
n  (m, n) ⋅ p 
n1 = , rezultă că între numerele din enunţ există   numere naturale.
d n
R8.2.7. Fie X o submulţime cu k elemente a mulţimii A = {a1 , a2 ,K , an } ,
k ∈ * . Să se determine numărul funcţiile f : A → A cu proprietatea
f (X ) = X .
Soluţie. Fie X = {ai1 , ai2 ,K , aik } . Condiţia f ( X ) = X arată că restricţia
lui f la X este bijectivă. Numărul bijecţiilor de la X la X este k! . O astfel de
bijecţie o putem prelungi în n n − k moduri la o aplicaţie f : A → A deoarece
fiecărui element din A \ X îi putem ataşa oricare element din codomeniul A.
Rezultă că numărul căutat este k !⋅ n n − k .
R8.2.8. Fie k, n numere naturale fixate 1 ≤ k < n şi fie S o mulţime de n
puncte din plan având proprietăţile următoare:
a) orice trei puncte distincte ale lui S nu sunt coliniare;

105
b) pentru orice punct P al lui S există cel puţin k puncte distincte în S,
egal depărtate de P.
1
Să se arate că are loc inegalitatea k < + 2n .
2
Soluţie. Fie P1 un punct din S. Considerând cercul C1 cu centrul în P1 a
cărui circumferinţă conţine k puncte din S, rezultă prin unirea acestor puncte,
Ck2 coarde care unesc puncte ale lui S.
Fie P2 un alt punct din S. Considerând cercul C2 cu centrul în P2,
obţinem analog Ck2 coarde. Deoarece C1 I C2 au în comun cel mult o coardă
rezultă cel mult Ck2 − 1 coarde care nu au fost considerate în C1. Considerând un
nou punct P3 apar cel mult Ck2 − 2 coarde noi etc. Însumând numărul coardelor
care este majorat de numărul segmentelor ce unesc puncte din S avem:
Ck2 + (Ck2 − 1) + (Ck2 − 2) + K + [Ck2 − (n − 1)] ≤ Cn2 ⇔
n(n − 1) n(n − 1)
⇔ n ⋅ Ck2 ≤ + ⇔ k (k − 1) ≤ 2(n − 1) ⇔ k 2 − k − 2(n − 1) ≤ 0
2 2

1 + 8n − 7 1
⇒k≤ < + 2n .
2 2
R8.2.9. Câte puncte cu ambele coordonate întregi se pot afla în interiorul sau
pe laturile unui pătrat de latură 3 .
Soluţie. Fie pătratul de y
latură 2 în reperul cartezian xOy (0,1)
(v. figura). Este clar că există cinci
astfel de puncte: vârfurile pătratului
şi punctul de intersecţie al (−1, 0) 0 (1, 0)
diagonalelor . Un alt punct de
x
coordonate întregi ar fi (−1,1) ,
(1,1) , (1, −1) , (−1, −1) .
Trebuie arătat că nici unul (0, −1) (−1,1)
din aceste puncte nu se poate afla în
interiorul sau pe laturile pătratului
de latură egală cu 3 (construit
asemenea cu cel de latură 2 ).

106
3
Distanţa de la O la latura pătratului de latură 3 este de , în timp ce
2
3
distanţa de la O la punctul (1, −1) este de 2> . De aici rezultă că punctul
2
(1, −1) este exterior pătratului de latură 3 .
R8.2.10. Fie dată o mulţime A cu m elemente şi o mulţime B cu n elemente
(m, n ∈ * ) . Să se găsească numărul de permutări ale mulţimii A U B astfel
încât primul element al unei astfel de permutări să fie din A, iar ultimul din B.
Se presupune că A I B = ∅ .
Soluţie. Două elemente din A U B , primul din A, iar al doilea din B, pot
fi alese în m ⋅ n moduri. La fiecare astfel de posibilitate, cele m + n − 2 elemente
rămase pot fi aşezate pe cele m + n − 2 poziţii rămase în ( m + n − 2 )! moduri. Ca
urmare, există m ⋅ n ⋅ (m + n − 2)! permutări de tipul cerut.
R8.2.11. Fie A o mulţime cu n elemente, n ∈ * . Să se determine
card{( X , Y ) X , Y ∈ P ( A) , X U Y = A} .
Soluţie. Notăm A = {a1 , a2 ,K , an } cu n ∈ * . Fie X ∈ P ( A) ,
X = {ai1 , ai2 ,K , aik } , unde i1 , i2 ,K , ik sunt k indici distincţi din mulţimea
{1, 2,K , n} , iar k ∈ {0,1, 2,K , n} . Considerăm X fixată şi Y ∈ P ( A) astfel încât
X U Y = A . Mulţimea Y poate fi aleasă astfel: ( A \ X ) U Z , unde Z este o
submulţime a lui X, adică Y poate fi aleasă în 2k moduri (numărul total de
submulţimi ale lui X). Numărul total de soluţii va fi:
n +1
2 + 2 + 2 +K + 2 = 2 − 1 .
0 1 2 n

R8.2.12. Fie A = {1, 2,K , n} unde n ∈ * . Să se arate că ecuaţia


X U Y U Z = A , în care două soluţii care diferă doar prin ordinea termenilor se
7 n + 3n +1 + 2
consideră egale, are soluţii distincte.
6
Soluţie. Notăm pentru n ∈ * cu an numărul soluţiilor ecuaţiei cu
X ≠ Y , Y ≠ Z şi Z ≠ X , cu bn numărul soluţiilor cu X = Y şi Y ≠ Z şi cu cn
numărul soluţiilor cu X = Y = Z . Evident, a1 = 0, b1 = 2 şi cn = 1, (∀) n ∈ * .
Vom demonstra prin inducţie după n ∈ * că:
7 n − 3n +1 + 2
an = , bn = 3n − 1, (∀) n ∈ * (1)
6
Presupunând afirmaţiile adevărate pentru k ∈ * , o soluţie a ecuaţiei
X U Y U Z = A U {k + 1} = {1, 2,K , k} U {k + 1} , având componentele distincte

107
două câte două se obţine fie dintr-o soluţie a ecuaţiei X U Y U Z = {1, 2,K , k} ,
având componentele distincte două câte două (deci numărată la ak) prin
adăugarea lui k + 1 la una din componente (deci trei posibilităţi), la două din
componente (deci trei posibilităţi) sau la fiecare dintre componente (deci o
posibilitate), fie dintr-o soluţie a ecuaţiei X U Y U Z = {1, 2,K , k} cu X = Y şi
Y ≠ Z , deci numărată la bk, prin adăugarea lui k + 1 la una dintre cele două
componente egale şi adăugând sau nu pe k + 1 la cea de-a treia componentă.
Aşadar, am obţinut
an +1 = 7 an + 2bn , (∀) n ∈ * (2)
De asemenea, o soluţie a ecuaţiei X U Y U Z = {1, 2,K , k} U {k + 1} având numai
două din componente egale se obţine dintr-o soluţie a ecuaţiei
X U Y U Z = {1, 2,K , k} cu X = Y şi Y ≠ Z (deci numărată la bk) prin
adăugarea lui k + 1 la fiecare din cele 2 componente egale (o posibilitate), sau
la cea de-a treia componentă (o posibilitate), sau la fiecare din componente (o
posibilitate), sau prin adăugarea lui k + 1 unei componente sau la două din
componentele unei soluţii având toate componentele egale (deci două
posibilităţi). Aşadar, se obţine:
bn +1 = 3bn + 2, (∀) n ∈ * (3)
Avem:
(3) ip.ind.
bk +1 = 3bk + 2 = 3(3k − 1) + 2 = 3k +1 − 1 şi
(2) ip.ind.
7 k − 3k +1 + 2 7 k +1 − 3k + 2 + 2
ak +1 = 7ak + 2bk = 7 ⋅ + 2(3 − 1) =
k
.
6 6
Din ultimele două relaţii se obţine că P (k + 1) este adevărată, deci relaţia (1)
este adevărată conform metodei inducţiei matematice. Ca urmare, numărul de
soluţii cerut este:
7 n − 3n +1 + 2 n 7 n + 3n +1 + 2
S n = an + bn + cn = + 3 −1+1 = .
6 6
R8.2.13. Fie
{
Fn = f f :{1, 2,K , n} → {1, 2,K , n}, f injectiv ã º i f (i ) ≠ i , (∀) i = 1, n . }
Determinaţi numărul elementelor lui Fn.
A finit ã
Soluţie. Fie A = {1, 2,K , n} . Cum f : A → A injectivă ⇒ f bijectivă
⇒ f permutare a mulţimii A. Considerăm
Ai = { f : A → A f permutare ºi f (i) = i} , i = 1, n . Folosind principiul
închiderii şi excluderii, numărul tuturor permutărilor ce admit cel puţin un punct
fix este:

108
n n
A1 U A2 UKU An = ∑ Ai − ∑ Ai I Aj + K + (−1) n −1 ⋅ I Ai ,
i =1 i ≤i < j ≤ n i =1

unde Ai = card Ai .
Deoarece Ai1 U Ai2 UKU Aik = (n − k )! şi în fiecare sumă din egalitatea
precedentă există Cnk termeni, obţinem:
A1 U A2 UKU An = Cn1 ⋅ (n − 1)!− Cn2 ⋅ (n − 2)!+ K + (−1) n −1 Cnn .
Aşadar, numărul permutărilor lui A fără puncte fixe este
Fn = n !− A1 U A2 UKU An =
= n !− Cn1 ⋅ (n − 1)!− Cn2 ⋅ (n − 2)!+ K + (−1) n −1 Cnn  =
 1 1 1 (−1) n 
= n !1 − + − + K + .
 1! 2! 3! n! 
R8.2.14. Să se cerceteze de câte ori într-o zi (12 ore), orarul, minutarul şi
secundarul împart cadranul unui ceas în trei arce congruente.
Soluţie. Privim cercul ca un disc de rază 1 în planul complex. Dacă într-
o unitate de timp orarul parcurge un arc de lungime α , atunci minutarul
parcurge un arc de lungime 12 ⋅ α , iar secundarul un arc de lungime 12 ⋅ 60 ⋅ α .
Notăm z = cos α + i sin α şi z0 (t ), zm (t ) , zs (t ) poziţiile orarului,
minutarului şi secundarului după timpul t. Avem z0 (t ) = z t , zm (t ) = z12t şi
2π 2π
zs (t ) = z12⋅60⋅t . Dacă ε = cos + i sin , (ε 3 = 1) , atunci la momentul t cele
3 3
 2π 
trei limbi ale ceasului împart cadranul în arce congruente  de lungime  , în
 3 
două situaţii:
a) zm (t ) = ε ⋅ z0 (t ) şi zs (t ) = ε ⋅ zm (t ) .
b) zm (t ) = ε ⋅ z0 (t ) şi zs (t ) = ε ⋅ zm (t ) .
În cazul a) avem: z12t = z t ⋅ ε şi z12⋅60⋅t = z12t ⋅ ε ⇔ z11t = ε şi z12⋅59t = ε .
Obţinem:
z12⋅59t = z11⋅59t ⋅ z 59t = ε 59 ⋅ z 59t = ε 59 ⋅ ( z11t )5 ⋅ z 4t = ε 59 ⋅ ε 5 ⋅ z 4t = ε 64 ⋅ z 4t = ε ⋅ z 4t ,
deci
z 4t = 1 ⇒ z t ∈ {±1, ±i} ⇒ z11t ∈ {±1, ±i} ⇒ z11t ≠ ε , contradicţie.
Analog, în cazul b), nu există soluţie. Ca urmare, numărul căutat este zero.
R8.2.15. Fie n ∈ * şi a1 ≤ a2 ≤ K ≤ an numere naturale nenule.

109
Determinaţi numărul funcţiilor injective f :{1, 2,K , n} → *
cu proprietatea
f (k ) ≤ ak , (∀) k = 1, n .
Soluţie. Construim o funcţie injectivă ca şi în enunţ. Avem a1 posibilităţi
de a alege f (1) , apoi a2 − 1 posibilităţi de a alege f (2) deoarece
f (2) ∈ {1, 2,K , a2 } \{ f (1)} .
Putem alege apoi pe f (3) în a3 − 2 moduri etc. Va rezulta că numărul
căutat este a1 (a2 − 1)(a3 − 2) ⋅K ⋅ (an − n + 1) .
R8.2.16. a) Care este cel mai mare număr de turnuri ce pot fi aşezate pe tabla
de şah astfel ca ele să nu se ameninţe? Câte astfel de aranjări există?
b) Care este cel mai mic număr de turnuri astfel ca ele să ţină sub
ameninţări toate pătratele tablei? În câte moduri pot fi aranjate acestea?
Soluţie. a) Pe fiecare linie şi fiecare coloană trebuie să fie câte un singur
turn. Aşadar, numărul maxim de turnuri este 8. O aranjare determină o
 1 2 K 8 
permutare   , turnul de pe linia i, este pus pe coloana
 σ (1) σ (2) K σ (8) 
σ (i ) . Se obţin 8! = 1⋅ 2 ⋅ 3 ⋅K ⋅ 8 moduri de aranjare.
b) Pentru a ameninţa pătratele unei linii avem nevoie de cel puţin un turn
pe ea, aşadar, numărul minim cerut este 8. Putem aranja câte un turn pe fiecare
linie în 88 moduri, putem aşeza câte un turn pe fiecare coloană în 88 moduri, dar
în acest mod unele aranjări s-au numărat de două ori (cele care ocupă simultan
toate liniile şi coloanele, deci 8! aranjări). Se obţin 2 ⋅ 88 − 8! moduri de
aranjare.
R8.2.17. Să se determine numărul funcţiilor f :{1, 2,K , n} → {1, 2,3, 4,5} cu
proprietatea că f (k + 1) − f (k ) ≥ 3, k = 1, n − 1 (n ∈ , n ≥ 2) .
Soluţie. Fie an , bn , cn , d n numărul funcţiilor căutate pentru care
f (n) = 1, 2, 4 sau 5 (în condiţiile din ipoteză avem f (k ) ≠ 3, k = 1, n ). Avem
relaţiile de recurenţă:
an +1 = cn + d n
b = d
 n +1 n

cn +1 = an
d n +1 = an + bn
Numărul căutat este xn = an + bn + cn + d n .

110
Avem: a2 = 2 ( f (2) = 1 º i f (1) = 4 sau f (1) = 5) ,
b2 = 1 ( f (2) = 2 º i f (1) = 5) , c2 = 1 ( f (2) = 4 şi f (1) = 1 ), d 2 = 2 ( f (2) = 5 şi
f (1) = 1 sau f (2) = 2 ).
Va rezulta an = d n şi bn = cn şi folosind relaţiile anterioare, găsim:
an +1 = an + bn
 ⇒ an +1 = an + an +1 , (∀) n ≥ 3 .
bn +1 = an
Avem: xn = 2(an + bn ) = 2an +1 , x2 = 6 = 2 ⋅ 3, x3 = 10 = 2 ⋅ 5 .
Din x2 = 2 F4 , x3 = 2 F5 şi xn +1 = xn + xn −1 rezultă xn = 2 ⋅ Fn + 2 unde ( Fn ) n ≥0 este
şirul lui Fibanacci, dat prin: F0 = 0, F1 = 1 , Fn +1 = Fn + Fn −1 , (∀) n ∈ * şi a
cărui termen general este:
1  1 + 5   1 − 5  
n n

Fn =   −  .
5  2   2  
 
R8.2.18. În câte moduri poate fi pavat un dreptunghi de dimensiuni 2 × n cu
plăci 1× 2 ?
Soluţie. Notăm cu an numărul căutat.

Fig.1

n +1

Fig.2

Dacă în an +1 o aranjare se termină cu o placă orizontală (de fapt două


plăci orizontale suprapuse, vezi fig.1), se obţin an −1 pavări iar dacă se termină
cu o placă verticală ca în figura 2, se obţin an pavări. Atunci obţinem recurenţa:
an +1 = an + an −1 , a1 = 1, a2 = 2 deci
an = Fn +1 , unde ( Fn ) n ≥0 este şirul lui Fibanacci.

111
9. Sume combinatorice

Fundamentarea analizei combinatorii ca disciplină ştiinţifică a început în


secolul al XVII-lea. Într-un manuscris din secolul al III-lea d.H. este precizată
 n  n(n − 1)
formula   = , iar în secolul al XII-lea, matematicianul hindus
 2 2
n
Bhaskara precizează formula generală pentru   . Un studiu mai sistematic se
p 
găseşte într-un manuscris al lui Levi Bengerson, la începutul secolului al XIII-
lea, când obţine formula de recurenţă care îi permite să calculeze Anp , şi în
particular numărul permutărilor de n obiecte. Tot el enunţă reguli echivalente
 n  n n 1
cu relaţiile   =   şi respectiv   = Anp , dar manuscrisul lui se
n − p  p  p  p!
pare că a fost ignorat de către contemporani.
B. Pascal (1623-1662) este primul care a observat relaţia dintre
combinări şi formula binomului. Dezvoltarea binomului lui Newton, ( x + 1) ,
m

era cunoscută de arabi în secolul al XIII-lea, aşa cum menţionează N. Tartaglia


(1500-1557) în „Tratatul general al numerelor”.
Cel care a dat un fundament propriu-zis ştiinţific combinărilor şi
permutărilor a fost G. W. Leibniz (1646-1716) în „Disertaţie despre arta
combinatorie”. Toate simbolurile actuale folosite în teoria combinatorilor
datează din secolul al XIX-lea.
Combinatorica se interferează cu disciplinele matematice axiomatizate
din care extrage metode sau pe care le serveşte cu rezultate. Metodele
combinatoricii sunt utilizate în rezolvarea problemelor de transport şi de stocare
a bunurilor. Legături au fost făcute între combinatorică şi problemele de
programare liniară, statistică, etc. Metodele combinatoricii sunt utilizate în
codificarea şi decodificarea informaţiilor, ca şi în alte probleme de teoria
informaţiilor.
În acest capitol ne-am propus să dăm câteva modalităţi de obţinere a
unor sume de combinări cu ajutorul binomului lui Newton sau pornind de la
anumite identităţi dar fără a apela la calculul diferenţial şi integral.

112
9.1. Noţiuni teoretice relativ la elemente de combinatorică

9.1.1. Exemplu: Cu elementele alfabetului latin pot fi formate


cuvintele limbilor ce folosesc acest alfabet. Să considerăm câteva cuvinte cu
patru litere constituite din elementele mulţimii {a, e, m, r}, submulţime a
mulţimii literelor alfabetului. Mulţimea cuvintelor {arme, rame, mare, eram}
reprezintă cuvinte cu sens precis, formate din aceleaşi litere, totalitatea
grupărilor fiind 24.

9.1.2. Observaţie: Fie mulţimile A = {a1 , a 2 , K , a n } şi


B = {b1 , b2 , K , bn } , n ∈ Ν * . Vom studia numărul aplicaţiilor bijective ale
mulţimii A pe mulţimea B, notate cu f : A → B . În general nu ne interesează
natura mulţimilor A şi B, astfel că elementele lor se pot nota fie cu literele
alfabetului, fie cu {1, 2, K , n} aplicaţiile definite fiind de forma f (k ) = ik , unde
ik ∈ {1, 2, K , n} .

9.1.3. Definiţie: Aplicaţiile bijective f : {1, 2, K , n} → {1, 2, K , n} se


numesc permutările mulţimii {1, 2, K , n} . Numărul acestor aplicaţii bijective se
notează cu Pn şi se citeşte permutări de n .
Aflarea numărului permutărilor unei mulţimi se face prin inducţie şi se
obţin Pn = 1 ⋅ 2 ⋅ 3 ⋅ L ⋅ n, ∀ n ∈ N * .

9.1.4. Notaţie: Convenim să notăm 1 ⋅ 2 ⋅ 3 ⋅ L ⋅ n = n! care se citeşte „n


factorial”. Cu această notaţie, Pn = n!, ∀ n ∈ N * , admitem prin definiţie 0 ! = 1 .

9.1.5. Definiţie: Aplicaţiile injective


f : (1, 2, K , k ) → (1, 2, K , n ), 0 ≤ k ≤ n, n, k ∈ N , n ≥ 1 , se numesc aranjamente
de n elemente luate câte k. Numărul acestor aplicaţii injective se notează cu
Ank şi se citeşte: aranjamente de n luate câte k .

113
Aflarea numărului aplicaţiilor injective ale mulţimii {1, 2, K , k} pe
mulţimea {1, 2, K, n} o vom face prin inducţie după k , având pe n fixat.
Pentru k = 1 , avem aplicaţiile f (1) = 1, f (1) = 2, K , f (1) = n , deci numărul lor
este n adică An1 = n .
Cercetăm pentru k=2 numărul aplicaţiilor injective
f : {1, 2} → {1, 2, K , n}.
 f (1) = 1
 ∀ p ∈ {2, 3, K , n}
 f (2) = p
 f (1) = 2
 ∀ p ∈ {1, 3, K , n}
 f (2) = p
…………………………
 f (1) = n
 ∀ p ∈ {1, 2, K , n − 1}.
 f (2) = p
Fiecare dintre aplicaţiile date fiind în număr de n − 1 , numărul total al
lor este n(n − 1) , deci An2 = n(n − 1) .
Pentru k = 3 aplicaţiile injective f : {1, 2, K ,3} → {1, 2, K , n} sunt
f : {1, 2} → {1, 2, K , n}, f (3) = p , p fiind diferit de valorile luate în 1 şi 2, deci
în total p poate lua n − 2 valori. Prin urmare numărul total al aplicaţiilor
injective f : {1, 2, K ,3} → {1, 2, K , n} este An3 = n(n − 1)(n − 2) .
Presupunem că Ank = n(n − 1)L (n − k + 1) , 0 ≤ k ≤ 1 şi demonstrăm
Ank +1 = n(n − 1)L (n − k + 1)(n − k ) .
Aplicaţiile injective f : {1, 2, K , k , k + 1} → {1, 2, K , n} sunt
f : {1, 2, K , k } → {1, 2, K , n} , f (k + 1) = p , p fiind diferit de valorile luate în
1, 2, K , k deci în total p poate lua n − k valori. În concluzie numărul total al
aplicaţiilor injective f : {1, 2, K , k , k + 1} → {1, 2, K , n} este
Ank +1 = [n(n − 1)L (n − k + 1)](n − k ) , 0 ≤ k < n
Pentru k = n aplicaţia injectivă f : {1, 2, K , n} → {1, 2, K , n} este şi
bijectivă. Se impune demonstraţia subjectivităţii. Demonstraţia o facem prin
reducerea la absurd. Avem f ({1, 2, K , n}) = n şi presupunem că f nu este

114
surjectivă. Rezultă că există ik ∈ {1, 2, K , n} care nu este imaginea nici unui
element din mulţimii de definiţie. Am ajuns la contradicţie, numărul valorilor
funcţiei fiind mai mic decât n , deci f nu este injectivă. Prin urmare, pentru
k = n avem Ann = Pn . Aşadar rezultă:
n!
Ank = .
(n − k )!

9.1.6. Definiţie: Aplicaţiile injective f : {1, 2, K , k } → {1, 2, K , n},


0 ≤ k ≤ n , n, k ∈ N , n ≥ 1 pentru care codomeniile sunt formate din mulţimi
distincte, se numesc combinări de n elemente luate câte k .
n
Numărul combinărilor de n elemente luate câte k se notează cu   şi
k 
se citeşte combinări de n luate câte k.
Au loc relaţiile:
 n  n(n − 1)(n − 2 )L (n − k + 1)
  = , 0 ≤ k ≤ n , n, k ∈ N , n ≥ 1 .
k  1⋅ 2 ⋅ 3 ⋅L⋅ k
 n  Ank
  = , 0 ≤ k ≤ n , n, k ∈ N , n ≥ 1 .
 k  Pk
n n!
  = , 0 ≤ k ≤ n , n, k ∈ N , n ≥ 1 .
 k  k!(n − k )!

n
9.1.7. Proprietăţi ale numerelor  
k 
(1) Formula combinărilor complementare
n  n 
  =   , 0 ≤ k ≤ n , n, k ∈ N , n ≥ 1 .
k  n − k 
(2) Formula de descompunere a combinărilor
 n   n − 1  n − 1
  =   +   , 0 < k < n , n, k ∈ N * .
 k   k   k − 1
(3) Formule de recurenţă pentru combinări
115
n n  n − 1
  =   , 0 < k ≤ n , n, k ∈ N * ;
k  k  k − 1
n k + 1  n + 1
  =   , 0 ≤ k ≤ n , n, k ∈ N , n ≥ 1 .
k  n + 1  k + 1
Demonstraţie:
(3)
n  n − 1 n (n − 1)(n − 2) ⋅ L ⋅ (n − k + 1) Ank  n 
 = ⋅ = = ;
k  k − 1 k 1 ⋅ 2 ⋅ L ⋅ (k − 1) Pk  k 
k + 1  n + 1 k + 1
  = ⋅
(n + 1)! = n! =  n  .
n + 1  k + 1 n + 1 (k + 1)!(n − k )! k!(n − k )!  k 

9.1.8. Teoremă (binomul lui Newton):


Are loc următoare formulă:

(a + b )n =  a n +  a n−1b +  a n−2 b 2 + L +  a n−k b k + L +  ab n−1 +  b n
n n n n n n
 0 1  2 k   n − 1 n

cunoscută sub denumirea de formula lui Newton (1643 - 1727).

9.1.9. Observaţii:
(1) Formula lui Newton poate fi scrisă sub formă condensată astfel:
nn
(a + b )
= ∑  a n − k b k .
n

k =0  k 

Dacă se doreşte o formulă analogă pentru binomul diferenţă, formula


devine:

(a − b )n = ∑ (− 1)k  a n−k b k .


n n
k =0 k 
n n n  n  n
(2)
Coeficienţii  ,  , K ,  , K ,  ,   se numesc coeficienţi
0 1 k   n − 1  n 
binomiali şi se calculează cu formulele combinărilor.

116
(3) Dacă n este un număr par, dezvoltarea conţine un număr impar de
termeni, existând un termen din mijloc al dezvoltării, T n  , care are
 2  +1
 

coeficientul terminal cel mai mare.


(4) Dacă n este impar, dezvoltarea conţine un număr par de termeni,
 
termenii din mijloc  T n  şi T n   având coeficienţii terminali egali, cu
   +1  2 +2 
 2   
valoare maximă.

9.1.10. Proprietăţi ale binomului lui Newton


(1) Numărul termenilor din dezvoltarea binomului (a + b )n este n + 1 .
(2) Coeficienţii binomiali ai termenilor extremi din dezvoltarea sunt
egali, de asemenea coeficienţii binomiali ai termenilor egali depărtaţi de
n  n 
extremi, întrucât   =   , k ∈ {0, 1, K , n}.
k  n − k 
n
(3) Termenul  a n − k b k este al (k + 1) -lea termen al dezvoltării
k 
n
binomului şi se numeşte termen general. Se notează cu Tk + 2 =  a n − k b k ,
k 
k ∈ {0, 1, K , n}.
(4) Între doi termeni consecutivi ai dezvoltării există relaţia:
n−k
Tk + 2 = ⋅ Tk +1
k +1

9.1.11. Identităţi în calculul cu combinări:


(1) Particularizând a = b = 1 în formula lui Newton, avem:
 n  n  n   n
2n =   +   + L +   +  
 0 1  n − 1  n 

(2) Avem: a)   +   +   + L = 2n −1


n n n
 0  2  4

117
n  n  n
b)   +   +   + L = 2n −1
1  3  5
Demonstraţie: (2 ) În formula lui Newton punem a = 1, b = −1 şi
obţinem:
 n  n  n  n nn
0 =   −   +   −   + L + (− 1)   .
 0  1  2  3 n
n  n  n  n
Adunând această relaţie cu 2n =   +   +   + L +   rezultă a) iar
 0 1  2  n
scăzându-le se obţine b).

9.1.12. Teorema binomului factorial


Se notează a(a − h )(a − 2h ) ⋅ L ⋅ [a − (n − 1)h] = a n| h , astfel în particular
a n|0 = a n , a1|h = a . Să se demonstreze că are loc egalitatea:

(a + b )n|h = a n|h +  a (n−1)|h ⋅ b1|h +  a (n−2 )|h ⋅ b 2|h + L + b n|h
n n
1  2
Demonstraţie: Teorema se demonstrează prin metoda inducţiei
complete. Ea conţine drept caz particular, (pentru h = 0 ) teorema obişnuită a
binomului lui Newton. Este uşor de verificat că pentru n = 1 şi n = 2 , teorema
binomului factorial este adevărată. Se va presupune că această teoremă este
adevărată pentru exponentul n, adică

(a + b )n|h +  a (n−1)|h ⋅ b +  a (n−2 )|h ⋅ b 2|h + L + b n|h şi se va arăta că în acest caz
n n
1  2
teorema va fi adevărată şi pentru n + 1 . Într-adevăr, înmulţind cu a + b − nh
ambii membrii ai egalităţii care exprimă teorema pentru exponentul n , în partea
stângă vom obţine evident (a + b )
( n +1)|h
; în partea dreaptă, unde figura suma
 n
având ca termen de rang k pe  a (n−k )|h ⋅ b k|h , vom obţine după înmulţirea cu
k 
a + b − nh ca expresie a acestui termen

 n  (n−k )|h k |h n


 a ⋅ b ⋅ (a + b − nh ) =  a (n−k )|h b k|h {[a − (n − k )h] + (b − kh )} =
k  k 

118
n n
=  a (n−k )|h [a − (n − k )h]b k|h +  a (n−k )|h b k|h (b − kh ) =
k  k 
n n
=  a (n−k +1)|h ⋅ b k|h +  a (a−k )|h b (k +1)|h .
k  k 
 n   n   n + 1
Conform relaţiei cunoscute   +   =   , rezultă, ca şi în
 k − 1  k   k 
cazul teoremei obişnuite a binomului lui Newton, că după înmulţirea
membrului întâi cu a + b − nh se va obţine o sumă de termeni de forma
 n + 1 (n+1−k )|h k|h
 a ⋅ b . Cu aceasta, s-a demonstrat că, dacă teorema binomului
 k 
factorial este adevărată pentru exponentul n , ea este adevărată şi pentru
exponentul n + 1 . Deoarece această teoremă este adevărată pentru n = 1 , rezultă
valabilitatea ei pentru orice n .

9.2. Metode de calcul al sumelor de combinari

9.2.1. Calculul unor sume cu combinări pornind de la identităţi


Se consideră identităţile:
n
n n
1  n  2 n +1 − 1
∑ k 
 
k =0  k 
= n ⋅ 2 n −1
şi ∑   =
k =0 k + 1  k  n +1
Să se arate că au loc următoarele identităţi:

(1) ∑ k 2   = n(n + 1) ⋅ 2 n−2 ;


n n
k =0 k 

(2) ∑ (k + p )2   = 2 n−2 (n 2 + n + 4 pn + 4 p 2 ) ;


n n
k =0 k  

(3) ∑ k (k + 1)(k + 2)  = n ⋅ 2 n−3 ⋅ (n 2 + 9n + 14) ;


n n
k =0 k  

(4) ∑ 1   = n ⋅ 2 + 1 ;


n +1
n n
k+2 k
k =0 (n + 1)(n + 2)
 

119
(5) ∑ k + 2   = (n + 3) ⋅ 2 −1
n n n
;
k +1 k
k =0   n +1
n
k+p  n  2 n +1 (n + 2 p − 2) − pn + 3 p + n − 8
(6) ∑   = .
k = 0 (k + 1)(k + 2 )  k  (n + 1)(n + 2)

Soluţii:
n −1
(1) Avem k 2   = k ⋅ k ⋅ n! = k ⋅ n ⋅   , deci
n
k  k!(n − k )!  k − 1
n
2n
n
 n − 1  n  n − 1 n  n − 1 
∑ k  
k  = n ∑ k 
 
 = n ∑ (k − 1)  + ∑   =
k =0   k =1  k − 1  k =1  k − 1  k =1  k − 1 
[ ]
= n (n − 1) ⋅ 2 n − 2 + 2 n −1 = n ⋅ 2 n − 2 (n − 1 + 2) = n(n + 1) ⋅ 2 n − 2 .

(2) ∑ (k + p )2   = ∑ (k 2 + 2kp + p 2 )  = ∑ k 2   + 2 p ∑ k   +
n n p
n n n n n
k =0 k   k =0 k   k =0  k kk =0  

n
( )
p
+ p 2 ∑   = n(1 + 1) ⋅ 2 n − 2 + 2 pn ⋅ 2 n −1 + p 2 ⋅ 2 n = 2 n − 2 ⋅ n 2 + n + 4 pn + 4 p 2
k =0  k 

(3) ∑ k (k + 1)(k + 2)  = ∑ (k 3 + 3k 2 + 2k )  = ∑ k 3   + 3 ⋅ ∑ k 2   +
n n n n n n n n
k =0 k   k =0 k   k k =0   k k =0  
n
( )
n
+ 2∑ k   = n 2 ⋅ (n + 3) ⋅ 2 n −3 + 3n(n + 1) ⋅ 2 n − 2 + 2n ⋅ 2 n −1 = n n 2 + 9n + 14 ⋅ 2 n −3
k =0  k 
n
n
cu observaţia că pentru calculul sumei ∑ k 3   s-a folosit identitatea
k =0 k 
n  n − 1
k 3   = k 2 ⋅ n  .
k   k − 1
n + 2
(4) ∑ 1   = ∑ 1 ⋅ n! = ∑ k + 1 
n n n n
 =
k =0 k + 2  k  k = 0 k + 2 k!(n − k )! k = 0 (n + 1)(n + 2 )  k + 2 

1 n
 n + 2 1  n  n + 2  n  n + 2 
( ) ( )
(n + 1)(n + 2) ∑  k + 2  (n + 1)(n + 2 ) ∑
= k + 1   = k + 2   − ∑  
k =0    k = 0  k + 2  k = 0  k + 2 

120
) [ ]}
n +1
=
1
(n + 2) 2 n+1 − 1 − 2 n+ 2 − (n + 2) − 1 = n ⋅ 2 + 1 .
{ (
(n + 1)(n + 2) (n + 1)(n + 2)
n +1
(5) ∑ k + 2   =∑ k + 2 ⋅ n! = 1 ∑ (k + 2) 
n n n n
=
k +1 k
k =0 k + 1 k!(n − k )! n + 1
  k =0 k + 1 k =0  
n
 n + 1  n  n + 1
∑ (k + 1)  + ∑  
2 n (n + 3) − 1
=
k =0  k + 1 k =0  k + 1 = .
n +1 n +1
(6) Avem
k+p n n! 1 k+p  n + 2
  = (k + p ) ⋅ ⋅ =  
(k + 1)(k + 2)  k  k!(n − k )! (k + 1)(k + 2) (n + 1)(n + 2)  k + 2 

n
k+p n 1 n
 n + 2
deci ∑   =
  (n + 1)(n + 2 ) ∑ (k + p )  =
k = 0 (k + 1)(k + 2 )  k  k =0  k + 2
1 n  n + 2 n
 n + 2 
= ∑ (k + 2 )
 
 + ( p − 2 )∑   =
(n + 1)(n + 2)  k =0  k + 2 k =0  k + 2  

1
(n + 1)(n + 2)
{[ ] [ (
(n + 2) ⋅ 2 n+1 − (n + 2) + ( p − 2) 2 n+ 2 − n − 3 = )]}
1
(n + 1)(n + 2)
[
2 n +1 (n + 2 p − 2) − pn + 3 p + n − 8 . ]

9.2.2. Calculul unor sume cu combinări aplicând teorema binomului


lui Newton
Să se aplice teorema binomului lui Newton la determinarea valorii
următoarelor sume:

(1)   − 2  + 3  − L + (− 1)n n  ;
n n n n
1  2  3 n
 2n   2n − 1  2n − 2  n
(2)   + 2  + 4  + L + 2 n   ;
n  n   n  n
2 2 2 2

(3)   +   +   + L +   ;


n n n n
 0  1  2 n
121
 n  n  n  n
(4)   +   +   +   + L ;
 0  3  6 9

(5)   +   +   +   + L ;


n n n n
 1   5   9  13 

Soluţii:
n  n − 1
(1) Folosim faptul că k   = n  şi obţinem:
k   k − 1
n n n n  n − 1  n − 1  n − 1
  − 2  + 3  − L + (− 1) ⋅ n  = n 
n −1
 −   +   − L +
1  2  3 n   0   1   2 

n −1  n − 1 
+ (− 1)   = n(1 − 1) = 0 .
n −1

 n − 1 
(2) Expresia căutată este egală cu coeficientul lui x n în polinomul:
(1 + x )2n + 2(1 + x )2n−1 + 2 2 (1 + x )2 n−2 + L + 2 n ⋅ (1 + x )n .
Se va transforma acest polinom folosind formula pentru suma termenilor
unei progresii geometrice:
(1 + x )2n + 2(1 + x )2 n−1 + 2 2 (1 + x )2 n−2 + L + 2 n (1 + x )n =
 2 22 2n 
= (1 + x )
2n
1 + + +L+ =
 1 + x (1 + x )
2
(1 + x )n 

2n 
= (1 + x ) 
2 n +1  2
− 1

[
− 1 = 2 n +1 ⋅ (1 + x ) − (1 + x )
n 2 n +1 1
. ]
 (1 + x )
n +1
 1 + x  1− x
1
Dar (pentru x < 1 ): = 1 + x + x 2 + x 3 + L . Deci suma căutată este
1− x
egală cu coeficientul lui x n din expresia:
( )
2 n +1 ⋅ (1 + x ) 1 + x + x 2 + L − (1 + x )
n 2 n +1
(1 + x + x 2
)
+L .
Dacă vom înmulţi un polinom în x cu 1 + x + x 2 + L , coeficientul lui
x n în acest produs va fi egal cu suma coeficienţilor puterilor lui x , nu mai mari
decât n din polinomul iniţial. Într-adevăr, termenii în x n din produs se obţin

122
prin înmulţirea termenilor în x k ai polinomului, unde k ≤ n , respectiv cu
termenii x n−k ai sumei 1 + x + x 2 + L. Astfel, produsul
( )
2 n +1 (1 + x ) 1 + x + x 2 + L , după desfacerea parantezelor va conţine termenul
n

în x n cu un coeficient egal cu suma tuturor coeficienţilor polinomului


2 n +1 (1 + x ) , adică cu un coeficient egal cu 2 n +1 ⋅ 2 n = 2 2 n +1 Coeficientul lui x n
n

în produsul (1 + x )2n+1 ⋅ (1 + x + x 2 + L) va fi egal cu suma coeficienţilor


polinomului (1 + x ) , care stau în faţa lui x 0 = 1, x, x 2 , K , x n , adică va fi egal
2 n +1

cu suma coeficienţilor din prima jumătate a polinomului (1 + x ) . Dar,


2 n +1

 2n   2n 
deoarece   =   , rezultă că această sumă este egală cu semisuma
 k   2n − k 
tuturor coeficienţilor lui (1 + x )
2 n +1
, adică este egală cu 2 n +1 : 2 = 2 2 n . De aici
rezultă că coeficientul lui xn în expresia
n
( )
2 n +1 (1 + x ) 1 + x + x 2 + L − (1 + x )
2 n +1
(1 + x + x 2
+L ) este egal cu
2 2 n +1 − 2 n = 2 2 n .
(3) Expresia căutată este egală cu coeficientul lui x n în produsul
următor:
 n  n  n 2  n   n  n   n  2 n n 
   +   x +   x + L +   x n  ⋅    +   x +  
  n − 2 x + L +   x 
 0 1 2 n   n n − 1
                 0 

 n  n n  n   n  2 n


Însă   =   şi deci,   +   x +   x + L +   x n =
n − k  k   n   n − 1  n − 2  0
 n  n  n n
=   +   x +   x 2 + L +   x n = (1 + x )? Astfel, trebuie să găsim
 0 1  2 n
doar coeficientul lui xn în produsul (1 + x )n ⋅ (1 + x )n = (1 + x )2n . Acest
 2n 
coeficient este egal cu   .
n

123
(4) ( )
Deoarece x 3 − 1 = ( x − 1) ⋅ x 2 + x + 1 fie ε 1 şi ε 2 cele două rădăcini
ale ecuaţiei de gradul al doilea x 2 + x + 1 = 0 . Observăm că ε 12 = ε 2 , ε 22 = ε 1 ,
ε 13 = ε 23 = 1 , 1 + ε 1 + ε 2 = 1 + ε 1 + ε 12 = 1 + ε 2 + ε 22 = 0 .
Conform binomului lui Newton se obţine:

(1 + 1)n =   +   +   +   + L +  
n n n n n
 0 1  2  3 n
n  n n n n
(1 + ε 1 )n =   + ε 1   + ε 12   + ε 13   + L + ε 1n  
0 1  2  3 n

(1 + ε 2 )n =   + ε 2   + ε 22   + ε 23   + L + ε 2n  
n n n n n
0 1  2  3  n
Conform proprietăţilor anterior anunţate ale numerelor ε 1 şi ε 2 suma
1 + ε 1k + ε 2k este egală cu zero pentru k prim, şi egală cu 1 + 1 + 1 = 3 pentru k
divizibil cu 3. Deci, adunând cele trei egalităţi, vom obţine:
 n  n  n  n 
2 n + (1 + ε 1 ) + (1 + ε 2 ) = 3   +   +   +   + L .
n n

 0  3  6 9 
Se trece acum la forma trigonometrică a numerelor complexe:
2π 2π 4π 4π π π
ε 1 = cos + i sin , ε 2 = cos + i sin , 1 + ε 1 = cos + i sin ,
3 3 3 3 3 3
π π
1 + ε 2 = cos − i sin . Utilizând formula lui Moivre, se obţin:
3 3
nπ nπ nπ nπ
(1 + ε 1 )n = cos+ i sin şi (1 + ε 1 ) = cos
n
+ i sin
3 3 3 3

de unde: 2 n + (1 + ε 1 ) + (1 + ε 2 ) = 2 n + 2 ⋅ cos
n n
.
3
n n n n 1 nπ 
Deci,   +   +   +   + L = 2 n + 2 cos .
 0  3  6 9 3 3 
(5) Se scrie dezvoltarea binoamelor (1 + 1)n , (1 − 1)n , (1 + i )n şi
(1 − i )n după teorema binomului lui Newton:
n  n  n  n n
2 n =   +   +   +   + L +  
 0 1  2  3 n

124
 n  n  n  n n n
0 =   −   +   −   + L + (− 1)  
 0  1  2  3 n

(1 + i )n =   + i  + i 2   + i 3   + L + i n  
n n n n n
0 1  2  3 n
n n n n n
(1 − i )n
=   − i  + i 2   − i 3   + L + (− 1) i n   .
n

0 1  2  3 n


Pentru rezolvarea problemei se va considera suma:
(1 + 1)n − (n − 1)n − i(1 + i )n + i(1 − i )n .
de verificat că 1 − (− 1) − i ⋅ i k + i (− 1) ⋅ i k
k k
Nu este greu = 0 pentru k
care dă prin împărţirea cu 4 resturile 0, 2 sau 3 şi
( ) ( )
1 − − 1k − i ⋅ i k + i ⋅ − 1k ⋅ i k = 4 pentru k care dă restul 1 prin împărţirea cu 4,
deci:
 n  n  n  n  
(1 + 1)n − (n − 1)n − i(1 − i )n
= 4   +   +   +   + L .
  1   5   9  13  
Folosind forma trigonometrică a numerelor 1 + i şi 1 − i se obţine:
 n  n  n  n  n−2 nπ
  +   +   +   + L = 2 n − 2 + 2 2 ⋅ sin .
 1   5   9  13  4

9.2.3. Calculul unor sume cu combinări aplicând teorema binomului


factorial
Să se utilizeze teorema binomului factorial la calculul valorii sumei:
 n  m   n  m   n  m   n  m 
   +    +    + L +   
 0  k   1  k − 1  2  k − 2   k  0 

Soluţie:
 n  n(n − 1)L (n − i + 1) n i|1
Deoarece   = = se va obţine
i i! i!
 n  n  n i|1 m (k −i ) |1 1 k! 1 k 
   = = ⋅ n i|1 m (k −1) |i ⋅ ⋅  n i|1 m (k −i ) |1 . Deci suma
 i  k − i  i!(k − i )! k! i!(k − i )! ki  i 
căutată este egală cu:

125
1   k  k |1  k  (k −1)|1  k  (k −2 )|1 2|1  k  k |1  (m + n )k |1  m + 1
  m +  m n +  
 2 m n + L +  n  = =  
k!   0  1   k   k!  k 
Suma se poate obţine şi plecând de la identitatea
(1 + x )n ⋅ (1 + x )m = (1 + x )m+ n unde se va determina coeficientul lui x k .

9.2.4. Calculul unor sume cu combinări aplicând relaţii de recurenţă


Să se arate că pentru orice număr natural n au loc identităţile:
n+ k 1
(1) ∑ 
n
 = 2n ;
k =0  k  2 k
1, n = 3 p
n
 2n − k  
(2) ∑ (− 1)  k
 = 0, n = 3 p + 1 , p ∈ N .
k =0  k  − 1, n = 3 p + 2

Soluţii:
n+ k 1
(1) Să notăm f (n ) = ∑ 
n
 k . Deducem f (1) = 2 şi f (n + 1) =
k =0  k 2
n +1 n + 1 + k n +1 n + k
  1   1 n +1  n + k  1  2n + 1 1
= ∑   k = ∑   k + ∑   k = f (n ) +   n +1 +
k =0  k 2 k =0  k 2 k =1  k − 1  2  n +1  2
1 n + 2  n + 1 + k − 1 1  2n + 2  1 1
+ ∑   k −1 −   n + 2 = f (n ) + f (n + 1) . Deci
2 k =1  k − 1  2  n +1  2 2
f (n + 1) = 2 f (n ) pentru orice n ≥ 1 , n ∈ N ceea ce implică f (n ) = 2 n .
(2) Suma căutată se mai scrie:
k  2n − k   2n   2n − 1
S n = ∑ (− 1)   =   −   +
k ≥0  k  0  1 
 2n − 2   2n + 1  2n   2n − 1  2n − 1  2n − 2   2n + 1
+   − L =   −   +   +   −   + L =   −
 2   0  1  0   2   1   0 

 2n   2n − 1  2n − 1  2n − 2   2n − 3   2n + 2   2n + 1
−   +   − L +   −   +   − L =   −   +
1  2   0   1   2   0   1 

126
 2n   2n   2n − 1  2n   2n − 1  2n − 2 
+   +   −   + L +   −   +   + L = S n +1 + 2 S n + S n −1
0 2  1  0  1   0 
.
Deci numerele S n verifică recurenţa: S n +1 = −(S n + S n −1 ) . Deoarece S1 = 0 ,
S 2 = −1 , S 3 = 1 , proprietatea rezultă adevărată prin inducţie după n , ţinând
seama de secvenţa găsită.

9.2.5. Calculul unor sume cu combinări


Să se demonstreze următoarele identităţi cu combinări:
p q n + k   n  n 
(1) ∑   
p
 =    ;
k = 0  k  k  p + q   p  q 
p q n + p + q − k   n + p  n + q 
(2) ∑   
p
 =    ;
k k
k = 0    p+q   p  q 
2 2
n + 2p − q n + p
(3) ∑  
p
p
  =   (Formula lui Li-Jen-Shu)
k =0  k   2p   p 

(4) ∑ (− 1)   = (− 1)n ⋅ (3n )3! (Formula lui Dixon)


3
2n 2n k

k =0 k  (n!)
(5) 1 − 1 + 1 − 1 + L + 1 = 1 , n ∈ N *
 2n   2n   2n   2n   2n  n + 1
         
1 2 3 4  2n − 1

Soluţii:
(1) Obţinem:
 p  q  n + k   p  q  k  k  n   n   p  q  k 
∑  k  k  p + q  ∑  k  k ∑  j  p + q −
 
k ≥ 0   
  =  = ∑ 
j  j ≥0  p + q −
∑    
j  k ≥0  k  k  j 
 k ≥0    j =0  

m
 p  q   p + q
ţinând seama de identitatea ∑  k  m − k  =   . Mai deducem:
k =0     m 
 p  q  k   p q!  p  q  q − j   q  p + q − j
∑  q  k  j  = ∑  k  (q − k )! j!(k − j )! = ∑  k  j  q − k  =  j  q

k ≥0     k ≥0   k ≥0       

127
Deci membrul stâng al identităţii de demonstrat devine egal cu:
 n  q  p + q − j  n!
∑     = ∑ =
j ≥0  p + q − j  j  q  j ≥0 (n − p − q + j )!(q − j )!( p − j )! j!
 n  p  n − p   n   p  n − p   n  n 
= ∑     =  ∑    =    ţinând iarăşi seama de:
j ≥0  p  j  q − j   p  j ≥0  j  q − j   p  q 
m
 p  a   p + q 
∑  
k =0  k  m − k 
 =  
 m 
(2) Identitatea (1) este o egalitate între două polinoame în n , de gradul
p + q . Această egalitate are loc pentru orice n număr natural, deci cele două
polinoame în n sunt identice. Rezultă că obţinem o egalitate dacă atribuim lui
n pe (−1 − n ) .
−1− n + k   n + p + q − k  − 1 − n  n + p
Deoarece   = (− 1) p + q    = (− 1) p  
 p+q   p+q  p   p 
−1− n n + a
şi   = (− 1)q   din (1) deducem (2 ) .
 q   q 
(3) Formula lui Li-Jen-Shu se deduce din (2) pentru q = p.
(4) Vom demonstra identitatea:
3
 2n  k
2n n
 2k  2n  2n + k 
∑ 
k   x = (1 + x )2n
+ ∑    (1 + x )2 n−2 k x k
k =0   i =1  k  2k  k 
de unde pentru x = −1 se obţine formula lui Dixon, adică:

(− 1) ⋅   = (− 1)n    = (− 1)n ⋅ (3n )3! .


3
2n 2n 2n 3n

k

k =0 k   n  n  (n!)
Egalând coeficienţii lui x p în identitatea pe care trebuie să o
demonstrăm, pentru 1 ≤ p ≤ 2n , găsim egalitatea:
3
 2n   2n  n  2k  2n  2n + k  2n − 2k 
  =   + ∑      .
 p   p  k =1  k  2k  k  p − k 
 2k  2n  2n − 2k   p  2n − p  2 p 
Dar     =     , deci identitatea
 k  2k  p − k   k  k  k 
precedentă pe care trebuie să o demonstrăm se reduce la:

128
2
 2n  n
 p  2n − p  2n + k 
  = ∑     (*)
p
  k = 0 k
  k  k 
pentru orice p = 1, K ,2n. Pentru a demonstra (*) vom utiliza identitatea:
p
 p  q  p−k k p
 p  q + k 
∑   
 k  k a b = ∑   (a − b ) p−k b k
k =0    k
k =0   k 
în care punem I: a = 0 , b = −1 şi q = 2n şi II: a = x , b = 1 + x şi
q = 2n − p .
p  2n  k  p  2n + k 
p
În cazul I deducem: (* *) (− 1)   = ∑ (− 1)    iar în cazul I
 p  k =0  k  k 
p
 p  2n − p  p −k p
 p  2n − p + k  2n + k 
se obţine: ∑    x (1 + x )k = ∑ (− 1) p−k ⋅     .
k =0  k  k  k =0  k  k  p 
 2n − p + k  2n + k   2n + k  2n 
Dar    =    , deci putem scrie:
 k  p   k  p 
p
 p  2n − p  2n + k  p  2n 
p
k  p  2n + k 
∑   
 k  k  k   = (− 1)  p ∑ (− 1)  k  k 
 
k =0       k =0   
2
 2n 
Ţinând seama de (* *) membrul drept este tocmai   ceea ce
 p
demonstrează, prin urmare, formula lui Dixon.
(5) Pentru demonstraţia acestei identităţi să observăm că pentru orice
număr natural n ≥ 1 , şi orice k = 0, 1 are loc egalitatea
1 1 k!(n − k )! k!(n − 1 − k )! k!(n − k )!− k!(n − 1 − k )! n
− = − = =
 n  n −1 n! (n − 1)! n!
   
k   k 
k!(n − 1 − k )!(n − k − n ) k!(n − 1 − k )!(− k − 1 + 1) 1 1 1
= = =− +
n! n!  n  n  n − 1
   
 k + 1  k 

129
n
1
Fie xn = ∑ (− 1)
k
. Ţinând cont de egalitatea de mai sus, pentru orice
k =0 n
 
k 
n ∈ N , n ≥ 1 avem:
   
   
k 1 1  k 1 
n −1 n −1
1 1
= ∑ (− 1)  + (− 1) = ∑ (− 1)  −
n
xn − xn−1 − + ⋅ +
 n   n − 1   n  n  n − 1 
k =0
    k =0
   
  k   k     k + 1 
        k 

n −1
1 1 n−1 1
+ (− 1) = ∑ (− 1) + ∑ (− 1) + (− 1) =
n k +1 k n

k =0  n  n k =0  n − 1
   
 k + 1  k 

(xn−1 ) + 1 ⋅ xn−1 + (− 1)n .


n
Din această egalitate deducem că:

xn−1 =
[
n 1 − (− 1)
n
 ]
0, n = 2 p
=  2n ,
n +1  n + 1 , n = 2 p + 1

1
deci 2 − xn = adică identitatea ce trebuia demonstrată.
n +1

130
10. Probleme de geometrie în spaţiu

Probleme rezolvate

R10.1.1. Fie ABCD un tetraedru şi A′ centrul de greutate al feţei BCD. Atunci


are loc relaţia
9 AA′2 = 3( AB 2 + AC 2 + AD 2 ) − ( BC 2 + CD 2 + DB 2 ) .
(Teorema medianei în spaţiu)
Soluţie. Avem
uuur 1 uuur uuur uuur
AA′ = ( AB + AC + AD) , deci
A 3
uuur uuur 1 uuur uuur uuur
AA′2 = AA′ ⋅ AA′ = ( AB + AC + AD) 2 =
9
1 uuur uuur uuur uuur
D = ( AB 2 + AC 2 + AD 2 + 2 AB ⋅ AC + 2 AB ⋅ AD +
9
A′ uuur uuur 1
B C
+2 AC ⋅ AD) = ( AB 2 + AC 2 + AD 2 + AB 2 + AC 2 −
9

BC 2 + AB 2 + AD 2 − BD 2 + AC 2 + AD 2 − CD 2 ) =

1
= (3 AB 2 + 3 AC 2 + 3 AD 2 − BC 2 − BD 2 − CD 2 )
9
de unde rezultă concluzia dorită.
R10.1.2. Fie ABCD un tetraedru. Să se arate că are loc relaţia:
uuur uuur AD 2 + BC 2 − AC 2 − BD 2
cos( AB, CD ) = .
2 AB ⋅ CD
Soluţie. Din definiţia produsului scalar avem:
uuur uuur uuur uuur uuur uuur uuur uuur uuur
uuur uuur AB ⋅ CD ( AD + DB ) ⋅ CD DA ⋅ DC − DB ⋅ DC
cos( AB, CD ) = = = =
AB ⋅ CD AB ⋅ CD AB ⋅ CD
1  DA2 + DC 2 − AC 2 DB 2 + DC 2 − BC 2  AD 2 + BC 2 − BD 2 − AC 2
=  − =
AB ⋅ CD  2 2  2 AB ⋅ CD
,
de unde rezultă concluzia problemei.
R10.1.3. Să se arate că dacă un tetraedru are două perechi de muchii opuse
perpendiculare, atunci şi a treia pereche are această proprietate (tetraedrul se
numeşte ortogonal sau ortocentric).

131
Soluţie. Fie tetraedrul ABCD astfel încât AD ⊥ BC şi AC ⊥ BD .
Metoda I. Trebuie să demonstrăm că AB ⊥ DC . Din ipoteză avem:
uuur uuur uuur uuur
AD ⋅ BC = 0 şi AC ⋅ BD = 0 .
Deoarece
A uuur uuur uuur uuur uuur uuur uuur uuur uuur
AB ⋅ DC = ( AC + CB) ⋅ DC = AC ⋅ DC + CB ⋅ DC =
uuur uuur uuur uuur uuur uuur
= AC ⋅ ( DB + BC ) + CB ⋅ ( DA + AC ) =
uuur uuur uuur uuur uuur uuur uuur uuur
= AC ⋅ DB + AC ⋅ BC + CB ⋅ DA + CB ⋅ AC =
ip. uuur uuur uuur uuur uuur uuur uuur
D = AC ⋅ BC + CB ⋅ AC = BC ⋅ ( AC − AC ) = 0 ,
rezultă AB ⊥ DC , adică are loc concluzia
problemei.
C
B Metoda a II-a. Din AD ⊥ BC şi AC ⊥ BD , conform problemei
precedente avem
uuur uuur AB 2 + DC 2 − AC 2 − BC 2
cos( AD, CB ) = = 0 şi
2 AD ⋅ CB
uuur uuur AB 2 + DC 2 − AD 2 − BC 2
cos( AC , DB ) = =0.
2 AC ⋅ DB
Prin urmare, obţinem: AB 2 + DC 2 = AC 2 + BD 2 şi AB 2 + DC 2 = AD 2 + BC 2 .
uuur uuur
Rezultă: AD 2 + BC 2 = AC 2 + BD 2 , deci cos( AB, CD ) = 0 , adică AB ⊥ CD .
R10.1.4. Într-un tetraedru ortogonal suma pătratelor muchiilor opuse este
aceeaşi.
A Soluţie. Fie tetraedrul ortogonal ABCD. Din
uuur uuur uuur uuur r uuur uuur uuur uuur
AB + BC + CD + DA = 0 ⇒ BC + DA = BA + DC .
Prin ridicare la pătrat, rezultă că
uuur uuur uuur uuur ip.
BC 2 + DA2 + 2 BC ⋅ DA = BA2 + DC 2 + 2 BA ⋅ DC ⇒
D ⇒ BC 2 + DA2 = BA2 + DC 2
(1)
Analog, din
uuur uuur uuur uuur r uuur uuur uuur uuur
AC + CB + BD + DA = 0 ⇒ CB + DA = CA + DB
B C şi apoi

uuur uuur uuur uuur uuur uuur


(CB + DA) 2 = (CA + DB) 2 ⇒ CB 2 + DA2 + 2CB ⋅ DA =
uuur uuur
= CA2 + DB 2 + 2CA ⋅ DB ⇒ CB 2 + DA2 = CA2 + DB 2

132
(2)
Din relaţiile (1) şi (2) rezultă concluzia dorită.
R10.1.5. Fie [ AA1 A2 A3 ] un tetraedru oarecare şi B1 , B2 , B3 mijloacele
muchiilor A2 A3 , A1 A3 şi respectiv A1 A2 . Se notează cu Mi punctele de
intersecţie cu ABi , i ∈ {1, 2,3} , ale unui plan paralel cu planul ( A1 A2 A3 ) . Să se
arate că dreptele Ai M i , i = 1,3 , sunt concurente într-un punct pe AG, unde G
este centrul de greutate al triunghiului A1 A2 A3 .

Soluţie. O condiţie necesară şi suficientă ca dreptele A1M 1 , A2 M 2 să fie


uuuuur uuuuuuur
coplanare este ca vectorii legaţi A1 A2 şi M 1M 2 să fie paraleli.
Deoarece
( A1 A2 A3 ) ( M 1M 2 M 3 ) există
λ∈ *
astfel încât
uuuuur uuuur λ uuuur uuuur
AM 1 = λ ⋅ AB1 = ( AA2 + AA3 )
2
şi
uuuuur uuuur λ uuur uuuur
AM 2 = λ ⋅ AB2 = ( AA1 + AA3 )
2
.
Rezultă că
uuuuuuur uuuuur uuuuur
M 1M 2 = M 1 A1 + AM 2 ) =
λ uuuur uuur λ uuuuur
= ( A2 A + AA1 ) = ⋅ A2 A1 ,
2 2
de unde obţinem că
M 1M 2 A1 A2
şi cum A1M 1 nu poate fi paralelă cu A2 M 2 rezultă că există
{P} = A1M 1 I A2 M 2 . Vom demonstra că P ∈ A3 M 3 . Într-adevăr
AP AA
A1 PA2 M 1 PA2 şi deci 1 = 1 2 .
PM 1 M 1M 2
uuur 2 uuuuur
Deoarece N1 N 2 = 2 M 1M 2 şi N1 N 2 = λ ⋅ A1 A2 , rezultă că A1 P = PM 1
λ
uuur uuur 2 uuur uuuuur uuur λ uuur uuuur uuuur
sau A1 A + AP = ⋅ ( PA + AM 1 ) şi deci AP = ⋅ ( AA1 + AA2 + AA3 ) .
λ λ +2

133
Deoarece G este centrul de greutate al A1 A2 A3 avem
uuur 1 uuur uuuur uuuur uuur 3λ uuur
AG = ⋅ ( AA1 + AA2 + AA3 ) şi urmează că AP = ⋅ AG . De aici rezultă că
3 λ +2

P ∈ AG şi P împarte segmentul AG în raportul .
λ+2
Repetând raţionamentele şi calculele pentru dreptele A2 M 2 şi A3 M 3
găsim că P ∈ A3 M 3 , deci dreptele AM i , i = 1,3 , sunt concurente.
R10.1.6. Pe muchiile AB, BC, CD, DA ale unui tetraedru se consideră
punctele M, P, N, Q astfel încât
BP AQ CN BM
= = = =k.
PC QD ND MA
Să se determine T ∈ ( AC ) şi S ∈ ( BD) astfel încât MN, PQ şi TS să fie
concurente.
Soluţie. Din ipoteză rezultă că MP AC şi NQ AC , deci MP NQ ,
de unde obţinem că MPNQ trapez.
Avem:
A
BP BP ip. BP k MP
= = = = ;
BC BP + PC BP + BP k + 1 AC
Q k

DQD DQ ip. DQ 1 NQ
= N = = =
M DA DQ + QA DQ + k ⋅ DQ k + 1 AC
C Fie {O} = MN I PQ . Atunci MN, PQ, TS
sunt concurente ⇔ (∃) k1 , k2 ∈ astfel
B uuur uuur uuur
P încât AO = k ⋅ AT + k ⋅ AS şi k + k = 1
1 2 1 2
(1)
MO MP
Avem: =k = şi
ON NQ
uuuur uuur
uuur AM + k AN
AO = ;
1+ k
uuur uuur
uuuur 1 uuur uuur AC + k ⋅ AD
AM = ⋅ AB ; AN = .
k +1 1+ k

134
uuur uuur
AT SD uuur α uuur uuur AD + β ⋅ AB
Fie =α , = β . Obţinem AT = ⋅ AC , AS = .
TC SB α +1 1+ β
2k 1+ k 2 1
Folosind şi relaţia (1), deducem că k1 = , k 2 = , α = 1, β = 2 .
(1 + k ) 2
(1 + k ) 2
k
R10.1.7. În tetraedrul ABCD din două vârfuri se duc perpendiculare pe feţele
opuse ce se intersectează în O. Să se arate că perpendicularele duse din O pe
celelalte două feţe intersectează feţele în ortocentrele lor.
Soluţie. Fie AE ⊥ ( BCD) şi CF ⊥ ( ABD) cu AE I CF = {O} .
A Avem:
uuur uuur
AO ⊥ ( BCD) ⇒ AO ⋅ BC = 0
uuuur uuur ⇒
OH1 ⊥ ( ABC ) ⇒ OH1 ⋅ BC = 0
uuur uuur uuuur uuur uuur uuuur uuuur
F⇒ AO D⋅ BC + OH1 ⋅ BC = 0 ⇒ ( AO + OH1 )⋅BC = 0 ⇒
uuuur uuur
O ⇒ AH1 ⋅ BC = 0 ⇒ AH1 ⊥ BC
(1)
H1 E uuur uuur
B C CO ⊥ ( ABD) ⇒ CO ⋅ AB = 0
Din uuuur uuur ⇒
OH1 ⊥ ( ABC ) ⇒ OH1 ⋅ AB = 0
uuur uuur uuuur uuur
⇒ CO ⋅ AB + OH1 ⋅ AB = 0 ⇒
uuur uuuur uuur uuuur uuur
⇒ (CO + OH1 ) ⋅ AB = 0 ⇒ CH1 ⋅ AB = 0 ⇒ CH1 ⊥ AB
(2)
Din (1) şi (2) rezultă H1 este ortocentrul ABC .
Analog, dacă OH 2 ⊥ ( ADC ) , atunci H 2 este ortocentrul triunghiului ADC.
R10.1.8. Fie [ ABCDA′B′C ′D′] o prismă oblică cu baza patrulater oarecare şi
fie punctele M ′ ∈ ( A′D′) , N ∈ ( BC ) , P′ ∈ ( D′C ′) , Q ∈ ( AB) , astfel încât
A′M ′ BN D′P′ AQ
= = x şi = = y.
A′D′ BC D′C ′ AB
Să se demonstreze că dreptele M ′N şi P′Q sunt concurente dacă şi
numai dacă x + y = 1 .
Soluţie. Fie punctele M ∈ ( AD) ,
N ′ ∈ ( B′C ′) , P ∈ (CD) , Q′ ∈ ( A′B′) astfel încât MM ′ NN ′ PP′ AA′ QQ′ .
Fie {O} ∈ MN I PQ şi {O′} = M ′N ′ I P′Q′ .

135
Dreapta OO′ este P’ C’
intersecţia planelor D’
( MNN ′M ′) şi ( PQQ′P′) . M’ O’
Fie {E} = M ′N I OO′ şi N’
A’
fie {F } = P′Q I OO′ . Q’
B’
AM
Avem =x şi F
AD
DP E
= y.
DC D P C
Se cunoaşte următoarea M
proprietate din geometria O
N
plană: A
Fie patrulaterul
ABCD şi punctele Q
B
M ∈ ( AD) , N ∈ BC ,
AM BN DP AQ
P ∈ (CD) , Q ∈ ( AB) astfel încât = = x şi = = y . Fie
AD BC DC AP
QO MO
{O} = MN I PQ . Atunci = x şi = y.
QP MN
OF QO OF
OFQ PP′Q(u − u ) ⇒ = = x⇒ =x
PP′ QP OO′
EO′ M ′O′ MO OE ′ OE
EM ′O′ NM ′N ′(u − u ) ⇒ = = = y⇒ = y⇒ = 1− y
NN ′ M ′N ′ MN OO′ OO′
.
Dreptele M ′N şi P′O sunt concurente ⇔
E = F ⇔ OE = OF ⇔ x = 1 − y ⇔ x + y = 1
R10.1.9. Să se demonstreze că dacă bisectoarele a două unghiuri plane ale
unui triedru sunt perpendiculare, atunci
O
r r bisectoarea celui de-al treilea unghi plan este
v1 v2 perpendiculară pe primele două bisectoare.
r Soluţie. Fie triedrul determinat de
v3
semidreptele [OA , [OB şi [OC , (OABC
este un tetraedru). Pe cele trei semidrepte,
r r r
considerăm vectorii unitari v1 , v2 , v3 ca în
B′ C figura alăturată. Direcţiile bisectoarelor
A r r r r
A′ considerate sunt date de v1 + v3 , v2 + v3 şi
C′ B r r
respectiv v1 + v2 . ([OB′ bisectoarea unghiului

136
AOC , [OA′ bisectoarea unghiului BOC iar [OC ′ bisectoarea unghiului
AOB ).
Presupunem că OA′ ⊥ OB′ şi demonstrăm că OC ′ ⊥ OA′ şi OC ′ ⊥ OB′ . Din
r r r r r r r r r r
OA′ ⊥ OB′ ⇒ (v2 + v3 ) ⋅ (v1 + v3 ) = 0 ⇒ v1 ⋅ v2 + v1 ⋅ v3 + v2 ⋅ v3 + 1 = 0
(1)
Avem
r r r r r r r r r r (1)
(v1 + v2 ) ⋅ (v1 + v3 ) = 1 + v1 ⋅ v3 + v1 ⋅ v2 + v2 ⋅ v3 = 0 ⇒
r r r r
⇒ (v1 + v2 ) ⊥ (v1 + v3 ) ⇒ OC ′ ⊥ OB′ .
Analog OC ′ ⊥ OA′ .
R10.1.10. Pe muchiile DA, DB şi AC ale unui tetraedru DABC se iau respectiv
punctele L, N, F astfel încât
uuur 1 uuur uuuur 1 uuur uuur 1 uuur
DL = DA , DN = DB , AF = AC .
2 3 4
În ce raport, planul ce trece prin punctele L, N, F împarte muchia BC.
uuur r uuur r uuur r
Soluţie. Fie DA = a , DB = b , şi DC = c . Avem:
uuur 1 r uuuur 1 r uuur 1 r r
D DL = a , DN = b , AF = (− a + c ) .
2 3 4
Fie {M } = ( LNF ) I BC . Atunci punctele L, F,
N cr M, N sunt coplanare ⇔ (∃) m, n ∈ astfel încât
L uuuur uuur uuur
r LM = m ⋅ LF + n ⋅ LN
r b
a (1)
C
Deoarece:
A F M uuur uur uuur 1 r 1 r r 1 r 1 r
LF = LA + AF = a + (− a + c ) = a + c ;
B 2 4 4 4
uuur uuur uuuur 1 r 1 r
LN = LD + DN = − ⋅ a + ⋅ b ;
2 3
uuuur uur uuur uuuur 1 r r r uuur 1r r r r
LM = LA + AB + BM = a + (− a + b ) + λ ⋅ BC = − a + b + λ (−b + c )
2 2
utilizând relaţia (1) obţinem
1r r r m r r  1 r 1 r
− a + (1 − λ )b + λ c = (a + c ) + n  − a + b  ⇔
2 4  2 3 

137
 1 m n
− 2 = 4 − 2
r 
1r r m nr n r m r  n
⇔ − a + (1 − λ )b + λ c =  −  a + b + c ⇔ 1 − λ = ⇔
2  4 2 3 4  3
 m
λ = 4

 8
m = 5
 m = 4λ 
  9
⇔ n = 3 − 3λ ⇔ n = .
−2 = 4λ − 6 + 6λ  5
  2
λ = 5

2 BM 2 BM 2 MC 5 MB 2
Rezultă BM = BC ⇒ = ⇒ = ⇒ 1+ = ⇒ =
5 BC 5 BM + MC 5 MB 2 MC 3
şi demonstraţia se încheie.
R10.1.11. Fie MABC un tetraedru oarecare uuuur şi M ′ un punct în interiorul
r
triunghiului ABC. Notăm rX , vectorul MX (vectorul de poziţie al punctului X
în raport cu originea M). Atunci are loc relaţia
r r r
r s A ⋅ rA + sB ⋅ rB + sC ⋅ rC
rM ′ = ,
s
(1)
unde s A , sB , sC sunt ariile triunghiurilor BM ′C , CM ′A , AM ′B respectiv ABC.
Soluţie. Avem
r r
r rA + k ⋅ rA′ M ′A
M rM ′ = , unde k = ,
1+ k M ′A′
iar { A′} = AM ′ I BC . Rezultă
AA′ s
k +1 = = , deci
M ′A′ s A
r  s r
C rA +  − 1)  rA′ r r
r  sA  s A ⋅ rA + ( sB + sC )rA′
rM ′ = =
A M′ A′ s s
sA
B .
r r
r rB + k ⋅ rC
Pe de altă parte, rA′ = , unde
1+ k

138
r r
A′B sC r sB ⋅ rB + sC ⋅ rC
k= = . Rezultă că rA′ = , deci
A′C sB sB + sC
r r r
r s A ⋅ rA + sB ⋅ rB + sC ⋅ rC
rM ′ = .
s
R10.1.12. Fie MABC un tetraedru oarecare şi M ′ un punct în interiorul
triunghiului ABC. Notăm cu s A , sB , sC , s ariile triunghiurilor
BM ′C , CM ′A , AM ′B respectiv ABC. Atunci are loc relaţia:
MA2 ⋅ s ⋅ s A + MB 2 ⋅ s ⋅ sB + MC 2 ⋅ s ⋅ sC − ( AB 2 ⋅ s A ⋅ sB + BC 2 ⋅ sB ⋅ sC + CA2 ⋅ sC ⋅ s A ) =

= s 2 ⋅ MM ′2
(extinderea la tetraedru a relaţiei lui Stewart).
Soluţie. Înmulţind scalar fiecare membru al relaţiei (1) din problema
precedentă cu el însuşi obţinem
uuur uuur
s 2 MM ′ = s A2 ⋅ MA2 + sB2 ⋅ MB 2 + sC2 ⋅ MC 2 + 2 s A ⋅ sB ⋅ MA ⋅ MB +
uuur uuuur uuuur uuur
+2 sB ⋅ sC ⋅ MB ⋅ MC + 2 sC ⋅ s A ⋅ MC ⋅ MA ⇔
⇔ s 2 ⋅ MM ′2 = s A2 ⋅ MA2 + sB2 ⋅ MB 2 + sC2 ⋅ MC 2 + 2s A ⋅ sB ⋅ MA ⋅ MB cos α +
+2 ⋅ sB ⋅ sC ⋅ MB ⋅ MC cos β + 2sC ⋅ s A ⋅ MC ⋅ MA cos γ ,
unde α = m ( BMA) , β = m(CMB) , γ = m( AMC ) .
Aplicând teorema cosinusului în triunghiurile AMB, BMC şi CMA
obţinem:
s 2 MM ′2 = s A2 ⋅ MA2 + sB2 ⋅ MB 2 + sC2 ⋅ MC 2 + s A ⋅ sB ( MA2 + MB 2 − AB 2 ) +
+ sB ⋅ sC ( MB 2 + MC 2 − BC 2 ) + sC ⋅ s A ( MC 2 + MA2 − AC 2 ) ⇔
⇔ s 2 ⋅ MM ′2 = MA2 ⋅ s A ( s A + sB + sC ) + MB 2 ⋅ sB ( sB + s A + sC ) +

+ MC 2 ⋅ sC ( sC + sB + s A ) − ( AB 2 ⋅ s A ⋅ sB + BC 2 ⋅ sB ⋅ sC + CA2 ⋅ sC ⋅ s A ) ,
de unde, utilizând faptul că s = sB + sC + s A se obţine relaţia dorită.
s A′C sC A′B
Observaţii. 1) Dacă M ′ = A′ , atunci s A = 0 , B = , = şi se
s BC s BC
obţine relaţia lui Stewart în triunghiul BMC.
2) Relaţia din concluzia problemei este echivalentă cu
s s s  s ⋅s s ⋅s s ⋅s 
MM ′2 = MA2 A + MB 2 B + MC 2 C −  AB 2 A 2 B + BC 2 B 2 C + CA2 C 2 A 
s s s  s s s 
R10.1.13. Fie B şi C două puncte arbitrare pe cercul C de diametru [OA], iar V
un punct arbitrar pe perpendiculara în O pe planul (OABC). Notăm A1, B1, C1

139
proiecţiile punctului O pe AV, BC respectiv CV. Să se arate că punctele O, A1,
B1, C1 sunt coplanare.
Soluţie.
V Vom arăta că B1 A1 ⊥ VA . Pentru
uuuur uur
aceasta este suficient ca B1 A1 ⋅ VA = 0 .
A1 Avem:
uur uuuur uur uuur uuur
VA ⋅ B1 A1 = VA ⋅ (VA1 − VB1 ) = VA ⋅ VA1 − VB1 ⋅ VA cos BVA =
VB
C1 = VO 2 − VA ⋅ VB1 ⋅ = VO 2 − VO 2 = 0 .
B1 VA
(Am folosit teorema catetei
O A VO 2 = VA ⋅ VA1 = VB ⋅ VB1 şi VB ⊥ BA ,
din teorema celor trei perpendiculare).
B C În mod analog, avem C1 A1 ⊥ VA şi
cum OA1 ⊥ VA , va rezulta
coplanaritatea punctelor O, A1, B1, C1.

Bibliografie

1. Dorin Andrica, Csaba Varga, Daniel Văcăreţu, Teme de geometrie, Editura


Promedia Plus, Cluj-Napoca
2. Dan Brânzei, Sebastian Aniţa, Geometrie, Editura Paralele 45
3. Constantin Avădanei, Neculai Avădanei, Constantin Borş, Cristina Ciurea,
De la matematica elementară spre matematica superioară, Editura
Academiei, Bucureşti, 1987
4. Mihai Cocuz, Culegere de probleme de matematică, Editura Academiei,
Bucureşti, 1984
5. Ariana-Stanca Văcăreţu, Daniel Văcăreţu, O relaţie de tip Stewart în spaţiu
şi câteva aplicaţii, G.M. 2/2002
6. * * * , Revistele de matematică: G.M., R.M.T.

140
11. Criterii de ireductibilitate pentru polinoame

Denumirea polinomului provine din “Elementele” lui Euclid


(sec. al III – lea î.H.) (gr. polys (“multe”), nomos (parte, membru)) fiind
adoptată în sensul algebrei clasice în 1691. În acest capitol polinoamele sunt
polinoame cu coeficienţi întregi iar în continuare vom prezenta aplicaţii la două
criterii de ireductibilitate ale polinoamelor cu coeficienţi întregi.

11.1. Criteriul lui Eisenstein şi criteriul lui Schöneman


11.1.1. Definiţie: Fie f∈Z[X] un polinom de grad n, n ≥1, n∈N. Spunem că f
este reductibil peste Z[X] dacă există două polinoame g,h∈Z[X] de grad mai
mic decât n, astfel încât f = g ⋅ h .
În caz contrar, spunem că f este polinom ireductibil peste Z[X] .
11.1.2. Propoziţie: Orice polinom de gradul întâi din Z[X] este ireductibil
peste Z[X].
Demonstraţie: Fie f∈Z[X], f = aX+b, a≠0. Dacă f ar fi reductibil peste Z[X], ar
exista g, h∈Z[X], astfel încât f = g ⋅ h , unde grad g <1 şi grad h <1. Cum g şi h
nu pot fi polinoame nule, rezultă gradg = gradh = 0 şi atunci obţinem
grad f = grad( g ⋅ h )=gradg+gradh sau 1=0, fals. Deci polinomul f este
ireductibil peste Z[X].
11.1.3. Criteriul lui Eisenstein:
Fie f∈Z[X], f = a0+a1X+…+anXn, n ≥1, n∈N, (a0, a1,.., an)=1.
Presupunem că există un număr prim p astfel încât p | ai, (∀) i =0,1,2, …, n-1,
p nu divide an şi p nu divide a0. Atunci f nu se poate scrie ca produsul a două
polinoame cu coeficienţi întregi.
Demonstraţie: Presupunem că f = g ⋅ h , g = b0+b1X+…+brXr,
h = c0+c1X+…+csXs, g,h∈Z[X]. Avem a 0 = b0 ⋅ c0 , a1 = b1 ⋅ c 0 + b0 ⋅ c1 ,
a 2 = b2 ⋅ c0 + b1 ⋅ c1 + b0 ⋅ c 2 ,…, am=bmc0+bm-1c1+…+ b0cm. Cum p | a0 şi p2 | a0
rezultă p | b0c0 rezultă p | b0 şi p nu divide c0. Din p | a1 avem p | b1c0 deci p | a2
adică p | b2c0 prin urmare p | b2 . Continuând din p | ar (r ≤ n-1) rezultă p | br
deci p | b0, contradicţie.

11.1.4. Definiţie : Fie Z mulţimea numerelor întregi şi n ∈ N * un număr natural


fixat. Pe mulţimea Z definim: x ≡ y (mod n ) ⇔ ( x − y )M n ( citim : x este
congruent cu y modulo n ). Prin clasa de echivalenţă a lui x în raport cu ,, ≡ ,, se

141
înţelege C ( x ) = x = {y ∈ Z y ≡ x (mod n )} . Relaţia de congruenţă modulo n pe

∧ ∧ ∧

Z determină mulţimea cât Z n = 0, 1,K , n − 1 numită mulţimea claselor de
 
resturi modulo n.

11.1.5. Criteriul de ireductibilitate al lui Schöneman


Fie p≥2 un număr prim, iar f∈Z[X] un polinom de forma f = gn + ph,
unde g,h∈Z[X], n∈N*, f având coeficientul dominant egal cu 1. Dacă g este
ireductibil în Zp[X] şi g nu divide pe h , atunci f este ireductibil în Z[X].

Demonstraţie: Dacă g = a0+a1X+a2X2+…+anXn∈Z[X], iar p≥2, p număr prim,


atunci g = a 0 + a1 X+ a 2 X2+…+ a n Xn∈Zp[X], unde ai este clasa lui ai modulo
p şi se numeşte polinomul redus modulo p al lui g. Se observă că gradh < n
gradg şi g are coeficientul dominant 1. Presupunem prin absurd că polinomul f
ar fi reductibil. Deci există f1, f2∈Z[X] astfel încât gradf1, gradf2 ≥ 1 şi f = f1 f2 .
Trecem la polinoamele reduse modulo p şi obţinem f1 ⋅ f 2 = f = g − n . Cum g
n1 n2
este ireductibil în Z[X], rezultă f1 = g , f 2 = g unde n1+n2 = n. Deci
f1= g n 1 + p h1, unde f2= g n 2 + p h2 , h1, h2∈Z[X], gradhi< ni gradg (deoarece f
are coeficientul dominant 1). Deci gn + p h = ( g n 1 +p h1).( g n 2 + ph2), de unde
obţinem h = g n 2 h1+ g n 1 h2+p h1h2 . Dacă n1,n2 > 0, atunci, din ultima egalitate
rezultă că g ⋅ s = h , cu s∈Z[X], deci g ar divide pe h, contradicţie. Atunci n1=n
sau n2=n, adică gradf1 = gradf2 sau gradf2=gradf, deci f este ireductibil în Z[X].

11.2. Aplicaţii ale criteriilor de ireductibilitate ale lui Eisenstein şi


Schöneman

11.2.1. Aplicaţii ale criteriului lui Eisenstein:

(1) Dacă p este un număr natural prim, atunci polinomul


p −1 p−2
f =X +X + L + X + 1 este ireductibil peste Z[X].

p −1 p−2 X p −1
Demonstraţie: Dacă f = X +X +L+ X +1 = sau dezvoltând
X −1
p p ( p − 1)
X p
= 1+ ( X − 1) + ( X − 2) 2 + ... + ( X − 1) p , iar
1! 2!
142
p( p − 1)
f = ( X − 1) p −1 + p( X − 1) p − 2 + ( X − 1) p −3 + ... + p
2!
p
pe X

 p
Coeficienţii   , k ≤ p − 1 sunt divizibili prin p şi are loc criteriul lui
k 
Eisenstein.
(2) Să se arate că polinomul f = Xn-120, n ≥1 nu se poate descompune în
produsul a două polinoame cu coeficienţi întregi.

Demonstraţie: Se aplică criteriul lui Eisenstein pentru p = 3, a0 = -120,


a1=…= an-1=0, an=1, iar 3 | 120, 3 | 0, 3 nu divide pe 1 şi 9 nu divide pe 120
deci f este ireductibil în Z[X].

11.2.2. Aplicaţii ale criteriului lui Schöneman :

(1) Fie p un număr prim de forma p = 4 k+3, iar a, b∈Z astfel încât p
divide pe a, p divide (b-1) şi p2 nu divide (b-1). Să se arate că
polinomul f = X2p+ a X+ b este ireductibil în Z[X]
(L.Panaitopol, D. Ştefănescu)

Soluţie: Fie f = (X2+1)p+ ph

1   p 2 p −1  p 2 p−2 p  2 
h = cX + d + −  ( X + 1) +  ( X + 1) + ... +  ( X + 1)
p  1  2   p − 1 
a b −1
c= ,d = . Atunci f = X2+1 este ireductibil în Z[X] iar h nu se divide
p p
prin g = X2+1 în Z[X] deoarece cx+d≠0 nu se divide prin g = X2+1 în
Z[X]. Din criteriul lui Schöneman rezultă că f este ireductibil în Z[X].

(2) Să se arate că polinomul f = (X2+2)n + 5 (X2n-1+10 X2+5) este


ireductibil în Z[X].
(L.Panaitopol, D. Ştefănescu)

Soluţie: Fie g = X2+2, h = X2n-1+ 10 Xn + 5 şi prin urmare g =X2+2 este


ireductibil în Z5[X], iar h =X2n-1 nu se divide prin X2+2 în Z5[X], deci, conform
criteriului lui Schöneman f este ireductibil în Z[X].

143

S-ar putea să vă placă și